Medcoffee 2 Flashcards

1
Q

Cervical sympathetic chain damage will cause?
sweating

decrease hearing

shoulder drop

increased papillary constriction

dec. pupil response

A

D) Reason

Miosis (pupil constriction)Associated with lesion of spinal cord above T1 (e.g., Pancoast tumor, Brown-Séquard syndrome [cord hemisection], late-stage syringomyelia).

How well did you know this?
1
Not at all
2
3
4
5
Perfectly
2
Q

maximum oxygen concentration in fetal blood is in?
umblical artery

umblical vein

umbilical cord

umbilical vesicle

pulmonary artery

A

B) Reason

During fetal circulation, oxygenated blood from the placenta to the fetus passes through the umbilical vein. Three vascular shunts develop in the fetal circulation to bypass blood flow around the liver and lungs.

How well did you know this?
1
Not at all
2
3
4
5
Perfectly
3
Q

unmyelinated pain receptors?
C fibers

A fibers

B fibers

delta fiber

D fibers

A

A) Reason

■ Fast pain is carried by group III fibers. It has a rapid onset and offset, and is localized.■ Slow pain is carried by C fibers (unmylenated). It is characterized as aching, burning, or throbbing that is poorly localized.

How well did you know this?
1
Not at all
2
3
4
5
Perfectly
4
Q

bronchopulmonary segment is supplied by?
primay bronchioles

secondary broncheoles

tertiary bronchioles

main bronchus

Quaternary bronchioles

A

C) Reason

Bronchopulmonary segments are the anatomic, functional,and surgical units of the lungs. Each lobar (secondary)bronchus, which passes to a lobe of the lung, gives offbranches called segmental (tertiary) bronchi

How well did you know this?
1
Not at all
2
3
4
5
Perfectly
5
Q

immediate action of angiotensin2?
vasoconstrition

Vasodilation

dec. renin
dec. Sympathetic flow
dec. BP

A

A) Reason

There are 4 effects of Angiotensin-II: 1) Increases Aldosterone 2) Increases Na-H exchange at PCTs (3) It increases thirst and therefore water intake.(4) It causes vasoconstriction of the arterioles, thereby increasing TPR and arterial pressure.

How well did you know this?
1
Not at all
2
3
4
5
Perfectly
6
Q

vitamin required for ALT?
B1

B3

B2

B9

B6

A

E) Reason

FUNCTION: Converted to pyridoxal phosphate, a cofactor used in transamination (e.g., ALT and AST), decarboxylation reactions, glycogen phosphorylase.

How well did you know this?
1
Not at all
2
3
4
5
Perfectly
7
Q

danger area of scalp containing vessels connecting emissary vein?
calvaria

loose conective tissue layer

pia matter

dura matter

Arachnoid

A

B) Reason

• Emissary veins are valveless channels that course through the bones of the skull and allow dural sinuses to communicate with extracranial veins. They can spread infection to dural sinuses when loose connective tissue layer is damaged

How well did you know this?
1
Not at all
2
3
4
5
Perfectly
8
Q

cortisol decreases ?
gluconeogenesis

peripheral utilization of glucose

glycogenolysis

pigmentation

pain threshold

A

B) Reason

Funtions: Increases Insulin resistance (diabetogenic)leading to decreased glucose use in periphral tissue.Increases Gluconeogenesis, lipolysis, and proteolysisDecreases ibroblast activity (causes striae) Decreases Inflammatory and Immune responses:

How well did you know this?
1
Not at all
2
3
4
5
Perfectly
9
Q

posterior cervical lymph node biopsy,nerve most likely to damage is?
CN XI

CN IX

CN X

spinal part of CN IX

CN XII

A

D) Reason

The spinal root of the accessory nerve arises from the nervecells in the anterior gray column of the upper five segmentsof the cervical part of the spinal cord .

How well did you know this?
1
Not at all
2
3
4
5
Perfectly
10
Q

About immunodeficiency conditions
The AIDS is secondary to HIV

The swiss type agammaglobulinemia is hereditary and deficient in both humoral as well as in cellular immunity

The bruton”s disease is an aplasia of T lymohocytes.

The multiple myloma is associatd with hypegammaglobulinemia

They may be primary to anemia, malnutrition, and metastases of bone

A

B) Reason

This is severe combined immunodeficiency deficiency that affects both B & T-cells.

How well did you know this?
1
Not at all
2
3
4
5
Perfectly
11
Q

A diabetic patient with ketoacidosis is being treated ,which will potentiate hypokalemia?
insulin

epinephrine

ampicillin

glucagon

cortisol

A

A) Reason

intracellular K+ comes out in exchange for H+ moving into the cells. Hence care should be taken while correcting acidosis which may lead to sudden hypokalemia. This is more likely to happen in treating diabetic ketoacidosis by giving glucose and insulin together.

How well did you know this?
1
Not at all
2
3
4
5
Perfectly
12
Q

Afebrile young patient presents with signs and symptoms of increased intracranial pressure ,what is probable cause?
miningitis

encephalitis

hydrocephalus

alzehmer disease

Parkinson”s

A

C) Reason

decreased CSF absorption or increased production by arachnoid granulations, which can lead to increased intracranial pressure, papilledema, and herniation .

How well did you know this?
1
Not at all
2
3
4
5
Perfectly
13
Q

scabies itching due to HSR of mice
proteins

lipids

feaces

polysacchride coat

wings

A

C) Reason

The pruritic lesions result from a delayed hypersensitivity reaction to the feaces of the mite. The mite is located within the stratum corneum of the epidermis.

How well did you know this?
1
Not at all
2
3
4
5
Perfectly
14
Q

scenario of RTA,maximum blood loss from?
heart

arterioles

arteries

kidneys

vein and venules

A

E) Reason

Veins and venules have more calibre and capacitance than that of arteries and accumulate more blood, and hence blood loss will be max from these vessels.

How well did you know this?
1
Not at all
2
3
4
5
Perfectly
15
Q

connected to desmosome are?
Tubulin

Intermediate filaments

Microfilaments

microtubules

cillia

A

B) Reason

Cadherins span between the cell membranes of desmosomes and internally desmosomes are anchored to intermediate filaments in large bundles called tonofilarnent

How well did you know this?
1
Not at all
2
3
4
5
Perfectly
16
Q

Most commonly associated with smoking among these?
CA bladder

CA esophagus

thromboangitis obilitarans

CA stomach

Churg Strauss

A

C) Reason

Thromboangiitis obliterans ( Buerger disease) is an acute inflammation involving small- to medium-sized a rteries of the extremities, extending to adjacent veins and nerves.1. This disease results in painful ischemic disease. often leading to gangrene. 2. It is clearly exacerbated by heavy cigarette smoking.

How well did you know this?
1
Not at all
2
3
4
5
Perfectly
17
Q

HCO3 is absorbed from?
DCT

collecting ducts

loop of henle

PCT

PCT & DCT

A

D) Reason

Early proximal convoluted tubule (PCT)—contains brush border. Reabsorbs all of the glucose and amino acids and most of the HCO3–, Na+, Cl–, PO4–, K+, and H2O.

How well did you know this?
1
Not at all
2
3
4
5
Perfectly
18
Q

injury to foramen ovale,which part of face is affected?
forehead

upper 1/3rd

middle 1/3rd

scalp

lower 1/3rd

A

E) Reason

The foramen ovale lies posterolateral to the foramen rotundum . It perforates the greater wing of the sphenoid, and it transmits the sensory and the motor roots of themandibular division (supplies the lower 1/3 rd of face) of the trigeminal nerve. It also transmitsthe lesser petrosal nerve .

How well did you know this?
1
Not at all
2
3
4
5
Perfectly
19
Q

a man running on a tread mill heart rate is doubled,effect on cardiac cycle?
0.2sec

  1. 1 sec
  2. 4 sec
  3. 6 sec
  4. 8 sec
A

C) Reason

Cardiac Cycle= 1mint/ number of beats that 60 sec/72= 0.82 sec. As the heart rate becomes double, the cardiac cycle duration will become half that is 0.4sec.

How well did you know this?
1
Not at all
2
3
4
5
Perfectly
20
Q

TM joint stability ligament?
articular surfaces

joint capsule

stylomandibular

stylomastoid

lateral temporomandibular

A

E) Reason

The lateral temporomandibular ligament is attached aboveto the articular tubercle at the root of the zygomatic archand below to the neck of the mandible. The fibers extenddownward and backward. This ligament limits the posteriormovement of the mandible.

How well did you know this?
1
Not at all
2
3
4
5
Perfectly
21
Q

An old man presents with history of bone pains, serum alkaline phosphatase is high and serum calcium is normal .most likely is is
Metastatic bone disease

Multiple myeloma

Osteomalacia

Paget”s disease of bones

Renal osteodystrohy

A

D) Reason

Paget Disease is the localized lesion of the bones in which there is imbalance b/w the osteoblasts and osteoclasts activity but serum Ca remains normal.

How well did you know this?
1
Not at all
2
3
4
5
Perfectly
22
Q

maximum feedback for blood pressure?
baroreceptor

brain bridge reflex

CNS Ischemic response

renin

chemoreceptor

A

A) Reason

■ The most important mechanisms for regulating arterial pressure are a fast, neurally mediated baroreceptor mechanism and a slower, hormonally regulated renin–angiotensin–aldosterone mechanism

How well did you know this?
1
Not at all
2
3
4
5
Perfectly
23
Q

Textile factory worker,which disease?
bysinosis

asbestosis

silicosis

asthma

pneumoconeosis

A

A) Reason

Organic Dusts include: Cotton, flax, hemp present in Textile manufacturing cuase ByssinosisRed cedar dust present in Lumbering, carpentry cause Asthma.

How well did you know this?
1
Not at all
2
3
4
5
Perfectly
24
Q

markers on B-cell?
CD3

CD 8,9

CD 19,20

CD 10,11

CD 22,28

A

C) Reason

B cells Ig (binds antigen)CD19, CD20, CD21 (receptor for EBV), CD40MHC II, B7

How well did you know this?
1
Not at all
2
3
4
5
Perfectly
25
Q

least mediated by local metabolites?
brain

blood vessels

kidney

skin

heart

A

D) Reason

Skin is richly supplied by the extensive distribution of the sympathetic nerve fibers. Its blood flow is least affected hy local metabolites rather is mediated by sympathetic stimulation or inhibition.

How well did you know this?
1
Not at all
2
3
4
5
Perfectly
26
Q

knee jerk?
L1-2

L2-3

L5-S1

L4-5

L3-4

A

E) Reason

Route Values of the different Deep Tendon reflexes are as: Bicep C5-6, Tricep: C3-4, Supinator C6-7, Knee Jerk L3-4, Ankle S1 and Jaw jerk V5 (CN V).

How well did you know this?
1
Not at all
2
3
4
5
Perfectly
27
Q

HIV patient is sero positive,what first step you would do?
isolate the patient for 1-2 months then test

treat immidiately

divorce spouse

leave home

admit in HIV clinic

A

A) Reason

ELISA/Western blot tests look for antibodies to viral proteins; these tests often are falsely negative in the first 1–2 months of HIV infection till you isolate the patient. It is falsely positive initially in babies born to infected mothers (anti-gp120 crosses placenta).

How well did you know this?
1
Not at all
2
3
4
5
Perfectly
28
Q

at FRC?
chest wall recoil

chest wall expand and lung recoils

lung expand

chest wall expand

lungs recoil

A

B) Reason

At FRC, inward pull of lung is balanced by outward pull of chest wall, and system pressure is atmospheric

How well did you know this?
1
Not at all
2
3
4
5
Perfectly
29
Q

Right sided weakness and left sided eye deviation,lesion is in?
right cortex

left cerebral cortex

corpus callosum

corona radiata

midbrain

A

B) Reason

In our brain, right side of body is controlled by left hemisphere and vice versa. So right side of body weakness is the feature of left sided lesion in brain and vice versa.

How well did you know this?
1
Not at all
2
3
4
5
Perfectly
30
Q

blood transfusion reaction?
O to A

O to B

O to AB

O+ to O-

A to AB

A

E) Reason

When A blood group is donated to a patient with blood group AB, Anti-B Antibodies of A blood group will damage the B antigens of AB blood group laeding to mismatch reaction.

How well did you know this?
1
Not at all
2
3
4
5
Perfectly
31
Q

virulence of staph aureus?
membrane lipids

cell membrane

polysacchride coat

coagulase +ve

cell wall

A

D) Reason

Staph make catalase because they have more “staff.” Bad staph (aureus) make coagulase and toxins. Forms fibrin clot around self leading to abscess.

How well did you know this?
1
Not at all
2
3
4
5
Perfectly
32
Q

minimum amount of urine reqired for draining wastes from body in
100ml

200-300ml

300-400ml

1200ml

500-600ml

A

E) Reason

Minimum amount (volume) of urine output that can prevwnt the adverse level of nitrogeneous wastes is 0.5ml/kg/hr that comes to be about 500-800ml in a person of 50-70kg.

How well did you know this?
1
Not at all
2
3
4
5
Perfectly
33
Q

In a patient with COPD which will inc
ERV

FVC

FEV

FRC

None of these

A

D) Reason

Obstructive lung volumes > normal (inc. TLC, inc. FRC, inc.RV); restrictive lung volumes < normal. In both obstructive and restrictive, FEV1 and FVC are reduced. In obstructive, however, FEV1 is more dramatically reduced compared to FVC, resulting in a decreased FEV1/FVC ratio.

How well did you know this?
1
Not at all
2
3
4
5
Perfectly
34
Q

Pterygoplatine fossa is also called
retropharyngeal

hypopharyngeal

parapharyngeal

deep vertebral

deep cervical

A

C) Reason

The pterygopalatine ganglion is a parasympathetic ganglion, which is suspended from the maxillary nerve in thepterygopalatine fossa (parapharyngeal fossa). It is secretomotor to the lacrimal andthe nasal glands .

How well did you know this?
1
Not at all
2
3
4
5
Perfectly
35
Q

which factor stabilize fibrinogen
factor X

Factor XI

factor XII

factor IX

factor 13

A

E) Reason

Factor X: Stuart factor; Stuart-Prower factorFactor XI: Plasma thromboplastin antecedent (PTA); antihemophilic factor CFactor XII: Hageman factorFactor XIII: Fibrin-stabilizing factorPrekallikrein: Fletcher factorHigh-molecular-weight Fitzgerald factor; HMWK.

How well did you know this?
1
Not at all
2
3
4
5
Perfectly
36
Q

Sudden loss of vision in one eye in an old man, what can be the cause
hypertension

Diabetes meillitis

retinal detachment

SLE

giant cell arteritis

A

E) Reason

Giant cell Arteritis: Clinical manifestations include: (1) Malaise and fatigue (2) Headache or claudication of the jaw (3) Tenderness, absent pulse, and palpable nodules along the course of the involved artery (4) Visual impairment, especially with involvement of the ophthalmic artery (5) Polymyalgia rheumatica, a complex of symptoms.

How well did you know this?
1
Not at all
2
3
4
5
Perfectly
37
Q

Muscle involved in dorsiflexion at ankle joint
tibialis longus

peroneus brevis

peroneus longus

tibialis posterior

tibialis anterior

A

E) Reason

• Dorsiflexion (toes pointing upward): Tibialis anterior, extensor hallucis longus, extensor digitorum longus, and peroneus tertius muscles

How well did you know this?
1
Not at all
2
3
4
5
Perfectly
38
Q

In Hypertensive retinopathy, which vessels will be affected
Capillaries

arterioles

arteries

venules

veins

A

A) Reason

  1. Non-proliferative: Manifestations include micro aneurysms, dilation of veins, hemorrhages, soft (cottonwool) exudates (microinfarcts), and hard exudates (deposits of protein that have leaked from damaged capillaries) , all of which can be viewed with the ophthalmoscope. b. The retinopathy also includes increased capillary permeability, edema, and diffuse thickening of basement membranes (microangiopathy) . c. The disorder is related to the duration of the disease, occurring in most patients with diabetes mellitus after 10 years. 2. Proliferative retinopathy:. New retinal vessel formation (neovascularization) and fibrosis, both extending into the vitreous, is characteristic.
How well did you know this?
1
Not at all
2
3
4
5
Perfectly
39
Q

Free nerve endings are the receptors for
touch

pressure

deep touch

tactile

Pain

A

E) Reason

The receptors for pain are free nerve endings in the skin, muscle, and viscera.■ Neurotransmitters for nociceptors include substance P. Inhibition of the release of substance P is the basis of pain relief by opioids.

How well did you know this?
1
Not at all
2
3
4
5
Perfectly
40
Q

Upper fibers of external oblique interdigitate with
serratus posterior

external oblique

tranversus abdominis

quadratus lumborum

serratus anterior

A

E) Reason

External oblique aponeurosis, reinforced laterally by origin of the internal oblique from the inguinalligament. Posteriorly, its upper fibers interdigitate with serratus anterior

How well did you know this?
1
Not at all
2
3
4
5
Perfectly
41
Q

Major Primary fetal adrenal cortex hormone is
estrogen

LH

Testosterone

DHEA

Aldosterone

A

D) Reason

Estrogens are produced by the interplay of the fetal adrenal gland and the placenta. The fetal adrenal gland synthesizes dehydroepiandrosterone-sulfate (DHEA-S), which is then hydroxylated in the fetal liver.

How well did you know this?
1
Not at all
2
3
4
5
Perfectly
42
Q

Bone marrow is present in
metaphysis

epiphysis

diaphysis

prophysis

compact bone

A

C) Reason

The central most portion of the diaphysis of larger long bones is filled with bone marrow, either hematopoietically active or fatty, without intervening bony trabeculae.

How well did you know this?
1
Not at all
2
3
4
5
Perfectly
43
Q

Cause of Atheroma can be
inc. HDL

dec. chylomicron

increased LDL

dec. LDL
inc. glycogen

A

C) Reason

Inflammation important in pathogenesis. Endothelial cell dysfunction causes macrophage and LDL accumulation leads to foam cell formation- fatty streaks- smooth muscle cell migration (involves PDGF and FGF), proliferation, and extracellular matrix deposition lead to fibrous plaque leads to complex atheromas

How well did you know this?
1
Not at all
2
3
4
5
Perfectly
44
Q

Athelet at rest, what will be changed in comparison to non-athlete
dec. stroke vol.

increased stroke vol.

dec. Cardiac output
inc. cardiac output

no effect in stroke vol.

A

B) Reason

In Athletes, there is physiological polycythemia that causes dec. heart rate. But at rest heart rate decreases more and hence stroke volume increases in order to maintain the cardiac output. (Cardiac Output= stroke volume x heart rate).

How well did you know this?
1
Not at all
2
3
4
5
Perfectly
45
Q

Satiety center is present in which part of hypothalamus
anterior

medial

anterolateral

anteromedial

ventmedial

A

E) Reason

Ventromedial area: It contains Satiety. Destruction (e.g.craniopharyngioma) leada to hyperphagia. Stimulated by leptin.

How well did you know this?
1
Not at all
2
3
4
5
Perfectly
46
Q

Increased time for ventricular filling, cause is
AV nodal delay

SA node

Purkingi fibers

bundle of his

AV fibers

A

A) Reason

SA node “pacemaker” inherent dominance with slow phase of upstroke.AV node—100-msec delay—atrioventricular delay; allows time for ventricular filling

How well did you know this?
1
Not at all
2
3
4
5
Perfectly
47
Q

Warfarin quick reversal by
vit. K

ffp

vit. K & FFP

Protamin

protein C

A

B) Reason

In an acute setting, Warfarin toxicity is treated with FFP and in prolong management, it is treated with vit. K.

How well did you know this?
1
Not at all
2
3
4
5
Perfectly
48
Q

Patient pleural effusion sample is exudative, what is in it
inc. fibroelasts

dec. proteins

increased protein

inc. eosinophils
inc. specific gravity

A

C) Reason

Increased protein content, cloudy. Due to malignancy, pneumonia, collagen vascular disease, trauma (occurs in states of increased vascular permeability). Must be drained in light of risk of infection.

How well did you know this?
1
Not at all
2
3
4
5
Perfectly
49
Q

A graft from same specie is
isograft

xenograft

synergic graft

allograft

autograft

A

D) Reason

Autograft From self.Syngeneic graft From identical twin or clone.Allograft From nonidentical individual of same species.Xenograft From different species.

How well did you know this?
1
Not at all
2
3
4
5
Perfectly
50
Q

A patient with chest pain was given aspirin, its MAO
Inhibit platelet synthesis

Inhibit coagulation factors

inhibit TXA2

inhibit prostacyclin

Inhibits prostagladin synthesis

A

E) Reason

MECHANISM: Irreversibly inhibits cyclooxygenase (both COX-1 and COX-2) enzyme by covalent acetylation. Platelets cannot synthesize new enzyme, so effect lasts until new platelets are produced: inc. bleeding time, dec. TXA2 and prostaglandins. No effect on PT or PTT

How well did you know this?
1
Not at all
2
3
4
5
Perfectly
51
Q

Old age women with hx of IHD with irregular heart beat
sotalol

proparanolol

amoidarone

cardioversion

Digoxin

A

E) Reason

CLINICAL USE: CHF (increases contractility); atrial fibrillation (decreases conduction at AV node and depression of SA node). Patient in this scenario is a case of atrial fibrillation.

How well did you know this?
1
Not at all
2
3
4
5
Perfectly
52
Q

End stage renal disease, what will be the finding
hypokalemia

hypernatremia

dec. Creatinine

increased creatinine

dec. urea

A

D) Reason

Consequences (MAD HUNGER):Metabolic Acidosis, Dyslipidemia (especially increased triglycerides), HyperkalemiaUremia—clinical syndrome marked by inc. BUN and inc. creatinineNausea and anorexia

How well did you know this?
1
Not at all
2
3
4
5
Perfectly
53
Q

Ligamentum teres of liver is derived from
left umbilical vein

right umbilical vein

colon remnant

fossa ovalis

none of these

A

A) Reason

After birth, the umbilical veins are closed, and the remnant in adults is the Ligamentum teres of the liver

How well did you know this?
1
Not at all
2
3
4
5
Perfectly
54
Q

Right homonomous heminopia, lesion in
left occipital lobe

left temoral lobe

left optic tract

right optic radiation

upper optic chiasma

A

A) Reason

Homonemous hemianopia is contralatral half eye vision vision and is due to the lesion of occipital lobe.

How well did you know this?
1
Not at all
2
3
4
5
Perfectly
55
Q

Aortic pressure is maximum during
isovolumic contraction

isovolumic relaxation

end of ventriclular ejection

atrial filling

ventricular filling

A

C) Reason

Systolic ejection—period between aortic valve opening and closing, at end of this phase, the aortic pressure becomes maximum

How well did you know this?
1
Not at all
2
3
4
5
Perfectly
56
Q

On an anomaly scan, anencehaly is suspected, what is serum marker
Alpha fetoprotein

β hcg

sudan red

dec. vimentin
inc. keratin

A

A) Reason

Anencephaly Malformation of anterior neural tube resulting in no forebrain, open calvarium (“frog-like appearance”). Clinical findings: increased AFP; polyhydramnios (no swallowing center in brain). Associated with maternal diabetes (type I). Maternal folate supplementation inc. risk.

How well did you know this?
1
Not at all
2
3
4
5
Perfectly
57
Q

Structure post to medial mellulous
tibialis anterior

tibialis posterior

peroneus longus

sural nerve

tibial artet

A

B) Reason

The tuberosity lies infront of and below the medial malleolus, and it attaches tothe main part of the tibialis posterior tendon (posterior to medial maleolus).

How well did you know this?
1
Not at all
2
3
4
5
Perfectly
58
Q

Hunger centre is located in which part of hypothalamus
inferior

superior

posterior

lateral

anterior

A

D) Reason

Lateral area: Hunger. Destruction causes anorexia, failure to thrive (infants). Inhibited by leptin.

How well did you know this?
1
Not at all
2
3
4
5
Perfectly
59
Q

Submandibular gland stone result in
hypertrophy

hypoplasis

dysplasia

Apoptosis

necrosis

A

D) Reason

Obstruction to secretion of any body fluid is trigger Factor for inflammation, in glandular organs, obstruction to secretions can lead to self digestion or Autophagy of the cell- Apoptosis

How well did you know this?
1
Not at all
2
3
4
5
Perfectly
60
Q

Retrorbital swelling is due to
lymphocytes

fibroelastic tissue

fat deposits

thyroid stimulating Igs

SM Igs

A

D) Reason

Striking exophthalmos (protrusion of the eyes), possibly due to autoimmune (Thyroid Stimulating Igs) mechanisms and independent of thyroid hyperfunction, is characteristic. Manifestations include the signs and symptoms of hyperthyroidism.

How well did you know this?
1
Not at all
2
3
4
5
Perfectly
61
Q

blood will accumulate in epidural space due to trauma to which artry
MMA

AMA

cerebral artery

spinal artery

posterior cerebral artery

A

A) Reason

Lucid interval after traumatic brain injury are the features of Epidural hematoma (middle meningeal artery rupture)

How well did you know this?
1
Not at all
2
3
4
5
Perfectly
62
Q

lupus nephritis is associated with
nephrotic syndrome

nephritic syndrome

drug induced

allergic

SLE

A

E) Reason

Classic presentation of SLE: rash, joint pain, and fever, most commonly in a female of reproductive age and African descent.Libman-Sacks endocarditis—wart-like vegetations on both sides of valve.Lupus nephritis (type III hypersensitivity reaction

How well did you know this?
1
Not at all
2
3
4
5
Perfectly
63
Q

cervical vertebra is closely associated with
cilliary ganglion

ophthalmic ganlion

sympathetic ganglion

stellate ganglion

salivary ganglion

A

D) Reason

In most people, the inferior cervical ganglion is fused to thefirst thoracic ganglion to form the stellate ganglion. It is located between the transverse process of the seventh cervical vertebra and the neck of the first rib.

How well did you know this?
1
Not at all
2
3
4
5
Perfectly
64
Q

Notochord derivative is
spinal cord

spinal nerves

nucleus pulposus

CSF

miningies

A

C) Reason

Nucleus pulposus is an inner soft, elastic, compressible material thatfunctions as a shock absorber for external forces placed on the vertebral column. The nucleus pulposus is the postnatal remnant of the notochord.

How well did you know this?
1
Not at all
2
3
4
5
Perfectly
65
Q

Calcitonin released from
thyroid gland

parathyroid

adrenal cortex

pituitary

hypothalamus

A

A) Reason

SOURCE: Parafollicular cells (C cells) of thyroid. FUNCTION: bone resorption of Ca2+. It opposes the effect of PTH

How well did you know this?
1
Not at all
2
3
4
5
Perfectly
66
Q

There is infection in between deep layers of cervical fascia and pre tracheal fascia , pus will go to
posterior to pericardium

in pericardium

anterior to pericardium

anterior mediastinum

posterior mediastinum

A

D) Reason

The pretracheal layer is a thin layer that is attached above tothe laryngeal cartilages . Infection between deep cervical fascia and pre tracheal fascia can affect the anterior mediastinum.

How well did you know this?
1
Not at all
2
3
4
5
Perfectly
67
Q

IgE bound to which of following
Basophils & mast cells

Basophils only

mast cell only

eosinophils

lymphocytes

A

A) Reason

Anaphylactic and atopic—free antigen cross links IgE on presensitized mast cells and basophils, triggering immediate release of vasoactive amines that act at postcapillary venules (i.e., histamine). Reaction develops rapidly after antigen exposure because of preformed antibody

How well did you know this?
1
Not at all
2
3
4
5
Perfectly
68
Q

Patient has pneumonia , now he has chest pain radiating to back and rustle sound on left side of chest even when breathing is held , which is affected
pleura

pericardium

anterior mediastinum

lateral mediastinum

posterior mediastinum

A

B) Reason

Acute pericarditis: Commonly presents with sharp pain aggravated by inspiration, and relieved by sitting up and leaning forward. Presents with friction rub. ECG changes include widespread ST-segment elevation and/or PR depression. Suppurative/purulent Type: usually caused by bacterial infections (e.g Pneumococcus, Streptococcus). Rare now with antibiotics.

How well did you know this?
1
Not at all
2
3
4
5
Perfectly
69
Q

Bronchopulmonary segment is
functional unit

structural unit

respiratory unit

lymphatic unit

broncheal unit

A

A) Reason

Bronchopulmonary segments are the anatomic, functional, and surgical units of the lungs. Each lobar (secondary) bronchus, which passes to a lobe of the lung, gives offbranches called segmental (tertiary) bronchi

How well did you know this?
1
Not at all
2
3
4
5
Perfectly
70
Q

Host versus graft reaction can be due to what
kidney transplant

blood transfusion

liver transplant

FFP transfusion

lung transplant

A

B) Reason

In rest of all conditions (except FFP) GVHD occurs Graft-versus-host Reaction/disease:Onset Varies. Grafted immunocompetent T cells proliferate in the immunocompromised host and reject host cells with “foreign” proteins leading to severe organ dysfunction.

How well did you know this?
1
Not at all
2
3
4
5
Perfectly
71
Q

Emphysema can be due to deficiency of
lipase

phospholipase A2

α1 anti-trypsin

ALP

fibroelastic growth factor

A

C) Reason

Enlargement of air spaces, decreased recoil,increased compliance, decreased DLCO resulting from destruction of alveolar walls.Two types:1. Centriacinar—associated with smoking B.2. Panacinar—associated with α1-antitrypsin deficiency

How well did you know this?
1
Not at all
2
3
4
5
Perfectly
72
Q

Source of creatinine is by breakdown of
heart

muscle

kidneys

blood

liver

A

B) Reason

Creatinine is the end product of the muscle metabolism that is the reason that Creatinine increases in myositis and other cases of muscle breakdown

How well did you know this?
1
Not at all
2
3
4
5
Perfectly
73
Q

Glucokinese is present in
muscle

Liver

heart

lungs

pancreas

A

B) Reason

Glucokinese is an enzyme that phosphorylates glucose to glucose-6-P, it present in liver β cells unlike that of hexokinase that is present in muscles but not in liver or pancreas

How well did you know this?
1
Not at all
2
3
4
5
Perfectly
74
Q

Leimyoma with glands and stroma covering
Metaplasia

dysplasia

hypertrophy

dystrophy

hyperplasia

A

E) Reason

Leiomyoma (fibroid):A Most common tumor in females. Often presents with multiple discrete tumors A. Increased incidence in blacks. Benign smooth muscle tumor; malignant transformation is rare. Estrogen sensitive—tumor size increased with pregnancy and decreased with menopause.

How well did you know this?
1
Not at all
2
3
4
5
Perfectly
75
Q

which of the following doesn’t have basement membrane
clara cell

Ependymal cells

peumocytes

osteoclasts

transitional epithelium

A

E) Reason

Transitional epithelium is a stratified epithelium in which the shape of the surface cells changes (undergoes transitions) depending on the degree of stretch. This epithelium has no basement membrane.When a transitional epithelium is not stretched (for example in an empty bladder) the cells of the surface layers are large and rounded.

How well did you know this?
1
Not at all
2
3
4
5
Perfectly
76
Q

A patient is diagnosed to have Hepatitis A, labs could be
inc. ALP

inc. bilurubin

inc bilirubin and ALT

dec. ALT

decreased heptoglobins

A

C) Reason

Acute viral hepatitis is characterized by jaundice and extremely high elevations of serum aspartate and alanine aminotransferases. Hyperbilirubinemia is also there

How well did you know this?
1
Not at all
2
3
4
5
Perfectly
77
Q

In an RTA Pelvis avulsion, muscle involved
rectus femoris

quadricep femoris

adductor longus

adductor brevis

sartorius

A

A) Reason

• Among the muscles of the anterior compartment, Iliopsoas, rectus femoris, sartorius, and adductormuscles, most prominent and superficial muscle is rectus femoris that is injured first in avulsion injury.

How well did you know this?
1
Not at all
2
3
4
5
Perfectly
78
Q

1/3rd of posterior Interventricular septum blood supply of heart is
LAD

RCA

LCA

LXA

posterior artery

A

B) Reason

Posterior interventricular artery: It is the terminal distribution of the right coronary artery and courses in the posterior interventricular sulcus to supply parts of the right and left ventricles and, importantly, the posterior third of the interventricular septum.

How well did you know this?
1
Not at all
2
3
4
5
Perfectly
79
Q

In Myasthenia gravis, what is responsible for pathogenesis
IgA

IgD

IgG

IgM

IgE

A

C) Reason

M. gravis is the most common NMJ disorder, PATHOPHYSIOLOGY: Autoantibodies (IgGs) to postsynaptic ACh receptors. CLINICAL Manifestations are Ptosis, diplopia, weakness.Worsens with muscle use

How well did you know this?
1
Not at all
2
3
4
5
Perfectly
80
Q

A female presented with some growth in vulvar region, it can be
atrophy

dysplasia

hyperplasia

dystrophy

hypertrophy

A

B) Reason

Dysplasia and carcinoma in situ:Disordered epithelial growth; begins at basal layer of squamocolumnar junction (transition zone) and extends outward

How well did you know this?
1
Not at all
2
3
4
5
Perfectly
81
Q

lesion in left somatosensory area will affect
agraphesthesia

asteriagnosis

right lower limb

left lower limb

paraplegia

A

C) Reason

Lesions in somatosensory cortex result in impairment of all somatic sensations on the opposite side of the body, including the face and scalp.

How well did you know this?
1
Not at all
2
3
4
5
Perfectly
82
Q

Congenital Toxoplamosis can be diagnosed by
IgA

IgD

IgE

IgM

IgG

A

D) Reason

For the diagnosis of acute and congenital infections, an immunofluorescence assayfor IgM antibody is used. IgM is used to diagnose congenital infection, because IgG can be maternal in origin.

How well did you know this?
1
Not at all
2
3
4
5
Perfectly
83
Q

Clara cells are present in
primary bronchiole

secondary bronchiole

tertiary bronchiole

terminal bronchioles

alveolar sacs

A

D) Reason

The epithelial lining of the airway is composed of ciliated cells (goblet and basal cells are absent in the terminal bronchioles) and an additional type called the Clara cell.

How well did you know this?
1
Not at all
2
3
4
5
Perfectly
84
Q

A patient presents with features of dysphagia and horner syndrome ,artrey involved is
AICA

MCA

PICA

ACA

vertebral artery

A

A) Reason

Features of obstruction of anterior communicatin Artery: Vomiting, vertigo, nystagmus; decreased pain and temperature sensation from ipsilateral face and contralateral body; dysphagia,hoarseness, decreased gag reflexipsilateral Horner syndrome; ataxia, dysmetria.

How well did you know this?
1
Not at all
2
3
4
5
Perfectly
85
Q

Spinal cord artery supply is by
vertebral artery

spinal artery

posterior cerebral artery

posterior cervical artey

occipital artery

A

A) Reason

There is only one anterior spinal artery formed bythe union of branches from each vertebral artery. Vertebral artery is the main supply of the spinal cord.

How well did you know this?
1
Not at all
2
3
4
5
Perfectly
86
Q

Anterior surface of heart is supplied by
posterior Interventricular artery

RCA

LCA

LXA

LAD

A

E) Reason

Left anterior descending artery (LAD)—supplies anterior 2/3 of interventricular septum, anterior papillarymuscle and anterior surface of left ventricle. LAD is the most common cardiac artery to be occluded.

How well did you know this?
1
Not at all
2
3
4
5
Perfectly
87
Q

ECG findings of hypoglycemia
ST depression

No P wave

short QT

tall T wave

prolong QT interval

A

E) Reason

Long QT interval:1. Bradycardia2. Hypocalcemia3. Acute MI4. Acute myocarditis5. Cerebrovascular accident6. Hypertrophic cardiomyopathy7. Hypothermia

How well did you know this?
1
Not at all
2
3
4
5
Perfectly
88
Q

Mediastinal pleura is supplied by
C1-2

Phrenic nerve

C4

pleural nerve

vagus nerve

A

B) Reason

The phrenic nerve is the only motor nerve supply to thediaphragm. It also sends sensory branches to the pericardium, the mediastinal parietal pleura, and the pleura and peritoneum covering the upper m and lower surfaces of thecentral part of the diaphragm.

How well did you know this?
1
Not at all
2
3
4
5
Perfectly
89
Q

Lady with pregnancy malnurished now present with anemia, what is cause..?
sideroblastic anemia

Megaloblastic anemia

Thalasemia

pernicous anemia

iron deficiency

A

E) Reason

Causes are: Decreased iron due to chronic bleeding (GI loss, menorrhagia), malnutrition/absorption disorders or increased demand (e.g., pregnancy)decreased final step in heme synthesis.

How well did you know this?
1
Not at all
2
3
4
5
Perfectly
90
Q

Progressive PR prolongation followed by missing a beat on ECG is diagnostic of
atrial flutter

2nd degree block

mobitz type2

mobitz type 1

3rd degree block

A

D) Reason

Mobitz type I (Wenckebach):Progressive lengthening of the PR interval until a beat is “dropped” (a P wave not followed by a QRS complex). Usually asymptomatic.

How well did you know this?
1
Not at all
2
3
4
5
Perfectly
91
Q

sulpha drug Mechanism of action
activate B6

activate B3

block DNA gyrase

block RNA synthesis

Inhibit dihydropteroate synthetase

A

E) Reason

Sulfonamides interfere with folic acid synthesis by preventing addition of para-aminobenzoic acid (PABA) into the folic acid molecule through competing for the enzyme dihydropteroate synthetase. Sulfonamides are therefore bacteriostatic and inhibit growth and multiplication of bacteria, but do not kill them. Humans, in contrast to bacteria, acquire folate (vitamin B9) through the diet.

How well did you know this?
1
Not at all
2
3
4
5
Perfectly
92
Q

Patient doctor relationship is based on
informed working

mutual benefit

mutual respect

mutual trust

long relation

A

D) Reason

Doctor and Patient relationship is very special and based on absolute privacy and mutual trust

How well did you know this?
1
Not at all
2
3
4
5
Perfectly
93
Q

Retinal art occlusion in previously healthy man, can be due to
rupture

Amylodosis

emboli

aneurysm rupture

silent hypertension

A

C) Reason

Sudden loss of vision commonly occurs in central retinal artery occlusion and central retinalvein occlusion, retinal detachment, papillitis, acute congestive glaucoma, emboli, vitreous haemorrhage,etc.

How well did you know this?
1
Not at all
2
3
4
5
Perfectly
94
Q

Skin malignancy with most malignant potentail
melanoma

SCC

BCC

lymphoma

juvenile nevus

A

A) Reason

MELANOMA:A. Malignant neoplasm of melanocytes; most common cause of death from skin cancer B. Risk factors are based on UVB-induced DNA damage and include prolonged exposure to sunlight, albinism, and xeroderma pigmentosum

How well did you know this?
1
Not at all
2
3
4
5
Perfectly
95
Q

Scapula is firmly attached to chest wall through
teres major

serratus posterior

external oblique

serratus anterior

teres minor

A

D) Reason

Serratus anterior Insertion is to the Medial border and inferiorangle of the scapula where it stabilizes the bone

How well did you know this?
1
Not at all
2
3
4
5
Perfectly
96
Q

Tunica albogenia gives rise to which of following
processus vaginalis

tunica vaginalis

umbilical cord

inguinal canal

inguinal ligament

A

A) Reason

Processus vaginalis(evagination of peritoneum):Forms tunica vaginalis as its remnant in male

How well did you know this?
1
Not at all
2
3
4
5
Perfectly
97
Q

Pt is on hormone replacement therapy for 10 yr , sister died due to breast ca, she has high risk of
CA endometrium

Ovarian cancer

Fallopian tube CA

Lung cancer

CA breast

A

E) Reason

Risk factors: inc. estrogen exposure, inc. total number of menstrual cycles, older age at 1st live birth, obesity (inc. estrogen exposure as adipose tissue converts androstenedione to estrone), BRCA1 and BRCA2 gene mutations, African American ethnicity (inc. risk for triple inc. breast cancer).

How well did you know this?
1
Not at all
2
3
4
5
Perfectly
98
Q

Kidney in relationship with lower ribs by
facia covers glands

garota facia

psoas muscle

quadratus lumborum

perinehric fat

A

A) Reason

Renal fascia: This is a condensation of areolar tissue outside the perirenal fat. It encloses the kidneys and thesuprarenal glands

How well did you know this?
1
Not at all
2
3
4
5
Perfectly
99
Q

Face sensation is carried by which nerve
facial

mandibular

ophthalmic

maxillary

Trigeminal

A

D) Reason

he maxillary nerve is purely sensory. It leaves the skull through the foramen rotundum and crosses thepterygopalatine fossa to enter the orbit through the inferiororbital fissure. It then continues as the infraorbital nerve in the infraorbital groove, and it emerges on the facethrough the infraorbital foramen. It gives sensory fibers tothe skin of the face and the side of the nose.

How well did you know this?
1
Not at all
2
3
4
5
Perfectly
100
Q

In asthma, which is involved
small bronchioles

terminal bronchioles

alveolar ducts

main bronchus

medial bronchioles

A

E) Reason

Asthma: Bronchial hyperresponsiveness causes reversible bronchoconstriction. Smooth muscle hypertrophy, Curschmann spirals (shed epithelium forms mucus plugs), and Charcot Leyden crystals (formed from breakdown of eosinophils in sputum).

How well did you know this?
1
Not at all
2
3
4
5
Perfectly
101
Q

Most common involved in carcinoma
vimentin

keratin

cadherin

fibrin

kertinocytes

A

B) Reason

A first step in the invasion of malignant cells through an epithelium results from a loss of expression of cadherins and loss of keratin that weakens the epithelium.

How well did you know this?
1
Not at all
2
3
4
5
Perfectly
102
Q

lesion at L1 , there was loss of pain and temp on right side, which path is affected
spinobulbar

medial leminiscus

lateral spinothalamic

anterior horn

lateral horn

A

C) Spinothalamic and dorsal columns/medial lemniscus:Take Pain and temperature; pressure, touch, vibration, and proprioceptionDestination to 1° somatose nsory cortex

How well did you know this?
1
Not at all
2
3
4
5
Perfectly
103
Q

Max calcium influx at cardiac cycle, in
R wave

ST segment

P wave

Q wave

T wave

A

B) Reason

Phase 2 = plateau (corresponds to ST Segment): Ca2+ influx through voltage-gated Ca2+ channels balances K+ efflux. Ca2+influx triggers Ca2+ release from sarcoplasmic reticulum and myocyte contraction.

How well did you know this?
1
Not at all
2
3
4
5
Perfectly
104
Q

Medial rotation of arm is by
teres minor

deltoid

triceps

serratus posterior

teres major

A

E) Reason

• Lateral rotation:Infraspinatus, teres minor, and posteriorfibers of deltoid muscles.• Medial rotation: Subscapularis, latissimus dorsi, teresmajor, and anterior fibers of deltoid muscles

How well did you know this?
1
Not at all
2
3
4
5
Perfectly
105
Q

What lies in the floor of the Lateral ventricle
thalamus

hypothalamus

uncus

amydala

hippocampus

A

E) Reason

The hippocampal formation extends along the floor of the inferior horn of the lateral ventricle in the temporal lobe and includes the hippocampus, the dentate gyrus, the subiculum, and adjacent entorhinal cortex.

How well did you know this?
1
Not at all
2
3
4
5
Perfectly
106
Q

Muscles of forceful expiration
diaphragm

rectus abdominis

scalenae anterior

scalenae posterior

intercostal muscle

A

B) Reason

Function: Compresses abdominal contents and flexes vertebral column, is also an accessory muscle of expiration especially forceful expiration

How well did you know this?
1
Not at all
2
3
4
5
Perfectly
107
Q

In aspirin overdose, which drug will increase its renal excretion
Cl

K

HCO3

Na

Ca

A

C) Reason

Overdose:causes respiratory alkalosis initially, which is then superimposed by metabolic acidosis (by increased urinary excretion).

How well did you know this?
1
Not at all
2
3
4
5
Perfectly
108
Q

ALT and AST require which cofactor
B1

B2

B6

B12

vit.D

A

C) Reason

FUNCTION: Converted to pyridoxal phosphate, a cofactor used in transamination (e.g., ALT and AST), decarboxylation reactions, glycogen phosphorylase.

How well did you know this?
1
Not at all
2
3
4
5
Perfectly
109
Q

How to prevent tetanus in population
toxoid

antitoxin

vaccination

clean instruments

proper care

A

A) Reason

Tetanus can be prevented by Toxoid.

How well did you know this?
1
Not at all
2
3
4
5
Perfectly
110
Q

type of hypersensitivity in post streptococcal GN
delayed response

type 1 HSR

type 2 HSR

type 4 HSR

type 3 HSR

A

E) Reason

Type III: SLEPolyarteritis nodosaPoststreptococcal glomerulonephritisSerum sicknessArthus reaction (e.g., swelling and inflammation following tetanus vaccine).

How well did you know this?
1
Not at all
2
3
4
5
Perfectly
111
Q

Pain from the mininges of middle cranial fossa
Trigeminal ganglion

Cilliary ganglion

ophthalmic ganglion

cerebellum

basal ganglia

A

A) Reason

The trigeminal ganglion lies in middle cranial fossa within a pouchof dura mater called the trigeminal cave. Middle cranial fossa pain is due to pressure of this ganglion on minininges.

How well did you know this?
1
Not at all
2
3
4
5
Perfectly
112
Q

common carotid artery has lateral relation with
recurrent laryngeal nerve

parathyroid

external carotid

internal jugular vein

common carotid

A

D) Reason

In the carotid sheath, the artery is related laterally to the internal jugular vein; the vagus nerve lies between these two structures.

How well did you know this?
1
Not at all
2
3
4
5
Perfectly
113
Q

A young Patient with Dx of Aplastic anemia was given Anti-thymocyte Igs, developed reaction at 10 days..cause..?
type 1 HSR

type 2 HSR

type 3 HSR

type 4 HSR

delayed response

A

C) Reason

Type III: SLEPolyarteritis nodosaPoststreptococcal glomerulonephritis, Serum sicknessArthus reaction (e.g., swelling and inflammation following tetanus vaccine)

How well did you know this?
1
Not at all
2
3
4
5
Perfectly
114
Q

A 7 year old boy has unilateral blindness, gene affected
RET

BRCA1

Rb

p53

neu1

A

C) Reason

The prototype is retinoblastoma, an intraocular childhood tumor (causing unilateral blindness) caused by inactivation of the Rb gene. The “two-hit” hypothesis of Knudson holds that two mutagenic events are required to induce alterations on both chromosomes.

How well did you know this?
1
Not at all
2
3
4
5
Perfectly
115
Q

splitting of s2 is because of
early mitral closure

early pulmonary close

early aortic closure

late aortic closure

late mitral closure

A

C) Reason

Normal splitting is due to delayed closure of pulmonic valve than aortic valve.decreased pulmonary impedance ( increased capacity of the pulmonary circulation) also occurs during inspiration, which contributes to delayed closure of pulmonic valve.

How well did you know this?
1
Not at all
2
3
4
5
Perfectly
116
Q

what is true regarding Crohn”s disease
transmural lesions

continuous lesion

only colon involved

only mucosa involved

female predominant

A

A) Reason

GROSS MORPHOLOGY: Transmural inflammation, fistulas. Cobblestone mucosa, creeping fat, bowel wall thickening (“string sign” on barium swallow x-ray), linear ulcers, fissures.

How well did you know this?
1
Not at all
2
3
4
5
Perfectly
117
Q

A middle age patient with chest pain and raised CK but normal troponin and ECG, cause is
MI

angina

musculosketal pain

stable angina

gastric pain

A

C) Reason

In this scenario, the most specific marker that is cardioselective (troponin and ECG ) are normal while CK can be raiswd in muscular pain.

How well did you know this?
1
Not at all
2
3
4
5
Perfectly
118
Q

If sodium absorption is increased at DCT it will result in?
inc. plasma vol.

dec. plasma vol.
inc. interstitial cells
inc. urine vol.
dec. urine vol.

A

A) Reason

When the Na is reabsorbed in DCTs, it causes fluid retention in the body and thus leading to increased plasma volume.

How well did you know this?
1
Not at all
2
3
4
5
Perfectly
119
Q

Connexins are part of
RER

SER

ribosomes

Golgi complex

nucleus

A

D) Reason

Each connexon consists of 6 connexin molecules. Unlike other intercellular junctions, gap junctions are not associated with any cytoskeletal filament and Golgi complex.

How well did you know this?
1
Not at all
2
3
4
5
Perfectly
120
Q

Regarding Hemostasis, Most common risk factor for thrombosis is
OCPs

endothelial injury

trauma

aspirin

Clopidogrel

A

A) Reason

Most common is OCP . Most initial is endothelial injury.

How well did you know this?
1
Not at all
2
3
4
5
Perfectly
121
Q

which sign differentiate ganglionic or muscrinic blockers
selectivity

postural hypotension

dosing

quantity

specificity

A

B) Reason

Unlike the muscrinic blockers, the ganglion blockers interrupt sympathetic control of venous tone, they cause marked venouspooling; postural hypotension is a major manifestation of this effect.

How well did you know this?
1
Not at all
2
3
4
5
Perfectly
122
Q

side effect of documerol
delayed coagulation

early coagulation

acidosis

dec. RBC
dec. WBC

A

A) Reason

TOXICITY: Nausea, headache, facial flushing, coagulopathy, hypotension, abdominal pain.

How well did you know this?
1
Not at all
2
3
4
5
Perfectly
123
Q

Regarding the vectors, which virus is transmitted by mosquitoes
HHV

RSV

gray fever

yellow fever

hey fever

A

D) Reason

Yellow fever virus: A flavivirus (also an arbovirus) transmitted by Aedes mosquitoes. Virus has a monkey or human reservoir

How well did you know this?
1
Not at all
2
3
4
5
Perfectly
124
Q

In patient with previous case of Vasculitis, hepatitis associated with Vasculitis is
EBV

hep. D
hep. A
hep. C

Hepatitis B

A

E) Reason

Polyarteritis nodosa: Young adults.Hepatitis B seropositivity in 30% of patients.Fever, weight loss, malaise, headache.

How well did you know this?
1
Not at all
2
3
4
5
Perfectly
125
Q

CSF has what difference as compared to plasma
low pH

high pH

low Na

low Ca

low Cl

A

A) Reason

A total of 400-500 cc of CSF is produced per day; ventricles and subarachnoid space contain 90-150 cc, so all of CSF is turned over 2-3 times per day. In comparison with plasma, it has less glucose, more proteins, less cells and low pH.

How well did you know this?
1
Not at all
2
3
4
5
Perfectly
126
Q

nuclei of cells producing proteins
eosinophilic

dark staining

basic stippling

light staining

purple staining

A

B) Reason

Hematoxylin is a blue dye which stains basophilic substrates that are the acidic cellular components such as DNA and RNA. Hematoxylin stains nuclei blue, and may tint the cytoplasm of cells with extensive mRNA (proteins synthesis) in their cytoplasm.

How well did you know this?
1
Not at all
2
3
4
5
Perfectly
127
Q

hypovolumic shock & anaphylaxis differ in ?
BP

cardiac output

heart rate

aetiology

manifestations

A

B) Reason

Difference between the Anaphylactic shock and hypovolumic shock is the cardiac output, it is decreased in hypovolumic shock and increased in anaphylactic shock.

How well did you know this?
1
Not at all
2
3
4
5
Perfectly
128
Q

Acrosome is formed from
RER

SER

nucleus

ribosomes

golgi complex

A

E) Reason

The acrosome, which is located over the anterior half of the nucleus, is derived from the Golgi complex of the spermatid and contains several hydrolytic enzymes.

How well did you know this?
1
Not at all
2
3
4
5
Perfectly
129
Q

Patient with Ptosis, dilated pupils and downward outward eye, nerve affected
CN I

CN-II

CN III

CN IV

CNVVII

A

C) Reason

The oculomotor nerve, therefore, is entirely motor. It isresponsible for lifting the upper eyelid; turning the eye upward, downward, and medially; constricting the pupil; andaccommodating the eye.

How well did you know this?
1
Not at all
2
3
4
5
Perfectly
130
Q

In a 49 year old man with chronic Hx of HCV positive, inflamation of portal triads and adjacent lobules with necrotic foci, diagnosis
cirrhosis

hep. A
hep. E

chronic hep. C

chronic hep. D

A

D) Reason

Chronic hepatitis is characterized by symptoms that last > 6 months. l. Inflammation predominantly involves portal tract and adjacent lobules and necrotic foci 2. Risk of progression to cirrhosis

How well did you know this?
1
Not at all
2
3
4
5
Perfectly
131
Q

gunshot injury to neck, patient is in respiratory distress, what affected
C1

C2

S1-2

L3-4

diaphragm

A

E) Reason

Respiratory distress is due to diaphragm paralysis caused by Phrenic nerve injury at cervical C3-C4 spinal segments.

How well did you know this?
1
Not at all
2
3
4
5
Perfectly
132
Q

Hemisection of spinal cord causes complete sensation loss at
at level of lesion

bellow lesion

above lesion

ipsilatrally

contralatrally

A

A) Reason

Hemisection: Ipsilateral loss of all sensation at the level of the lesion Ipsilateral LMN signs (e.g., flaccid paralysis) at the level of the lesionIf lesion occurs above T1, patient may present with Horner syndrome due to damage of oculosympathetic pathway

How well did you know this?
1
Not at all
2
3
4
5
Perfectly
133
Q

receptors responsible for total periphral resistance
alpha 1

alpha 2

beta 2

beta 1

muscrinic

A

A) Reason

Alpha 1 receptors are Present at blood vessels and they maintain the total periphral resistance due the state of vasoconstriction or Vasodilation.

How well did you know this?
1
Not at all
2
3
4
5
Perfectly
134
Q

majority of growth factors have their receptors on
cell wall

SER

RER

membrane

nucleus

A

D) Reason

Growth factor Receptors: ■ Hormone binds to the extracellular side of the receptor. As many of the growth factor receptors are located in cell membrane.

How well did you know this?
1
Not at all
2
3
4
5
Perfectly
135
Q

In Barrett esophagus, pathogenesis is
hypertrophy

hyperplasia

dysplasia

columnar Metaplasia

anaplasia

A

D) Reason

Barrett esophagus: Glandular metaplasia—replacement of nonkeratinized (stratified) squamous epithelium with intestinal epithelium (nonciliated columnar with goblets cells) in the distal esophagus

How well did you know this?
1
Not at all
2
3
4
5
Perfectly
136
Q

which of following is associated with decreased risk of cancer
banana

apple

guava

orange

strawberry

A

D) Reason

It is a normal notation the antioxidants are regarded as protective against the carcinogenesis. Orange contains vit. C that is a strong antioxidant.

How well did you know this?
1
Not at all
2
3
4
5
Perfectly
137
Q

Dicyclomine acts on which receptors
sympathetic

nicotinic

muscrinic

alpha 1

alpha 2

A

C) Reason

The anticholinergicdrugs dicyclomine (antimuscrinic) and hyoscyamine are used as antispasmodics to relieve abdominal pain; however,their efficacy has not been convincingly demonstrated

How well did you know this?
1
Not at all
2
3
4
5
Perfectly
138
Q

during exercise resp. rate increases because of?
tactile reflex

inc. K
inc. O2
inc. K
inc. CO2

A

E) Reason

■ Sensory information (Pco2, lung stretch, irritants, muscle spindles, tendons, and joints) is coordinated in the brain stem.■ The output of the brain stem controls the respiratory muscles and the breathing cycle.

How well did you know this?
1
Not at all
2
3
4
5
Perfectly
139
Q

which will prevent wound contraction
vit. C

vit. D

GH

glucagon

corticosteroids

A

E) Reason

Functions of Corticosteroids: Increases Insulin resistance (diabetogenic)Increases Gluconeogenesis, lipolysis, and proteolysisDecreases Fibroblast activity (causes striae)Decreases Inflammatory and Immune responses:

How well did you know this?
1
Not at all
2
3
4
5
Perfectly
140
Q

fracture of neck of fibula will cause injury to which artery?
fibular

peroneal

posterior tibial

anterior tibial

dorsalis pedis

A

D) Reason

Neurovascular Structures related to the head of the fibula are anterior tibial artry and common peroneal nerve that can be damaged during the fractures

How well did you know this?
1
Not at all
2
3
4
5
Perfectly
141
Q

Patient with CABG 10 days back is with C/O purpra, sec. platelets, diagnosis
ITP

drug induced TTP

DIC

HUS

Thrombocytopenia

A

B) Reason

Thrombotic thrombocytopenic purpura (TIP) (1) Characteristics include platelet-derived hyaline microaggregates in small vessels, thrombocytopenia, and microangiopathic hemolytic anemia.

How well did you know this?
1
Not at all
2
3
4
5
Perfectly
142
Q

If superior mesenteric artery is damaged, which structure is likely to be spared?
spleen

liver

pancreas

dudenum

descending colon

A

E) Reason

Forgut Derivatives Esophagus, Stomach, Duodenum (first and second parts), Liver, Pancreas, Biliary apparatus, Gallbladder are supplied by the Superior mesenteric artery.

How well did you know this?
1
Not at all
2
3
4
5
Perfectly
143
Q

An 8 months baby was born to HIV positive mother, how to confirm infection
WBCs

CBC

ELISA

PCR RNA

western blots

A

D) Reason

HIV PCR/viral load tests determine the amount of viral RNA in the plasma. High viral load associated with poor prognosis. Also use viral load to monitor effect of drug therapy. It also determines the viral load in new born uptill age of 18 months

How well did you know this?
1
Not at all
2
3
4
5
Perfectly
144
Q

Which is bounded by inner and outer membrane and continued with nuclear membrane
cristae

cisternae

F1 particles

fibrils

Perinuclear cisteria

A

E) Reason

RER is a single, lipid bilayer continuous with the outer nuclear membrane. It is organized into stacks of large flattened sacs called cisternae, nuclear cisternae to the nuclear membrane.

How well did you know this?
1
Not at all
2
3
4
5
Perfectly
145
Q

drug for angina that cause no vasodilation
Captopril

hydralazine

Methyldopa

Hydraline

metoprolol

A

E) Reason

Metoprolol is a cardiac selective beta blockers that acts centrally on heart and least on blood vessels and hence has no Vasodilation property.

How well did you know this?
1
Not at all
2
3
4
5
Perfectly
146
Q

2nd vertebra has characteristic feature of?
atlas

spinous process

odontoid process

lateral body

spine

A

B) Reason

The axis (C2) is the point of articulation where the head turns on the neck, providing an “axis of rotation” through the odontoid.

How well did you know this?
1
Not at all
2
3
4
5
Perfectly
147
Q

A young boy with long term history of dirrohea, biopsy show villous atrophy, no improvement with gluten free diet, diagnosis is
giardiasis

celiec disease

amebiasis

intestinal TB

whipples sprue

A

A) Reason

Giardia lamblia:Giardiasis: bloating, flatulence, foul-smelling, fatty diarrhea (often seen in campers/hikers)—think fat-rich Ghirardellichocolates for fatty stools of Giardia. Villous atrophy on microscopy. Treated with metronidazole.

How well did you know this?
1
Not at all
2
3
4
5
Perfectly
148
Q

As an SHO, which sample would you send for blood gases analysis
heart

coagulated

serum sample

Arterial

veinous

A

D) Reason

Arterial blood gases are examined in the fresh sample of an arterial blood with a heparanized syringe.

How well did you know this?
1
Not at all
2
3
4
5
Perfectly
149
Q

Patient presents with long term memory loss, lesion is at
amygdala

cerebellum

basal ganglia

limbic system

uncus

A

D) Reason

Limbic system Collection of neural structures involved in emotion, long-term memory, olfaction, behavior modulation, and autonomic nervous system function.

How well did you know this?
1
Not at all
2
3
4
5
Perfectly
150
Q

What prevent complications in procedures
proper care

regard

profeciency

Professional competence

informed consent

A

D) Reason

In an any medical, the chances of complications are reduced by better management as well as high level of professional competence.

How well did you know this?
1
Not at all
2
3
4
5
Perfectly
151
Q

Among the following, most common method to detect tumor in blood
RFTs

LFTs

CBC

grading

tumor marker

A

E) Reason

Tumor markers Tumor markers should not be used as the 1° tool for cancer diagnosis. They may be used to monitor tumor recurrence and response to therapy, but definitive diagnosis can be made only via biopsy

How well did you know this?
1
Not at all
2
3
4
5
Perfectly
152
Q

hormone which brings about protein anabolism and fat catabolism
LH

FSH

Cortisol

glucagon

GH

A

E) Reason

Direct actions of growth hormone(a) ↓ glucose uptake into cells (diabetogenic)(b) ↑ lipolysis(c) ↑ protein synthesis in muscle and ↑ lean body mass(d) ↑ production of IGF.

How well did you know this?
1
Not at all
2
3
4
5
Perfectly
153
Q

5g protein is lost in pt. what is probable cause of edema.
inc. capillary pressure

dec. capillary pressure

increased vascular pressure

dec. vascular pressure

colloid osmotic pressure

A

E) Reason

Decreased Colloid osmotic pressure: Decreased plasma protein concentrationSevere liver disease (failure to synthesize proteins)Protein malnutritionNephrotic syndrome (loss of protein in urine).

How well did you know this?
1
Not at all
2
3
4
5
Perfectly
154
Q

which post-Op analgesia given in an asthmatic patient
spinal

sub dural

epidural

morphine

ketorolac

A

E) Reason

Ketorolac is only contraindicated in asthmatic if patient also has known aspirin allergy, otherwise is best post-Op analgesia.

How well did you know this?
1
Not at all
2
3
4
5
Perfectly
155
Q

what changes hb dissociation curve to extreme left
CO2

CO

SO2

NO3

N2O

A

B) Reason

In addition, binding of CO to hemoglobin increases the affinity of remaining sites for O2, causing a shift of the curve to the left

How well did you know this?
1
Not at all
2
3
4
5
Perfectly
156
Q

which nerve damage will lead to supination loss
radial

radial & muskulocutaneus

muskulosketal

ulnar

axillary

A

B) Reason

Supination: Biceps (radial nerve) and supinator (muskulosketal nerve) muscles. This movement returns the pronated hand to the anatomic positionso that the palm faces anteriorly and the thumb lies on thelateral side.

How well did you know this?
1
Not at all
2
3
4
5
Perfectly
157
Q

Role of parathyroid hormone
stimulate adenylate cyclase

inhibit adenylate cyclase

increase IP3

increase guanulate

increase DAG

A

A) Reason

Adenylate cyclase mechanism 1. Hormone binds to a receptor in the cell membrane (step 1).2. GDP is released from the G protein and replaced by GTP (step 2), which activates the G protein. The G protein then activates or inhibits adenylate cyclase. If the G protein is stimulatory (Gs), then adenylate cyclase will be activated

How well did you know this?
1
Not at all
2
3
4
5
Perfectly
158
Q

which receptor cause secretion of epinephrine from adrenal medulla
nicotinic

parasympathetic

sympathetic

nicotinic ganglionic

muscrinic

A

D) Reason

Epinephrine is stress hormone that is secreted by the adrenal medulla in case of stressful conditions by the activation of sympathetic nicotinic ganglionic receptors.

How well did you know this?
1
Not at all
2
3
4
5
Perfectly
159
Q

Bile salts cause
Proteins absorption

absorption of fat

Carbohydrates absorption

Vitamins absorption

minerals absorption

A

B) Reason

Functions of Bile salts: Digestion and absorption of lipids and fat-soluble vitaminsCholesterol excretion (body’s only means of eliminating cholesterol) Antimicrobial activity (via membrane disruption)

How well did you know this?
1
Not at all
2
3
4
5
Perfectly
160
Q

CSF is present in
sub arachnoid space

sub dural

sub pia

spinal canal

epidural

A

A) Reason

• Subarachnoid space lies between the arachnoid and pia mater containing CSF: site of subarachnoid hemorrhage (described later).

How well did you know this?
1
Not at all
2
3
4
5
Perfectly
161
Q

shortest acting hypnotic
dorzolamide

fentinyl

midazolam

ketamine

halothane

A

C) Reason

Midazolam is widely used adjunctively with inhaled anesthetics and intravenous opioids. Theonset of its CNS effects is slower than that of thiopental, and has shortest duration of action.

How well did you know this?
1
Not at all
2
3
4
5
Perfectly
162
Q

Diagnostic test for TB
Pleural Biopsy

AFB

Chest X-ray

Gram staining

wall polysacchride

A

A) Reason

Diagnostic test of T.B. is Pleural Biopsy

How well did you know this?
1
Not at all
2
3
4
5
Perfectly
163
Q

A pt. is diagnosed with bilateral S2-S4 lesion. Consequences?
urinary incontinence

dribbling

UV prolapse

atonic bladder

rectal incontinence

A

D) Reason

Lesion at S2-S4 causes atonic bladder

How well did you know this?
1
Not at all
2
3
4
5
Perfectly
164
Q

milk production is result of which hormone activity?
Dopamine

oxytoxin

5 HT

Prolactin

LH

A

D) Reason

Prolactin—induces and maintains lactation and decreases reproductive function. Oxytocin—assists in milk letdown; also promotes uterine contractions.

How well did you know this?
1
Not at all
2
3
4
5
Perfectly
165
Q

Why to prefer Troponin over CK-MB
fast to detect

low price

fear

tissue specific

sensitive

A

D) Reason

CK-MB is is non-specific enzyme that can also be increased in stress, muscle and other tissue disorder, but troponin is cardiac specific enzyme.

How well did you know this?
1
Not at all
2
3
4
5
Perfectly
166
Q

what cell layers are present at 2nd week of development
epiblast

hypoblast

epiblast & hypoblast

meroblast

xenoblast

A

C) Reason

In 2nd week: • The embryoblast differentiates into the epiblast and hypoblast, forming a bilaminar embryonic disk. • The epiblast forms the amniotic cavity and hypoblast cells migrate to form the primary yolk sac.

How well did you know this?
1
Not at all
2
3
4
5
Perfectly
167
Q

A patient taking carbonic anhydrase for 4 Week, what change will occur
dec. K

dec. Cl
dec. HCO3
inc. HCO3

no change in HCO3

A

C) Reason

TOXICITY Hyperchloremic metabolic acidosis, inc. plasma HCO3, paresthesias, NH3 toxicity, sulfa allergy

How well did you know this?
1
Not at all
2
3
4
5
Perfectly
168
Q

Dicrotic notch in aortic pressure is due to
aortic closure

pulmonary closure

mitral closure

tricuspid closure

bicuspid closure

A

A) Reason

Isovolumic ventricular relaxation, Ventricular volume is constant (isovolumetric) because all of the valves are closed.■ The “blip” in the aortic pressure tracing occurs after closure of the aortic valve and is called the dicrotic notch, or incisura.

How well did you know this?
1
Not at all
2
3
4
5
Perfectly
169
Q

what will be cardiac cycle in sec if H/R varies from 75 to 225/min
0.1 sec.

  1. 26 sec
  2. 22 sec
  3. 29 sec
  4. 3 sec
A

B) Reason

Cardiac cycle = 60 sec/number of beats = 60/225 = 0.26 sec

How well did you know this?
1
Not at all
2
3
4
5
Perfectly
170
Q

short term effect of angiotensin 2?
Vasodilation

inc. renin

vasoconstriction

dec. K absorption
onc. Cl absorption

A

C) Reason

(1) It increases thirst and therefore water intake.(2) It causes vasoconstriction of the arterioles, thereby increasing TPR and arterial pressure

How well did you know this?
1
Not at all
2
3
4
5
Perfectly
171
Q

Purkinje fibers have higher velocity. reason?
fast fibers

more Na channel

less K channel

less Cl channel

less number of myofibrils

A

E) Reason

■ reflects the time required for excitation to spread throughout cardiac tissue.■ depends on the size of the inward current during the upstroke of the action potential. The larger the inward current, the higher the conduction velocity.■ is fastest in the Purkinje system due to less number of myofibrils.

How well did you know this?
1
Not at all
2
3
4
5
Perfectly
172
Q

Which among the following is the most superficial structure in parotid gland
nerve

lymph nodes

vein

artery

gland

A

B) Reason

• Parotid nodes: Situated on usually superficially or within the parotid salivarygland. They receive lymph from the scalp above theparotid gland, the eyelids, the auricle, and the external auditory meatus.

How well did you know this?
1
Not at all
2
3
4
5
Perfectly
173
Q

A person on oral anticoagulant with controlled INR (2-2.5), developed pulmonary edema and INR of 5, what drug caused this change
vancomycin

metronidazole

Clopidogrel

aspirin

ciprofloxacin

A

E) Reason

TOXICITY:Bleeding, teratogenic, skin/tissue necrosis A, drug-drug interactions ( Pulmonary edema with quinolones).

How well did you know this?
1
Not at all
2
3
4
5
Perfectly
174
Q

Gunshot to the radial groove will cause
loss extension of wrist

loss of Flexion of wrist

loss of supination

loss of pronation

loss of reversion

A

A) Reason

Behind and below the deltoid tuberosity isa spiral groove, in which the radial nerve lies. As it supplies extensors of wrist that is why loss of wrist extension due to radial nerve injury.

How well did you know this?
1
Not at all
2
3
4
5
Perfectly
175
Q

An 18 year old boy from remote area, with Hx of high grade fever for 5 days, is now presented with toxic look and dec. platelets and RBC and derranged coagulation, diagnosis
DIC

CML

sepsis

Thrombocytopenia

anemia

A

A) Reason

Widespread activation of clotting leads to a deficiency in clotting factors, which creates a bleeding state.Causes: Sepsis (gram-negative), Trauma, Obstetric complications, acute Pancreatitis, Malignancy, Nephrotic syndrome,

How well did you know this?
1
Not at all
2
3
4
5
Perfectly
176
Q

A foreign body will lodge into which segment?
right bronchus

left bronchi

laft upper

right middle

right upper

A

A) Reason

Right lung is more common site for inhaled foreign body because the right main stem bronchus is wider and more vertical than the left.

How well did you know this?
1
Not at all
2
3
4
5
Perfectly
177
Q

In last trimester what changes are mostly found regarding respiration.
dec. CO

dec. CO2
dec. HCO3
dec. K
dec. O2

A

E) Reason

In 3rd trimester of pregnancy, gravid utrus pushes causes upward push of the diaphragm and decreases tidal volume and hence increases respiratory rate and dec. O2 saturation.

How well did you know this?
1
Not at all
2
3
4
5
Perfectly
178
Q

What extends into sulci and fissures of brain tissue
dura matter

pia mater

Arachnoid

sub Arachnoid

ventricles

A

B) Reason

Pia mater tightly invests the surfaces of the brain and cannot be dissected away, having the same relationship with the brain extends into sulci as well as spinal pia mater.

How well did you know this?
1
Not at all
2
3
4
5
Perfectly
179
Q

Rise in blood flow is seen in a stroke pt. cause?
inc. O2

inc. pCO2
dec. pCO2
dec. K
dec. Na

A

B) Reason

The Cushing reaction is an example of the response to cerebral ischemia. Increases in intracranial pressure cause compression of the cerebral blood vessels, leading to cerebral ischemia and increased cerebral Pco2. The vasomotor center directs an increase in sympathetic outflow to the heart and blood vessels, which causes a profound increase in arterial pressure.

How well did you know this?
1
Not at all
2
3
4
5
Perfectly
180
Q

thrombus of which artery will lead to Infarct of 1/3 of posterior Interventricular septum
LCA

RCA

LAD

LXA

posterior coronary artery

A

B) Reason

Posterior descending/interventricular artery (PDA)— a branch of RCA supplies posterior 1/3 of interventricular septum and posterior walls of ventricles

How well did you know this?
1
Not at all
2
3
4
5
Perfectly
181
Q

Iron absorbed from which part of GIT plays role in iron absorption
ileum

jejunum

duodenum

Colon

stomach

A

C) Reason

Iron is Absorbed as Fe2+ in duodenum.Folate is Absorbed in jejunum and ileum.B12 is Absorbed in terminal ileum along with bile acids, requires intrinsic factor.

How well did you know this?
1
Not at all
2
3
4
5
Perfectly
182
Q

A new born developed hemolysis of severe form, type of hypersensitivity in Erythroblastosis fetalis
type 1 HSR

type 2

type 3 HSR

type 4 HSR

delayed response

A

B) Reason

Type II: Autoimmune hemolytic anemiaPernicious anemiaIdiopathic thrombocytopenic purpuraErythroblastosis fetalisAcute hemolytic transfusion reactions, Rheumatic fever

How well did you know this?
1
Not at all
2
3
4
5
Perfectly
183
Q

Lumber puncture should be done where
above L4

above L3

at L3

above L3

bellow L3

A

A) Reason

A lumbar puncture is used to inject anesthetic material in the epidural space or to withdraw CSF from the subarachnoid space. • A spinal tap is typically performed at the L4-L5 or above L4 interspace.

How well did you know this?
1
Not at all
2
3
4
5
Perfectly
184
Q

Free radicals mainly removed by
transferrin

ferritin

transferrtin

iron

Phosphatase

A

A) Reason

Free radicals can be eliminated by enzymes (e.g., catalase, superoxide dismutase, transferrin, glutathione peroxidase), spontaneous decay, antioxidants (e.g., vitamins A, C, E).

How well did you know this?
1
Not at all
2
3
4
5
Perfectly
185
Q

After an accident an old lady can’t understand spoken words but canunderstand written words lesion in ?
broca

wernicke

temporal lobe lesion

primary auditory

secondary auditory

A

D) Reason

Patient can understand the written words indicating that his visual and the frontal cortex are intact but only problem is the primary auditory cortex lesion

How well did you know this?
1
Not at all
2
3
4
5
Perfectly
186
Q

Regarding neoplasm, best way to detect spreading tumors
grading

tumor marker

staging

HIDA scan

bone scan

A

C) Reason

Staging of the tumors is the best way to detect the spread and extent of the tumors, one important system is the TNM staginh system.

How well did you know this?
1
Not at all
2
3
4
5
Perfectly
187
Q

What is the span of the isthmus of the thyroid
2nd tracheal ring

2nd to 4th tracheal ring

3rd-4th tracheal

3-5th tracheal ring

5th-6th tracheal ring

A

B) Reason

The thyroid gland (from 2nd-6th tracheal rings) consists of right and left lobes connectedby a narrow isthmus from 2nd-4th tracheal rings. It is surrounded by a sheathformed of pretracheal fascia, and this sheath attaches thegland to the larynx and the trachea. I

How well did you know this?
1
Not at all
2
3
4
5
Perfectly
188
Q

patient has Tedency to fall on left, lesion is on
basal ganglia lesion

flocular lobe

cerebellar peduncle

left cerebellum

right cerebellum

A

D) Reason

Cerebellum Modulates movement; aids in coordination and balance.Input:Contralateral cortex via middle cerebellar peduncle. Ipsilateral proprioceptive information via inferior cerebellar peduncle from the spinal cord (input nerves = climbing and mossy fibers). Lesion causes ipsilatral ataxia.

How well did you know this?
1
Not at all
2
3
4
5
Perfectly
189
Q

No x wave is found on EEG in a pt. Dx?
REM

Non REM

wake

deep sleep

opened eyes

A

A) Reason

REM sleep (25%) beta waves: Loss of motor tone, brain O2 use, and variable pulse and blood pressure; when dreaming and penile/clitoral tumescence occur; may serve a memory processing function

How well did you know this?
1
Not at all
2
3
4
5
Perfectly
190
Q

An 11 year boy developed carpopedal spam after radius fracture, serum Ca 6.8 urinary Ca is normal, cause..?
rickets

hypokalemia

secondary hypoparathyroid

tertiary hypoparathyroid

primary hypoparathyroid

A

A) Reason

Rickets is the most appropriate answer.

How well did you know this?
1
Not at all
2
3
4
5
Perfectly
191
Q

vertical nystagmus is due to?
occipital lobe lesion

basal ganglia lesion

tippling up and down

cerebellar lesion

tipping head sideways

A

E) Reason

Lesions of the vestibular nuclei and pathways may produce nystagmus, vertigo, nausea, and vomiting. It can be horizontal by up & down tippling or vertical by side by side tippling of head.

How well did you know this?
1
Not at all
2
3
4
5
Perfectly
192
Q

Gram negative UTI,culture gives foul smelling greenish colonies treatment .
clindanycin

metronidazole

vancomycin

gentamicin

ampicillin

A

E) Reason

E.Coli is the causative agent here.

How well did you know this?
1
Not at all
2
3
4
5
Perfectly
193
Q

Tumor involved superior bronchus of lung, which segment is likely to collapse?
apical

middle

lower

superior

7th segment

A

A) Reason

Superior Bronchus of the lungs is closely related with the apical segment and it contains superior apical segment that can be involved in carcinoma of superior bronchus.

How well did you know this?
1
Not at all
2
3
4
5
Perfectly
194
Q

Regarding carcinogenesis of CA stomach, which cause CA stomach
alcohol

hydrocarbon

phenacetin

aniline

H. pylori

A

E) Reason

Stomach cancer:Almost always adenocarcinoma. Early aggressive local spread and node/liver metastases. Often presents with acanthosis nigricans. Intestinal—associated with H. pyloriinfection, dietary nitrosamines (smoked foods), tobacco smoking, achlorhydria, chronic gastritis.

How well did you know this?
1
Not at all
2
3
4
5
Perfectly
195
Q

A 50 year old man with factory worker exposed to hydrocarbons is at risk of
pancreatic cancer

HCC

CA stomach

CA urinary bladder

CA lung

A

D) Reason

Risk Factors of CA Urinary bladder are: Associated with problems in your Pee SAC: Phenacetin, Hydrocarbons, Smoking, Aniline dyes, and Cyclophosphamide.

How well did you know this?
1
Not at all
2
3
4
5
Perfectly
196
Q

reflex system of ANS is directly controlled by?
medulla

pons

Midbrain

cerebrum

cerebellum

A

D) Reason

A large part of the frontal cortex rostral to the central sulcus is related to the control of movements, reflex action, primarily on the opposite side of the body

How well did you know this?
1
Not at all
2
3
4
5
Perfectly
197
Q

A boy ARF now has fever with pancystolic murmur and speelomegaly, treatment
gentamicin

Penicillin

metronidazole

clarythromycin

azithromycin

A

B) Reason

IE: Mitral valve is most frequently involved. Tricuspid valve endocarditis is associated with IV drug abuse (don’t “tri” drugs). Associated with S. aureus, Pseudomonas, and Candida.Complications: chordae rupture, glomerulonephritis, suppurative pericarditis, emboli. Treated with with IV penicillin.

How well did you know this?
1
Not at all
2
3
4
5
Perfectly
198
Q

A 14 year old boy with Spleenomegaly and all cell line dec. what investigation
CBC

USG

CT

periphral smear

bone marrow biopsy

A

E) Reason

Characteristics include decreased platelet count and prolonged bleeding time; bone marrow aspiration reveals decreased megakaryocytes when caused by decreased platelet production and increased megakaryocytes when caused by increased platelet destruction.

How well did you know this?
1
Not at all
2
3
4
5
Perfectly
199
Q

Regarding carcinogenesis, which cancer is related to oncogene
CA uterus

CA ovary

CA breast

Cervical cancer

HCC

A

C) Reason

Risk factors: inc. estrogen exposure, inc. total number of menstrual cycles, older age at 1st live birth, obesity (inc. estrogen exposure as adipose tissue converts androstenedione to estrone), BRCA1and BRCA2 gene mutations, African American ethnicity (inc. risk for triple breast cancer)

How well did you know this?
1
Not at all
2
3
4
5
Perfectly
200
Q

sesamoid cartilage is found in
elbow

vertebral disc

ear lobule

knee

ala of nose

A

E) Reason

NASAL CAVITY: The nasal cavity is divided into right and left halves by thenasal septum. Ala of nose contain a small suspended cartilage, sesamoid cartilage. The septum is made up of the septal cartilage, the vertical plate of the ethmoid, and the vomer.

How well did you know this?
1
Not at all
2
3
4
5
Perfectly
201
Q

19 years old girl history of repeated infections , CBC done Hb=>5, WBC=>3.5 , plt40,000 , neutrophils18%, lymphosytes 80%, next to do ?
marrow biopsy

CBC

periphral smear

Electrophoresis

USG abdomen

A

A) Reason

All cell line are decreased RBCs, WBCs and Platelets (Pancytopenia) in a case of Aplastic anemia manifests as anemia, repeated infections and dec. platelets, can be diagnosed by bone marrow biopsy (shows pancytopenia)

How well did you know this?
1
Not at all
2
3
4
5
Perfectly
202
Q

Skilled motor activity is carried via
corticobulbar

spinothalamic

corticospinal tract

medial leminiscus

faciculus cuneatus

A

C) Reason

Corticospinal tract: UMNcell body in 1° motor cortex then descends ipsilaterally (through internal capsule), most fibers decussate at caudal medulla (pyramidal decussation) then descends contralaterally. Associated with skilled motor actions.

How well did you know this?
1
Not at all
2
3
4
5
Perfectly
203
Q

part of ECG from end of P wave to start of R wave is known as
PR interval

PR wave

PR segment

QR wave

QR segment

A

C) Reason

PR segment is the part of ECG from end of P wave to start of R wave and it represents the atrial depolarization.

How well did you know this?
1
Not at all
2
3
4
5
Perfectly
204
Q

Chief parameter for plasma blood flow measurment?
Urea

Creatinine

Inulin

PABA

PAH

A

E) Reason

Measurement of renal plasma flow (RPF)—clearance of para-aminohippuric acid (PAH)■ PAH is filtered and secreted by the renal tubules.■ Clearance of PAH is used to measure RPF.

How well did you know this?
1
Not at all
2
3
4
5
Perfectly
205
Q

patient with RHC pain now developed lung abscess, route of spread?
Hematogenous

lymphatic

direct spread

veinous

peritoneal

A

C) Reason

In the hiven scenario, the patient was fisrt having liver abscess that later on directly invaded into the chest causing right lung abscess.

How well did you know this?
1
Not at all
2
3
4
5
Perfectly
206
Q

All are rick Factors for CA stomach except
H. pylori

blood type O

smoking

nitrites

smoked food

A

B) Reason

associated with H. pyloriinfection, dietary nitrosamines (smoked foods), tobacco smoking, achlorhydria, chronic gastritis. Commonly on lesser curvature; looks like ulcer with raised margins.

How well did you know this?
1
Not at all
2
3
4
5
Perfectly
207
Q

right sided relation of trachea as it descends into Superior mediastinum
hemi azygus vein

Phrenic nerve

sympathetic chain

vagus nerve

azygus vein

A

E) Reason

Relations of Trachea in Superior Mediastinum are Right side: Azygos vein, right vagus nerve, and pleura.• Left side: Arch of the aorta, left common carotid, left subclavian arteries, left vagus nerve, left phrenic nerve, andpleura.

How well did you know this?
1
Not at all
2
3
4
5
Perfectly
208
Q

single gene autosomal dominant
Achondroplasia

Fredreck ataxia

Cri du chat syndrome

PCOD

meatal atresia

A

A) Reason

Achondroplasia: Is a single gene autosomal dominant trait. Failure of longitudinal bone growth (endochondral ossification) leading to short limbs. Membranous ossification is not affected and cause large head relative to limbs.

How well did you know this?
1
Not at all
2
3
4
5
Perfectly
209
Q

how to differentiate leukocytes from other cells on bone marrow
golgi complex

RER

cell membrane

staining

presence of nuclei

A

E) Reason

Leukocyte: Are the nucleated cell developed in marrow.Divided into granulocytes (neutrophil, eosinophil, basophil) and mononuclear cells (monocytes, lymphocytes). Responsible for defense against infections.

How well did you know this?
1
Not at all
2
3
4
5
Perfectly
210
Q

anterior mediastinum consists of?
thymus

heart

Phrenic nerve

azygus vein

trachea

A

A) Reason

ANTERIOR MEDIASTINUMThe contents of the anterior mediastinum include thesternopericardial ligaments, lymph nodes, and remains ofthymus.

How well did you know this?
1
Not at all
2
3
4
5
Perfectly
211
Q

A pt. has cardiac pacemaker. on ECG p wave absent & QRS present. location on pacemaker?
bundle of His

ventricle

Atria

AV node

SA node

A

D) Reason

Normal pacemaker of the heart is the SA node, but in case of any ectopic pacemaker especially the AV node, the P wave will be absent but the QRS complexes will be present.

How well did you know this?
1
Not at all
2
3
4
5
Perfectly
212
Q

alongwith thermo receptors, pain receptors also carry sensation of
cold

tactile

pressure

touch

vibration

A

A) Reason

Nociceptors: ■ are associated with the detection and perception of noxious stimuli (nociception).■ The receptors for pain are free nerve endings in the skin, muscle, and viscera. They also sense for extreme of temperature (heat and cold)

How well did you know this?
1
Not at all
2
3
4
5
Perfectly
213
Q

Regarding fibrocartilage truth is
elbow

knee

intervertebral disc

hip joint

sutures

A

C) Reason

Fibrocartilage has type I collagen in addition to type II, giving it greater resistance to being stretched (tensile strength), and is the type of cartilage found in intervertebral disks of the vertebral column and the menisci of the knee, and may form the attachment of ligaments and tendons to bone

How well did you know this?
1
Not at all
2
3
4
5
Perfectly
214
Q

A 54 yr lady concerned with her family history, with positive family H/O CA breast and IHD, what to take to avoid fracture
Ca supplements

complete HRT

vit. D

estrogen

tamoxifen

A

B) Reason

Hormone replacement therapy: Used for relief or prevention of menopausal symptoms (e.g., hot flashes, vaginal atrophy) and osteoporosis (decrease estrogen, decrease osteoclast activity).

How well did you know this?
1
Not at all
2
3
4
5
Perfectly
215
Q

In pseudostratified epithelium, nuclei lies
at lower pole

at different level

upper pole

lateral border

at top

A

B) Reason

The epithelial lining of the trachea and bronchi is pseudostratified columnar in which all cells lie on the same basal membrane with their nuclei at different levels but only some reach the luminal surface.

How well did you know this?
1
Not at all
2
3
4
5
Perfectly
216
Q

How does K enters in cells of hypokalemic pt?
K gates

Na/K ATPase

K/Cl chanel

H-K exchange

K/HCO3 chanel

A

B) Reason

Na+/K+ATPase (or Na–K+ pump) in cell membranes transports Na+ from intracellular to extracellular fluid and K+ from extracellular to intracellular fluid; it maintains low intracellular [Na+] and high intracellular [K+].

How well did you know this?
1
Not at all
2
3
4
5
Perfectly
217
Q

Inc. neuronal activity before skilled action occurs in
cortical association area

motor cortex

prefrontal

temporal lobe

uncus

A

A) Reason

Lesions, usually in the dominant hemisphere and which include areas 5 and 7 of the posterior parietal association areas, often result in apraxia (also seen with lesions to the premotor cortex). Apraxia is a disruption of the patterning and execution of learned motor movements

How well did you know this?
1
Not at all
2
3
4
5
Perfectly
218
Q

most early sign of vit. A deficiency
keratitis

keratosis

dry skin

Bitot spot

night blindness

A

E) Reason

DEFICIENCY: Night blindness (nyctalopia); dry, scaly skin (xerosis cutis); alopecia; corneal degeneration (keratomalacia); immune suppression.

How well did you know this?
1
Not at all
2
3
4
5
Perfectly
219
Q

viscosity of blood is mainly due to?
WBCs

platelets

lymphocytes

RBCs

plasma proteins

A

D) Reason

Erythrocyte: Carries O2 to tissues and CO2 to lungs. Anucleate and biconcave A, with large surface area-to-volume ratio for rapid gas exchange. It is the main contributer of blood viscosity.

How well did you know this?
1
Not at all
2
3
4
5
Perfectly
220
Q

Which bone forms glenoid fossa?
temporal

mandible

maxilla

zygomatic

vomer

A

A) Reason

The temporomandibular joint articulates above with themandibular fossa (glenoid fossa) and the articular tubercle of the temporalbone. It articulates below with the head of the mandible.

How well did you know this?
1
Not at all
2
3
4
5
Perfectly
221
Q

A 42 year old patient has chronic cough and right lung upper lobe consolidation and fatty liver with inc. AST and ALT, cause
hepatitis C

chronic hepatitis B

acute hepatitis A

chronic alcoholism

acute hepatitis D

A

D) Reason

Alcoholic cirrhosis: Final and irreversible form. Micronodular, irregularly shrunken liver with “hobnail” appearance. Sclerosis around central vein (zone III). Has manifestations CLD, derranged coagulation, raised AST and ALT and sometimes Pnemonia.

How well did you know this?
1
Not at all
2
3
4
5
Perfectly
222
Q

In left suprarenal surgery what is likely to be damaged?
renal vein

IVC

aorta

sympathetic chain

renal artery

A

B) Reason

VEINS:There is a single vein on each side. The right suprarenal veindrains into the inferior vena cava; the left suprarenal veindrains into the left renal vein.

How well did you know this?
1
Not at all
2
3
4
5
Perfectly
223
Q

newly laid uncalcified bone matrix is called
osteoblast

osteoid

spongy material

costeod

immature bone

A

B) Reason

Osteoblasts and osteocytes produce the type I collagen in bone, along with other components of bone ECM (osteoid). That is ossified later on.

How well did you know this?
1
Not at all
2
3
4
5
Perfectly
224
Q

After cerebral hemorrhage pt. lacks motivation. site of involvment?
frontal lobe

temporal lobe

parietal lobe

basal ganglia

uncus

A

B) Reason

Temporal lobe lesion causes depression and loss of motivation along with auditory impairment. Patients with unilateral damage to the primary auditory cortex show little loss of auditory sensitivity but have some difficulty in localizing sounds in the contralateral sound field.

How well did you know this?
1
Not at all
2
3
4
5
Perfectly
225
Q

A man got stab wound on right chest, consequences?
contralatral collapse

open pneumothorax

ipsilatral collapse

right consolidation

left consolidation

A

C) Reason

During the chest stab wound, ipsilatral lung is developed into an opened valve pneumothorax leading to ipsilatral lung collapse.

How well did you know this?
1
Not at all
2
3
4
5
Perfectly
226
Q

progesterone has a vital role which is
dec. uterus contraction

inc. utrus contraction

ripen cervix

mature the follicle

prevent leakage

A

A) Reason

FUNCTION: Stimulation of endometrial glandular secretions and spiral artery developmentMaintenance of pregnancydec. myometrial excitability

How well did you know this?
1
Not at all
2
3
4
5
Perfectly
227
Q

Amoebic liver abscess, treatment
tinidazole

ampicillin

gentamicin

ciprofloxacin

neomycin

A

A) Reason

Entamoeba hisrolytica:Amebiasis: bloody diarrhea (dysentery), liver abscess (“anchovy paste” exudate), RUQ pain (histology shows flask-shaped ulcer if submucosal abscess of colon ruptures). Treated with metronidazole and tinidazole.

How well did you know this?
1
Not at all
2
3
4
5
Perfectly
228
Q

basal ganglia has avital neurotransmitter which is?
glutamate

Aspartate

glycine

GABA

Ach

A

D) Reason

Excitatory pathway—cortical inputs stimulate the striatum, stimulating the release of GABA, which disinhibits the thalamus via the GPi/SNr (inc. motion).Inhibitory pathway—cortical inputs stimulate the striatum, which disinhibits STN via GPe, and STN stimulates GPi/SNr to inhibit the thalamus (dec. motion).

How well did you know this?
1
Not at all
2
3
4
5
Perfectly
229
Q

cause of fatty liver in our society
smoking

alcohlism

fast food

obesity

diabetes

A

D) Reason

In Western society, most common cause of fatty liver is alcohlism and in our society where alcohlism is not common, here cause is obesity.

How well did you know this?
1
Not at all
2
3
4
5
Perfectly
230
Q

Time from atrial depolarization to start of ventriclar depolarization
P wave

ST wave

PR wave

PR segment

PR interval

A

E) Reason

P wave—atrial depolarization. Atrial repolarization is masked by QRS complex.PR interval is the time from atrial depolarization to start of ventriclar depolarization—represents conduction delay through AV node (normally < 200 msec).

How well did you know this?
1
Not at all
2
3
4
5
Perfectly
231
Q

ICF & ECF differ by
has no cations

has inorganic anions

has more Na

has less K

has more K

A

E) Reason

Intracellular fluid (ICF)■ is two-thirds of TBW.■ The major cations of ICF are K+ and Mg2+

How well did you know this?
1
Not at all
2
3
4
5
Perfectly
232
Q

Bronchoconstriction on taking penicillin, now drug of choice
Norepinephrine

epinephrine

glutamine

acetylcholine

ranitidine

A

B) Reason

Epinephrine: Affects β > α Clinical uses: Anaphylaxis, open angle glaucoma, asthma, hypotension; α effects predominate at high doses.

How well did you know this?
1
Not at all
2
3
4
5
Perfectly
233
Q

a person lost hearing in a bomb blast, type of deafness
conductive

sensory

mixed

central

destructive

A

C) Reason

As in case of blast injury, both the eardrum and choclea are affected, so mixed type of hearing loss it is

How well did you know this?
1
Not at all
2
3
4
5
Perfectly
234
Q

A rock crusher developed lymphadenopathy and bireferengence on biopsy, Dx..?
asbestosis

pneumoconiosis

anthracosis

byssinosis

silicosis

A

E) Reason

Rock crush is associated with silicosis exposure

How well did you know this?
1
Not at all
2
3
4
5
Perfectly
235
Q

Patient after an RTA has pH of 7.5, CO2 as 20mmHg and HCO3 as 20meq. Dx…?
respiratory acidosis

metabolic acidosis

respiratory alkalosis

metabolic alkalosis

ARDS

A

C) Reason

Patient given in this scenario is having high pH (alkalosis) and dec. CO2 shows it to be respiratory alkalosis. Respiratory alkalosis■ is caused by increased alveolar ventilation and loss of CO2.a. Decreased arterial Pco2, which is the primary disturbance, causes a decrease in [H] and [HCO3] by mass action.

How well did you know this?
1
Not at all
2
3
4
5
Perfectly
236
Q

Patient after an RTA has pH of 7.5, CO2 as 20mmHg and HCO3 as 20meq. Dx…?
respiratory acidosis

metabolic acidosis

respiratory alkalosis

metabolic alkalosis

ARDS

A

C) Reason

Patient given in this scenario is having high pH (alkalosis) and dec. CO2 shows it to be respiratory alkalosis. Respiratory alkalosis■ is caused by increased alveolar ventilation and loss of CO2.a. Decreased arterial Pco2, which is the primary disturbance, causes a decrease in [H] and [HCO3] by mass action.

How well did you know this?
1
Not at all
2
3
4
5
Perfectly
237
Q

Bicarb absorb in nephron at?
PCT and DCT

thick limb

loop of henle

only DCT

PCT only

A

E) Reason

a. Early proximal tubule—special features■ reabsorbs Na+ and H2O with HCO3 glucose, amino acids, phosphate, and lactate.

How well did you know this?
1
Not at all
2
3
4
5
Perfectly
238
Q

patient with CRF has productive cough, drug of choice
gentamicin

vancomycin

methotrexate

Cisplatin

ciprofloxacin

A

E) Reason

As the patient is having compromised renal functions, and quinolones are relatively safe in CRF than Aminoglycosides and penicillin.

How well did you know this?
1
Not at all
2
3
4
5
Perfectly
239
Q

A lady with HTN, hirsuitism and inc. ACTH, cause
paraneoplastic syndrome

Wilms tumors

pheochromocytoma

Cushing syndrome

histiocytoma

A

D) Reason

FINDINGS: Hypertension, weight gain, moon facies, truncal obesity, buffalo hump, hyperglycemia (insulin resistance), skin changes (thinning, striae), osteoporosis, amenorrhea, and immune suppression.

How well did you know this?
1
Not at all
2
3
4
5
Perfectly
240
Q

Patient presents with hypoventilation and pin point pupil, cause
morphine poisoning

atropine poisoning

organophosphate

CCl4 poisoning

alcohlism

A

A) Reason

TOXICITY: Addiction, respiratory depression, constipation, miosis (pinpoint pupils), additive CNS depression with other drugs.

How well did you know this?
1
Not at all
2
3
4
5
Perfectly
241
Q

final diagnosis of COPD is by
V/Q= 0

V/Q= 0.1

V/Q= infinite

V/Q= 2

V/Q= 1

A

B) Reason

V/Q approaches 0 = airway obstruction (shunt) and in COPD. In shunt, 100% O2 does not improve Po2.V/Q approaches ∞ = blood flow obstruction (physiologic dead space). Assuming < 100% dead space, 100% O2 improves Po2

How well did you know this?
1
Not at all
2
3
4
5
Perfectly
242
Q

lesion in non-dominant Broca”s area
global aphasia

sensory aphsia

anomic aphasia

agraphesthesia

combined aphasia

A

C) Reason

Broca area Lesion: Nonfluent/Anomic aphasia with intact comprehension.Broca area—inferior frontal gyrus of frontal lobe.

How well did you know this?
1
Not at all
2
3
4
5
Perfectly
243
Q

sulphar containing amino acid
glutamate

glycine

aspartate

methionine

tryptophan

A

D) Reason

• Sulphur containing amino acids: Cysteine,Methionine • Amino acids with amide group: Asparagine, Glutamine

How well did you know this?
1
Not at all
2
3
4
5
Perfectly
244
Q

Blunt trauma immediately below and lateral to knee will injure
common peroneal

tibial

sural

anterior tibial

posterior tibial

A

A) Reason

The common peroneal nerve descends through thepopliteal fossa. It then passes laterally aroundthe neck of the fibula, pierces the peroneus longus muscle,and divides into the superficial and the deep peroneal nerves.

How well did you know this?
1
Not at all
2
3
4
5
Perfectly
245
Q

RR 18/min, tidal volume 350ml and dead space is 100ml, alveolar ventilation would be?
2.5 L

3 L

4 L

4.5L

5L

A

D) Reason

TV-D space x RR = 350-100 x 18 = 250 x 18= 4500ml/min = 4.5L/ min

How well did you know this?
1
Not at all
2
3
4
5
Perfectly
246
Q

At which level does 10th intercostal nerve enter rectus sheet?
hypochondrium

lumber region

hypogastrium

umblicus

iliac fossa

A

D) Reason

T 10 vertebra is located at the level of umblicus, and its nerve root enters the rectus sheet at its level

How well did you know this?
1
Not at all
2
3
4
5
Perfectly
247
Q

Premalignant lesion of SCC is
lichen planus

actinic keratitis

dermatitis

leukoplakia

keratosis

A

B) Reason

Actinic keratosis:1. This premalignant epidermal lesion is caused by chronic excessive exposure to sunlight. 2. Characteristics include rough, scaling, poorly demarcated plaques on the face, neck, upper trunk, or extremities

How well did you know this?
1
Not at all
2
3
4
5
Perfectly
248
Q

Patient with recurrent infections and hypocalemia since birth, what is the defect
inc. cell in paracortex

dec. cell in paracortex

abnormal T cells

IgA deficiency

abmormal B cells

A

B) Reason

Paracortex Houses T cells. Region of cortex between follicles and medulla. Contains high endothelial venules through which T and B cells enter from blood. Not well developed in patients with DiGeorge syndrome.

How well did you know this?
1
Not at all
2
3
4
5
Perfectly
249
Q

In SLE, biopsy shows immune complexes deposits on renal biopsy, mechanism is?
thick basement membrane

Antigen-Ig complexes

effaced membrane

Villous atrophy

dec. connective tissue

A

B) Reason

Systemic lupus erythematosus (SLE) 1. SLE, the prototype connective tissue disease, most often affects women (80% of patients), usually those of childbearing age. 2. SLE is marked by the presence of a spectrum of ANAs and by extensive deposits of antigen-antibody complexes mediated inflammatory lesions involving multiple organ systems.

How well did you know this?
1
Not at all
2
3
4
5
Perfectly
250
Q

Pt. is undergoing EEG with eyes closed but fully awake. which waves will be seen?
gamma wave

delta wave

theta wave

alpha waves

beta wave

A

D) Reason

Awake (eyes open) Alert, active mental concentration Beta (highest frequency, lowest amplitude)Awake (eyes closed) Alpha waves.

How well did you know this?
1
Not at all
2
3
4
5
Perfectly
251
Q

most common aquired thrombotic disorder
OCP induced thrombosis

Bud chiari syndrome

pulmonary embolism

SLE

anti phospholipid syndrome

A

E) Reason

Antiphospholipid Antibody Syndrome: This syndrome (previously called the lupus anticoagulant syndrome) is the most common aquired thrombotic disorder, it has protean clinical manifestations, including recurrent thromboses, repeated miscarriages, cardiac valve vegetations, and thrombocytopenia.

How well did you know this?
1
Not at all
2
3
4
5
Perfectly
252
Q

Graft between genetically identical twins
need HLA matching

lab. limitation

ethically not done

not rejected

sometime rejected

A

D) Reason

Identical twins are the genetically same to each other in all the antigenic and cellular proteins, so their no tissue rejection reaction occur in transplant of identical twins.

How well did you know this?
1
Not at all
2
3
4
5
Perfectly
253
Q

Drug that dec. LES and inc. emptying
domperidone

5HT3 inhibitor

Metoclopramide

ampicillin

eryhromycin

A

C) Reason

MECHANISM: D2 receptor antagonist. decrease resting tone, contractility, LES tone, motility. Does not influence colon transport time.

How well did you know this?
1
Not at all
2
3
4
5
Perfectly
254
Q

A young boy has a soft to firm mass below hyoid from past a few months.What is the diagnosis?
adenoma

thyroglossal sinus

thyroglossal fistula

MNG

thyroglossal duct cyst

A

E) Reason

Thyroglossal duct cyst presents as an anterior midline neck mass that moves with swallowing or protrusion of the tongue (vs. persistent cervical sinus leading to branchial cleft cyst in lateral neck).

How well did you know this?
1
Not at all
2
3
4
5
Perfectly
255
Q

drug to avoid in CRF
ampicillin

cisplatin

ciprofloxacin

metronidazole

6-MP

A

B) Reason

TOXICITY of Cisplatin and Carboplatin: Nephrotoxicity and acoustic nerve damage. Prevent nephrotoxicity with amifostine (free radical scavenger) and chloride diuresis.

How well did you know this?
1
Not at all
2
3
4
5
Perfectly
256
Q

failure of ant. neuropore to close
myelocele

miningeocele

miningomyelocele

acraniotomy

anencephaly

A

E) Reason

Anencephaly: Malformation of anterior neural tube resulting in no forebrain, open calvarium (“frog-like appearance”). Clinical findings: inc. AFP; polyhydramnios (no swallowing center in brain). Associated with maternal diabetes (type I). Maternal folate supplementation dec. risk.

How well did you know this?
1
Not at all
2
3
4
5
Perfectly
257
Q

Propylthiouracil oxidizes what
monoiodide

iodide

thyroglobulin

thyroid binding globulin

thyroxine

A

B) Reason

MECHANISM: Block thyroid peroxidase, inhibiting the oxidation of iodide and the organification (coupling) of iodine leading to inhibition of thyroid hormone synthesis. Propylthiouracil also blocks 5′-deiodinase, which dec. peripheral conversion of T4 to T3

How well did you know this?
1
Not at all
2
3
4
5
Perfectly
258
Q

blood antigen secreted in
urine

marrow

feaces

saliva

csf

A

D) Reason

Crosses epithelial cells by transcytosis. Most produced antibody overall, but released into secretions (tears, saliva, mucus) and early breast milk (known as colostrum).

How well did you know this?
1
Not at all
2
3
4
5
Perfectly
259
Q

pain of appendicitis is referred to?
T8

T9

T10

T11

T12

A

C) Reason

PAIN OF APPENDICITISVisceral pain in the appendix is produced by distention of its lumen or spasm of its muscle. The afferentpain fibers enter the spinal cord at the level of thetenth thoracic segment, and a vague referred pain isfelt in the region of the umbilicus (T10). Later, the pain shiftsto where the inflamed appendix irritates the parietalperitoneum, and then the pain is precise, severe, and localized.

How well did you know this?
1
Not at all
2
3
4
5
Perfectly
260
Q

Gastric ulcer is most commonly found at
antrum

pylorus

body

fundus

cardia

A

A) Reason

Disruption of mucosal barrier leads inflammation. Antrum is the most common site of ulceration.Can be caused by stress, NSAIDs (dec. PGE2and hence dec. gastric mucosa protection), alcohol, uremia.

How well did you know this?
1
Not at all
2
3
4
5
Perfectly
261
Q

while inspiring s2 splits as a result of?
late aortic closure

early closure of aortic

early pulmonary close

late tricuapid closure

late bicuspid closure

A

B) Reason

Normal splitting: Inspiration, drop in intrathoracic pressure inc. venous return to the RV increased RV stroke volume leading to decreased RV ejection time causing delayed closure of pulmonic valve. decreased pulmonary impedance (increased capacity of the pulmonary circulation) also occurs during inspiration, which contributes to delayed closure of pulmonic valve.

How well did you know this?
1
Not at all
2
3
4
5
Perfectly
262
Q

which phase of sleep has theeta waves?
REM sleep

wake

non REM stage 1

colsed eyes

frightened

A

C) Reason

Non-REM sleep:Stage N1: (5%) Light sleep Thetawaves.Stage N2: (45%) Deeper sleep; when bruxism occurs Sleep spindles and K complexesStage N3: (25%) Deepest non-REM sleep (slow-wave sleep); when sleepwalking, night terrors, and bedwetting occur.Delta waves: (lowest frequency, highest amplitude)

How well did you know this?
1
Not at all
2
3
4
5
Perfectly
263
Q

what is the Origin of systemic arterial thrombi
pulmonary embolism

veinous thrombi

mural thrombi

periphral fibrosis

MI

A

C) Reason

Arterial thrombi a. These thrombi are formed inside the vessels in the areas of active blood flow. b. When mature, they demonstrate alternate dark gray layers of platelets interspersed with lighter layers of fibrin. This layering results in the lines of Zahn.

How well did you know this?
1
Not at all
2
3
4
5
Perfectly
264
Q

right sided hemiplegia with no sensory loss, lesion is in
Right cerebral cortex

cerebellam

medulla oblongata

internal capsule

mid brain

A

D) Reason

Most commonly caused by systemic hypertension D . Also seen with amyloid angiopathy, vasculitis, and neoplasm. Typically occurs in basal ganglia and internal capsule (Charcot-Bouchard aneurysm of lenticulostriate vessels), but can be lobar. Presents with crossed hemiplegia.

How well did you know this?
1
Not at all
2
3
4
5
Perfectly
265
Q

right sided hemiplegia with no sensory loss, lesion is in
Right cerebral cortex

cerebellam

medulla oblongata

internal capsule

mid brain

A

D) Reason

Most commonly caused by systemic hypertension D . Also seen with amyloid angiopathy, vasculitis, and neoplasm. Typically occurs in basal ganglia and internal capsule (Charcot-Bouchard aneurysm of lenticulostriate vessels), but can be lobar. Presents with crossed hemiplegia.

How well did you know this?
1
Not at all
2
3
4
5
Perfectly
266
Q

Regarding kidney what is right?
separate fascia for gland

left relates to aorta

right is little bellow

covered with fat

covered with fascial sheath

A

E) Reason

• Renal fascia: This is a condensation of areolar tissue outside the perirenal fat. It encloses the kidneys and thesuprarenal glands.

How well did you know this?
1
Not at all
2
3
4
5
Perfectly
267
Q

1 alpha hydroxilation of vit. D takes place in
liver

kidney

lungs

suprarenal

spleen

A

B) Reason

SOURCE: D3 from sun exposure in skin. D2 ingested from plants. Both converted to 25-OH in liver and to 1,25-(OH)2 (active form) in kidney

How well did you know this?
1
Not at all
2
3
4
5
Perfectly
268
Q

Patient presents with parotid swelling & pain that exaggerate on chewing, nerve responsible is?
facial

ophthalmic nerve

auriculotemporal

mandibular nerve

maxillary nerve

A

C) Reason

• The auriculotemporal nerve, which supplies the skin ofthe auricle, the external auditory meatus, the temporomandibular joint, and the scalp. This nerve also conveys postganglionic parasympathetic secretomotor fibers fromthe otic ganglion to the parotid salivary gland .

How well did you know this?
1
Not at all
2
3
4
5
Perfectly
269
Q

How will ECG represent ventricular depolarization?
PR wave

Q wave

QRS complex

T wave

P wave

A

C) Reason

QRS complex—ventricular depolarization (normally < 120 msec).QT interval—mechanical contraction of the ventricles.

How well did you know this?
1
Not at all
2
3
4
5
Perfectly
270
Q

patient with fever, fits, dark urine and shivering, Dx..?
epilepsy

minigitis

cerebral malaria

encephalitis

hydrocephalus

A

C) Reason

All the features given in scenario are characteristics of black water fever ( dark colored urine is the clue), while CNS involvement is the point to the Dx of Cerebral Malaria.

How well did you know this?
1
Not at all
2
3
4
5
Perfectly
271
Q

Which of the following vessels does NOT supply the retina?
anterior cilliary

lateral Cilliary

superior Cilliary

posterior Cilliary artey

upper Cilliary

A

D) Reason

Blood Supply1. The choriocapillaris of the choroid supplies the pigment epithelium, the layers of rods and conesand the outer nuclear layers.2. Central retinal artery supplies rest of the layers of the retina. It is a branch of the ophthalmicartery and is an end artery

How well did you know this?
1
Not at all
2
3
4
5
Perfectly
272
Q

which phase of cardiac cycle posseses maximum aortic pressure?
rapid ejection

slow ejection phase

slow relaxation

rapid filling

slow contraction

A

B) Reason

Durig the slow ejection phase, Ventricular volume is constant (isovolumetric) because all of the valves are closed.■ The “blip” in the aortic pressure tracing occurs after closure of the aortic valve and increased pressure.

How well did you know this?
1
Not at all
2
3
4
5
Perfectly
273
Q

testosterone plays a role in puberty that is
thinning of laryngeal cartilage

thickening of laryngeal cartilage

edema of larynx

edema of pharynx

thinning of bones

A

B) Reason

FUNCTION of Testosterone: Differentiation of epididymis, vas deferens, seminal vesicles (genitalia, except prostate)Growth spurt: penis, seminal vesicles, sperm, muscle, RBCsDeepening of voice(thickens laryngeal cartilage) Closing of epiphyseal plates (via estrogen converted from testosterone) Libido

How well did you know this?
1
Not at all
2
3
4
5
Perfectly
274
Q

fetal part of placenta is formed by
amnion

chorion frundosum

amnion frundosum

allantois

allantois frundosum

A

B) Reason

Fetal componentof Placenta: Cytotrophoblast Inner layer of chorionic villi (the frundosum). Cytotrophoblast makes Cells.Syncytiotrophoblast Outer layer of chorionic villi; secretes hCG (structurally similar to LH; stimulates corpus luteum to secrete progesterone during first trimester).

How well did you know this?
1
Not at all
2
3
4
5
Perfectly
275
Q

Pregnant lady advised to eat green vegetables, because it contains
B12

B5

B6

B1

folate

A

E) Reason

FUNCTION: Converted to tetrahydrofolate (THF), a coenzyme for 1-carbon transfer/methylation reactions.Important for the synthesis of nitrogenous bases in DNA and RNA.Found in leafy green vegetables. Absorbed in jejunum. Folate from green vegetables and foliage.

How well did you know this?
1
Not at all
2
3
4
5
Perfectly
276
Q

Patient with raised bilirubin, direct is 9 and indirect bilurubin is 1, cause
Criglar najjar

hepatitis

HCC

bile deficiency

pancreatic enzymes deficiency

A

B) Reason

Conjugated or the direct bilurubin is raised in hepatitis in which conjugating system of enzymes is intact while unconjugated or the indirect bilurubin is increased in enzyme defects.

How well did you know this?
1
Not at all
2
3
4
5
Perfectly
277
Q

Olfactory pathway is/has
direct pathway

trochlear pathway

mitral cells to cortex

olfactory cells to cortex

mucosal cells to cortex

A

C) Reason

• The respiratory area is lined by pseudostratified, ciliated columnar epithelium. Goblet cells are present as well. The olfactory area is in the posterosuperior area and contains bipolar neurons and mitral cells. Olfactory neurons are constantly replenished.

How well did you know this?
1
Not at all
2
3
4
5
Perfectly
278
Q

Patient took antibiotics for 2 months post-Op develops dirrohea, what would be changes seen on microscopy
Villous atrophy

hyperkeratosis

Lymphocytic infiltrate

pseudomembrane

edematous villi

A

D) Reason

C. difficile: Produces 2 toxins. Toxin A, enterotoxin, binds to the brush border of the gut. Toxin B, cytotoxin, causes cytoskeletal disruption via actin depolymerization causing pseudomembranous colitis leading to diarrhea. Often 2° to antibiotic use, especially clindamycin or ampicillin

How well did you know this?
1
Not at all
2
3
4
5
Perfectly
279
Q

CHF can best be diagnosed by which of following
ANP

Chest X-ray

BNP

ECG

angiography

A

C) Reason

B-type (brain) natriuretic peptide(BNP): Released from ventricular myocytes in response to tension. Similar physiologic action to ANP, with longer half-life. BNP blood test used for diagnosing heart failure (very good negative predictive value). Available in recombinant form (nesiritide) for treatment of heart failure.

How well did you know this?
1
Not at all
2
3
4
5
Perfectly
280
Q

kidneys get how much %age of C/O?
25

27

29

30

45

A

A) Reason

The blood flow through the renal system circulation (25% of cardiac output) can be subdivided into the hepatic arterial flow (7% of cardiac output) and the mesenteric-portal circulation (18% of cardiac output

How well did you know this?
1
Not at all
2
3
4
5
Perfectly
281
Q

what struture is injured superficial to flexor retinaculum in wrist injury?
ulnar nerve & artey

ulnar nerve only

ulnar artery only

radial nerve

median nerve

A

A) Reason

Ulnar nerve and artery pass over the Flexor Retinaculum, and this bridge and the carpal bones form the carpaltunnel for the passage of the median nerve and the longflexor tendons of the fingers

How well did you know this?
1
Not at all
2
3
4
5
Perfectly
282
Q

single best drug for mild CHF
ACE Inhibitor

beta blockers

nephedipine

verapamil

furosemide

A

A) Reason

Treatment of CHF: ACE inhibitors, β-blockers (except in acute decompensated HF), angiotensin II receptor blockers, and spironolactone dec. mortality. Thiazide or loop diuretics are used mainly for symptomatic relief. Hydralazine with nitrate therapy improves both symptoms and mortality in select patients.

How well did you know this?
1
Not at all
2
3
4
5
Perfectly
283
Q

Which Structure is of same size in new born & adults?
retina

heart valves

uncus

middle ear

inner ear

A

E) Reason

Inner ear size remains same in child and adult.

How well did you know this?
1
Not at all
2
3
4
5
Perfectly
284
Q

HLA matching is important because it prevents
anaphylactic

urticaria

graft vs host reaction

arthus reaction

lupus reaction

A

C) Reason

HLA matching is necessary to prevent the transplant rejection reactions. Host antibodies against donor HLA antigens and leukocytes can cause type 2 HSR.

How well did you know this?
1
Not at all
2
3
4
5
Perfectly
285
Q

flexor reflex is carried out by which receptors
tactile

nociceptors

free nerve endings

meisssner

pacinian

A

B) Reason

Flexor withdrawal reflex■ is polysynaptic.■ results in flexion on the ipsilateral side and extension on the contralateral side.Somatosensory and pain afferent fibers elicit withdrawal of the stimulated body part from the noxious stimulus.

How well did you know this?
1
Not at all
2
3
4
5
Perfectly
286
Q

Venous supply of heart is through
left cardiac vein

coronary sinus

right cardiac vein

coronary hilum

SVC

A

B) Reason

• Coronary sinus The coronary sinus is the main vein of the coronary circulation; it lies in the posterior coronary sulcus. It drains to an opening in the right atrium. It develops from the left sinus venosus.

How well did you know this?
1
Not at all
2
3
4
5
Perfectly
287
Q

Patient ECG shows 3:1 ratio of P to QRS complexes, Diagnosis.?
3rd degree block

1st degree block

mobitz type 2

mobitz type 2

atrial fibrillation

A

D) Reason

Mobitz type II: Dropped beats that are not preceded by a change in the length of the PR interval (as in type I). It is often found as 2:1 block, where there are 2 or more P waves to 1 QRS response. May progress to 3rd-degree block. Often treated with pacemaker.

How well did you know this?
1
Not at all
2
3
4
5
Perfectly
288
Q

Virulent strain of staph are identified by
coagulase test

gram staining

ZN staining

lipase

Hemolytic reaction

A

A) Reason

The enzyme is coagulase.Coagulase, by clotting plasma, serves to wall off the infectedsite, thereby retarding the migration of neutrophils into the site. It is positive in virulent strains of Staphylococci.

How well did you know this?
1
Not at all
2
3
4
5
Perfectly
289
Q

How the patient of HIV is monitored
PCR

Elisa

CD4 count

CD8 count

Western blot

A

C) Reason

Also use viral load to monitor effect of drug therapy.AIDS diagnosis ≤ 200 CD4+ cells mm3(normal: 500–1500 cells/mm3).HIV-positive with AIDS-defining condition (e.g., Pneumocystis pneumonia) or CD4 percentage < 14%.

How well did you know this?
1
Not at all
2
3
4
5
Perfectly
290
Q

why people fear of hospitals
rude behavior

uncooperative staff

fear of microbes

fear of infection

fear of being handicap

A

E) Reason

people fear hospitals because they think they will become handicapped and dependent on the medications.

How well did you know this?
1
Not at all
2
3
4
5
Perfectly
291
Q

Structure close to crus cerebri
homocullus

uncus

striatum

substantia nigra

globus pallidus

A

D) Reason

The midbrain comprises two lateral halves called the cerebral peduncles; each of these is divided into an anterior part, the crus cerebri; and a posterior part, the tegmentum, by a pigmented band of gray matter, the substantia nigra.

How well did you know this?
1
Not at all
2
3
4
5
Perfectly
292
Q

V/Q is seen increased in a pt. reason?
pulmonary embolism

sleep

coma

ARDS

exercise

A

A) Reason

In pulmonary embolism V/Q become infinity.

How well did you know this?
1
Not at all
2
3
4
5
Perfectly
293
Q

left cercumflex artery occlusion causes infarct of
left atrium

posterior surface

right ventricle

left atrium & ventricle

left ventricle only

A

E) Reason

Left circumflex coronary artery (LCX)— a branch of LCA supplies lateral and posterior walls of left ventricle

How well did you know this?
1
Not at all
2
3
4
5
Perfectly
294
Q

A runner iinjured his foot dueing practice and now he has pain when he tries to stand on his toes.This is due to
plantaris tendon

achilles tendon

posterior retinaculum

dorsal interossi

plantaris muscle

A

A) Reason

Plantar flexion (toes pointing downward): Gastrocnemius, soleus, plantaris, peroneus longus, peroneus brevis,tibialis posterior, flexor digitorum longus, planteris tendon and flexor hallucis longus muscles.

How well did you know this?
1
Not at all
2
3
4
5
Perfectly
295
Q

A multigravid female has C/O something coming out of vagina.Upon examination, cervix is coming out of external os.This is caused by damage of
vaginal vault

utrosacral ligament

uterine ligament

cervical ligament

pelvic diaphragm

A

B) Reason

PROLAPSE OF THE UTERUS:The levatores ani muscles and the transverse cervical,pubocervical, uterosacral and sacrocervical ligaments are of verygreat importance in supporting and positioning the uterus. Damage to these structures during childbirthor as a result of general poor body muscular tone mayresult in downward displacement of the uterus, cal led uterine prolapse

How well did you know this?
1
Not at all
2
3
4
5
Perfectly
296
Q

trypsinogen is activated by
enterokinase

CCK

bile

lipase

secretin

A

A) Reason

Trypsinogen: is Converted to trypsin by enterokinase/enteropeptidase, a brush-border enzyme on the duodenal and jejunal mucosa

How well did you know this?
1
Not at all
2
3
4
5
Perfectly
297
Q

centrilobular congestion is due to
fatty liver

fatty hepatosis

cirrhosis

nutmeg liver

HCC

A

D) Reason

Nutmeg liver Due to backup of blood into liver. Commonly caused by right-sided heart failure and Budd-Chiari syndrome. The liver appears mottled like a nutmeg. If the condition persists, centrilobular congestion and necrosis can result in cardiac cirrhosis.

How well did you know this?
1
Not at all
2
3
4
5
Perfectly
298
Q

Bilateral loss of pain and temperature at T4-T8, lesion is..?
Hemisection

complete section

central spinal lesion

disc prolapse

pulposus rupture

A

C) Reason

Syringomyelia: Cystic cavity (syrinx) within the spinal cord A(if central canal leading to hydromyelia). Crossing anterior spinal commissural fibers are typically damaged first. Results in a “cape-like,” bilateral loss of pain and temperature sensation in upper extremities (fine touch sensation is preserved).

How well did you know this?
1
Not at all
2
3
4
5
Perfectly
299
Q

Which vein is formed by the union of subclavian vein with internal jigular vein?
bracheocephalic trunk

jugular vein

thoracic duct

sterna chylli

left thoracic duct

A

A) Reason

Subclavian veing alongwith internal jugulat vein forms the brachiocephalic trunk.

How well did you know this?
1
Not at all
2
3
4
5
Perfectly
300
Q

Pancreas is affected by secretin by?
HCl inhibition

thick secretion

inc. Cl secretion
dec. HCO3 secretion
inc. HCO3 secretion

A

E) Reason

Secretin S cells (duodenum):Functions: Increases pancreatic HCO3 secretion gastric acid secretionbile secretion.

How well did you know this?
1
Not at all
2
3
4
5
Perfectly
301
Q

a lady with dec. IgA, IgD, IgM and chronic dirrohea as well as recurrent cough, Diagnosis..?
IgG deficiency

IgE deficiency

IgM deficiency

IgD deficiency

IgA deficiency

A

E) Reason

Selective IgAdeficiency:Unknown. Most common 1° immunodeficiency.Majority Asymptomatic. Can see Airway and GI infections, Autoimmune disease, Atopy, Anaphylaxis to IgA-containing products.

How well did you know this?
1
Not at all
2
3
4
5
Perfectly
302
Q

true about paraneoplastic syndromes is
large cell CA

squamous cell CA

adeno CA

small cell CA lung

NeuroEndocrine tumor

A

D) Reason

Small Cell CA: Undifferentiated , very aggressive.Paraneoplastic Syndrome: May produce ACTH, ADH, or Antibodies against presynaptic Ca2+ channels (Lambert-Eaton myasthenic syndrome). Amplification of myconcogenes common. Inoperable; treat with chemotherapy

How well did you know this?
1
Not at all
2
3
4
5
Perfectly
303
Q

oxytocin is stimulated& released by
adrenaline

Dopamine

LH

prolactin

suckling

A

E) Reason

Lactation After labor, the dec. in progesterone and estrogen disinhibits lactation. Suckling is required to maintain milk production, since inc. nerve stimulation inc. oxytocin and prolactin.

How well did you know this?
1
Not at all
2
3
4
5
Perfectly
304
Q

L1 vertebra fractured, injury to which structure
conus medularis

filum terminale

sacral plexus

lumber plexus

S1

A

A) Reason

The spinal cord is tapered below to formthe conus medullaris (L1-L2). The filum terminale is aprolongation of the pia mater that extends from the conus tothe back of the coccyx

How well did you know this?
1
Not at all
2
3
4
5
Perfectly
305
Q

It tells distribution of value in a data
confidence interval

frequency distribution

Hazards ratio

p-value

none of these

A

B) Reason

Distribution of frequency is the number of values repetition that a specified quantity appears in the given data. Here the most frequently appeared quantity is known as mode.

How well did you know this?
1
Not at all
2
3
4
5
Perfectly
306
Q

what brings about stimulation of thirst
inc. Cl conc.

dec. Na conc.

renin directly

Angiotensin-II

Angiotensin-I

A

D) Reason

Functions of Angiotensin-II are: (1) It increases thirst and therefore water intake.(2) It causes vasoconstriction of the arterioles, thereby increasing TPR and arterial pressure.

How well did you know this?
1
Not at all
2
3
4
5
Perfectly
307
Q

what kind of receptors are Messner”s receptors ?
low frequency

high frequency

low adapting

free nerve endings

least distributed

A

A) Reason

Meissner corpuscle are low frequency receptors, Present in nonhairy skin sense the Velocity and are Rapidly adapting receptors.

How well did you know this?
1
Not at all
2
3
4
5
Perfectly
308
Q

Tumor marker for CA colon is
CEA

PPD

CA-19-9

RET

Rb

A

A) Reason

CA Colon: Screen patients > 50 years old with colonoscopy or stool occult blood test.“Apple core” lesion seen on barium enema x-ray.CEA tumor marker: good for monitoring recurrence, not useful for screening.

How well did you know this?
1
Not at all
2
3
4
5
Perfectly
309
Q

clot retraction is brought about via
lymphocytes

Basophils

platelets

RBCs

leukocytosis

A

C) Reason

Platelet (thrombocyte) Involved in 1° hemostasis. Small cytoplasmic fragment derived from megakaryocytes. Life span of 8–10 days. When activated by endothelial injury, aggregates with other platelets and interacts with fibrinogen to form platelet plug and clot retraction.

How well did you know this?
1
Not at all
2
3
4
5
Perfectly
310
Q

movement of larynx while swallowing
downward

upward

lateral

anterior

pushed back

A

B) Reason

The main part of the larynx is thus elevated to the posterior surface of the epiglottis, and the entrance into the larynx is now closed.

How well did you know this?
1
Not at all
2
3
4
5
Perfectly
311
Q

What kind of lesion results in pure sensory loss due to damage to nerve when it enters spinal cord
tabes dorsalis

Hemisection

complete section

canal lesion

motor lesion

A

A) Reason

Tabes dorsalis Caused by 3° syphilis. Results from degeneration (demyelination) of dorsal columns and roots leading to impaired sensation and proprioception and progressive sensory ataxia (inability to sense or feel the legs causes poor coordination)

How well did you know this?
1
Not at all
2
3
4
5
Perfectly
312
Q

Which artery gives rise to Right bronchial artery
right post-intercostal artery

left post. intercostal artery

descending aorta

arch of aorta

axillary artery

A

A) Reason

The bronchi, connective tissue, and visceral pleura are supplied by the bronchial arteries, which are branches of the descending thoracic aorta. Right broncheal artery is the branch of the right posterior intercostal artery.

How well did you know this?
1
Not at all
2
3
4
5
Perfectly
313
Q

Characteristic finding of Transudate is
high protein

high cells ratio

high specific gravity

low specific gravity

high lipids

A

D) Reason

Transudate: Decreased protein content, blood cells as well as low specific gravity. Causes are CHF, nephrotic syndrome, or hepatic cirrhosis.

How well did you know this?
1
Not at all
2
3
4
5
Perfectly
314
Q

in temperature regulation how does post. hypothalamus plays role
piloerection

thirst

vasoconstriction

Vasodilation

shivering

A

E) Reason

Shivering is the most potent mechanism for increasing heat production. ■ Cold temperatures activate the shivering response, which is orchestrated by the posterior hypothalamus

How well did you know this?
1
Not at all
2
3
4
5
Perfectly
315
Q

while performing surgery rise in TPR is due to?
epinephrine

ACTH

Cortisol

GH

Dopamine

A

B) Reason

Surgery is one of the stressful conditions, and releases stress hormones via activation the hypothalamus secretion of ACTH.

How well did you know this?
1
Not at all
2
3
4
5
Perfectly
316
Q

Triglycerides in plasma are hydrolized by
pancreatic lipase

protease

hydroxylase

lipase

lipoprotein lipase

A

E) Reason

Lipoprotein lipase (LPL)—degradation of TG circulating in chylomicrons and VLDLs. Found on vascular endothelial surface.

How well did you know this?
1
Not at all
2
3
4
5
Perfectly
317
Q

Blood flow to kidneys is maintained by?
tubuloglumerlar feedback

renin Angiotensin system

local metabolism

sympathetic

parasympathetic

A

A) Reason

Tubuloglomerular feedback:, in which increased renal arterial pressure leads to increased delivery of fluid to the macula densa. The macula densa senses the increased load and causes constriction of the nearby afferent arteriole, increasing resistance to maintain constant blood flow.

How well did you know this?
1
Not at all
2
3
4
5
Perfectly
318
Q

slowest growing organism in artificial media
S aureus

Pseudomonas

M. tuberculosis

N. miningitis

M. bovis

A

C) Reason

Mycobacterium Tuberculosis is one of the slow growing organism on the culture media, that is the reason that its culture is waited till 6-8 weeks at least.

How well did you know this?
1
Not at all
2
3
4
5
Perfectly
319
Q

A patient with liomyoma, has dec. endometrial glands embedded in stroma
atrophy

dystrophy

hypertrophy

anaplasia

dysplasia

A

B) Reason

Leiomyoma (fibroid):Most common tumor in females. Often presents with multiple discrete tumors with decreased glandular tissue and increased stroma by glandular dystrophy. Increased incidence in blacks. Benign smooth muscle tumor; malignant transformation is rare.

How well did you know this?
1
Not at all
2
3
4
5
Perfectly
320
Q

outer layer of nerve is called
epimysium

epineuron

epineurium

perimysium

perinureum

A

C) Reason

The whole nerve trunk is ensheathed by epineurium, each fasciculus by perineurium, and each nerve fibre by a delicate endoneurium. The toughness of a nerve is due to its fibrous sheaths, otherwise the nerve tissue itself is very delicate and friable

How well did you know this?
1
Not at all
2
3
4
5
Perfectly
321
Q

effect of drug in old age
more dose required

can cause toxicity at therapeutic dose

less dose required

more safe window

more side effects

A

B) Reason

In old age, drug elimination system of body (liver as well as kidneys) become senile and their clearing capacity become less than in adulthood. That”s why therapeutic dose can cause toxicity.

How well did you know this?
1
Not at all
2
3
4
5
Perfectly
322
Q

Body is caught in bacterial infection. what will be 1st line of defence?
eosinophils

basophils

neutrophils

mucous membranes

skin

A

E) Reason

Innate defenses can be classified into three major categories: (1)physical barriers, such as intact skin and mucous membranes; (2) phagocytic cells,such as neutrophils, macrophages, and natural killer cells; and (3) proteins, such ascomplement, lysozyme, and interferon

How well did you know this?
1
Not at all
2
3
4
5
Perfectly
323
Q

Hypoxic pulmonary vasoconstriction is through
Na-H antiport

Ca channels

Cl chanel

ATP sensing K chanel

ATP sensing Na chanel

A

D) Reason

■ In the lungs, hypoxia causes vasoconstriction.■ This response is the opposite of that in other organs, where hypoxia causes vasodilation by opening hyoxia induced K chanel.■ Physiologically, this effect is important because local vasoconstriction redirects blood away from poorly ventilated, hypoxic regions of the lung and toward well-ventilated regions.

How well did you know this?
1
Not at all
2
3
4
5
Perfectly
324
Q

blood flow to muscle during exercise is regulated by
sympathetic

parasympathetic

O2 level

local metabolites

working level

A

D) Reason

Local metabolic control■ Blood flow in skeletal muscle exhibits autoregulation and active and reactive hyperemia.■ Demand for O2 in skeletal muscle varies with metabolic activity level, and blood flow is regulated to meet demand.■ During exercise, when demand is high, these local metabolic mechanisms are dominant.■ The local vasodilator substances are lactate, adenosine, and K+.

How well did you know this?
1
Not at all
2
3
4
5
Perfectly
325
Q

Vasodilation by sympathetic system is through
muscrinic

alpha 2 agonist

alpha 1 agonist

beta 2 agonists

beta 1 agonist

A

D) Reason

β2 Activation: Vasodilation, bronchodilation, inc. heart rate, inc. contractility, inc. lipolysis, inc. insulin release, dec. uterine tone (tocolysis), ciliary muscle relaxation, inc. queous humor production

How well did you know this?
1
Not at all
2
3
4
5
Perfectly
326
Q

patient with gram -ve sepsis and bleeding diathesis
DIC

ITP

TTP

HUS

hemophilia

A

A) Reason

Widespread activation of clotting leads to a deficiency in clotting factors, which creates a bleeding state.Causes: Sepsis (gram-negative), Trauma, Obstetric complications, acute Pancreatitis, Malignancy, Nephrotic syndrome,

How well did you know this?
1
Not at all
2
3
4
5
Perfectly
327
Q

hyperpolarization is result of?
efflux of Ca

eflux of K

efflux of Na

influx of Cl

efflux of Cl

A

B) Reason

Hyperpolarization makes the membrane potential more negative (the cell interior becomes more negative).

How well did you know this?
1
Not at all
2
3
4
5
Perfectly
328
Q

What triggers RBCs production in marrow?
GH

testosterone

erythropoitin

Aldosterone

Cortisol

A

C) Reason

Erythropoietin Released by interstitial cells in the peritubular capillary bed in response to hypoxia. It is induces the production of RBCs.

How well did you know this?
1
Not at all
2
3
4
5
Perfectly
329
Q

low voltage QRS complex are characteristic of
recent MI

old MI

atrial fibrillation

atrial flutter

ventricular fibrillation

A

B) Reason

Delayed complications are Dressler syndrome, HF, arrhythmias, true ventricular aneurysm (outward bulge during contraction, “dyskinesia”).

How well did you know this?
1
Not at all
2
3
4
5
Perfectly
330
Q

connections of pons
substantia nigra

striatum

globus pallidus

ipsilatral cerebellum

contralatral cerebellum

A

D) Reason

  • The telencephalon forms the cerebral hemispheres - The diencephalon forms 4 thalamic derivatives - The mesencephalon forms the midbrain - The metencephalon forms the pons and cerebellum that are closely related to wach other.- The myelencephalon forms the medulla
How well did you know this?
1
Not at all
2
3
4
5
Perfectly
331
Q

At site of injury which chemotactic factor appears 1st
IL2

IL3

IL4

IL8

TNF

A

D) Reason

IL-6: An endogenous pyrogen. Also secreted by Th2cells. Causes fever and stimulates production of acute-phase proteins.IL-8: Major chemotactic factor for neutrophils.

How well did you know this?
1
Not at all
2
3
4
5
Perfectly
332
Q

true hermaphrodite genotype
XO

XXY

XYY

XXXY

OY

A

B) Reason

True hermaphroditism (46,XX or 47,XXY):Also called ovotesticular disorder of sex development. Both ovary and testicular tissue present (ovotestis); ambiguous genitalia. Very rare.

How well did you know this?
1
Not at all
2
3
4
5
Perfectly
333
Q

commonest heart dominance circulation is
left side

codominance

ipsilatral

contralatral

right sided

A

E) Reason

Right-dominant circulation = 85% = PDA arises from RCA.Left-dominant circulation = 8% = PDA arises from LCX.Codominant circulation = 7% = PDA arises from both LCX and RCA.

How well did you know this?
1
Not at all
2
3
4
5
Perfectly
334
Q

In which phase of cell cycle, cell has double chromatid
prophase

metaphase

telophase

anaphase

interphase

A

C) Reason

Towards the end of the cell division, the telephase, chromatids reach to the corresponding opposite poles in form of double chromatids around which new membrane formed to give daughter cell nuclei.

How well did you know this?
1
Not at all
2
3
4
5
Perfectly
335
Q

Micronodular cirrhosis is best diagnosed by
serum Copper level

serum bilurubin

AST

serum cerulloplamin

ALT

A

D) Reason

Characterized by:Decreased Ceruloplasmin, Cirrhosis, Corneal deposits (Kayser-Fleischer rings). Copper accumulation, Carcinoma (hepatocellular)Hemolytic anemiaBasal ganglia degeneration (parkinsonian symptoms).

How well did you know this?
1
Not at all
2
3
4
5
Perfectly
336
Q

left renal vein is related to
posterior to aorta

anterior to aorta

lateral to aorta

superior to aorta

inferior to aorta

A

B) Reason

The renal vein emerges from the hilum of the kidney in front of the aorta and drains into theinferior vena cava.

How well did you know this?
1
Not at all
2
3
4
5
Perfectly
337
Q

which of following is the most common indication of mitral valve replacement
atheromatous

stenosis

rupture of papillary muscles

regurgitation

senile

A

C) Reason

Ventricular free wall rupture laeding to cardiac tamponade; papillary muscle rupture causes severe mitral Regurgitation; that is a one of the important indication of mitral valve replacement.

How well did you know this?
1
Not at all
2
3
4
5
Perfectly
338
Q

which drug at therapeutic dose causes tachycardia
captopril

Norepinephrine

isosorbide dinitrane

Methyldopa

hydralazine

A

C) Reason

TOXICITY Reflex tachycardia (treat with β-blockers), hypotension, flushing, headache, “Monday disease” in industrial exposure: development of tolerance for the vasodilating action during the work week and loss of tolerance over the weekend results in tachycardia, dizziness, and headache upon reexposure.

How well did you know this?
1
Not at all
2
3
4
5
Perfectly
339
Q

core temperature 99F but pt. is calm & cold, reason?
drowning

cardiogenic shock

hypovolumic shock

heat exhaustion

heat stroke

A

D) Reason

Heat exhaustion is caused by excessive sweating. As a result, blood volume and arterial blood pressure decrease laeding to cold clamy extrimities and syncope (fainting) occurs.

How well did you know this?
1
Not at all
2
3
4
5
Perfectly
340
Q

Which reaction occur in mitochondria
lipolysis

glycogesis

glycogenolysis

glycolysis

TCA

A

E) Reason

Reactions in Mitochondria: Fatty acid oxidation (β-oxidation), acetylCoA production, TCA cycle, oxidative phosphorylation.

How well did you know this?
1
Not at all
2
3
4
5
Perfectly
341
Q

Which reaction occur in mitochondria
lipolysis

glycogesis

glycogenolysis

glycolysis

TCA

A

E) Reason

Reactions in Mitochondria: Fatty acid oxidation (β-oxidation), acetylCoA production, TCA cycle, oxidative phosphorylation.

How well did you know this?
1
Not at all
2
3
4
5
Perfectly
342
Q

blood vessels are part of which ligament?
gastrospleenic

gastrodudenal

gastrohepatic

quadrate ligament

round ligament

A

A) Reason

Gastrosplenic Ligament: It connects Greater curvature and spleen. It contains Short gastrics, left gastroepiploic vesselsIt separates greater and lesser sacs on the left side.

How well did you know this?
1
Not at all
2
3
4
5
Perfectly
343
Q

initial sign of resuscitation after compensated shock is?
dec. BP

inc. pulse rate
dec. pulse rate
inc. BP

cold skin

A

C) Reason

Hypovolumia (blood loss aftr hemmorhage) causes hypoglycemic shock (low BP and high pulse rate). Fluid resuscitation causes heart rate to decrease to normal and than increase BP towards normal value.

How well did you know this?
1
Not at all
2
3
4
5
Perfectly
344
Q

hypothalmo-hypophysial portal system is
adrenal cortex

adrenal medulla

hypothalamus

posterior pituitary

adenohypophysis

A

E) Reason

Anterior pituitary (adenohypophysis):Secretes FSH, LH, ACTH, TSH, prolactin, GH, melanotropin (MSH). Derived from oral ectoderm (Rathke pouch).

How well did you know this?
1
Not at all
2
3
4
5
Perfectly
345
Q

How hormonal therapy dec. tumor size
necrosis

hypotrophy

hypertrophy

atrophy

Apoptosis

A

E) Reason

Hormones act by inhibiting the growth factors and dec. cancer growth. Tamoxifen is the typical example, Tamoxifenmoxifen, a selective estrogen receptor modulator (see Chapter 40), blocks the binding ofestrogen to receptors of estrogen-sensitive cancer cells in breast tissue.

How well did you know this?
1
Not at all
2
3
4
5
Perfectly
346
Q

Neurovascular bundle of rib lies in
superior border

lateral border

posterior border

internal border

inferior border

A

E) Reason

• The costal groove is located along the inferior border of each rib (upper aspect of the intercostal space) and provides protection for the intercostal nerve, artery, and vein which are located in the groove. The vein is most superior and the nerve is inferior in the groove (VAN).

How well did you know this?
1
Not at all
2
3
4
5
Perfectly
347
Q

Sympathetic neurotransmitter
epinephrine

Norepinephrine

Dopamine

phenylephrine

Ach.

A

B) Reason

Release of sympathetic neurotransmitter norepinephrine from a sympathetic nerve ending is modulated by norepinephrine itself, acting on presynaptic α2-autoreceptors, angiotensin II, and other substances.

How well did you know this?
1
Not at all
2
3
4
5
Perfectly
348
Q

BCG acts via
CD8 delayed response

CD4 delayed HSR

type 2 HSR

type 1 HSR

type e HSR

A

B) Reason

Type IV: Multiple sclerosisGuillain-Barré syndromeGraft-versus-host diseasePPD (test for M. tuberculosis)and BCG.Contact dermatitis (e.g., poison ivy, nickel allergy).

How well did you know this?
1
Not at all
2
3
4
5
Perfectly
349
Q

A patient with loss of appetite, HBc-Ab reactive, IgM unreactive, HBsAg unreactive
chronic infection

acute infection

carrier state

past HBV exposure

heavy infection

A

D) Reason

Here the markers of HBV given in this scenario show that no recent infection (absent IgM) and furthermore presence of antibodies indicates a previous infection.

How well did you know this?
1
Not at all
2
3
4
5
Perfectly
350
Q

regarding drug receptors
lipoprotein

glycoprotein

glycolipids

protein in nature

lipoploysacchride

A

D) Reason

Proteins:1. Integral proteins■ are anchored to, and imbedded in, the cell membrane through hydrophobic interactions.■ may span the cell membrane.■ include ion channels, transport proteins, receptors, and guanosine 5′-triphosphate (GTP)–binding proteins (G proteins).

How well did you know this?
1
Not at all
2
3
4
5
Perfectly
351
Q

A male loses 8% blood after RTA in half hr. most likely site of loss?
arterioles

arteries

veins

heart

spleen

A

C) Reason

Veins have more distensibility and calibre than that of arteries and accumulate more blood than that of arteries. Hence blood loss is mainly from veins.

How well did you know this?
1
Not at all
2
3
4
5
Perfectly
352
Q

hormone responsible for inc. pancreatic secretion and intracellular Ca
lipase

enterokinase

gastrin

secretin

CCK

A

E) Reason

Cholecystokinin (CCK): secreted from I cells (duodenum, jejunum)Increases pancreatic secretionIncreases gallbladder contractionDecreases gastric emptyingIncreases sphincter of Oddi relaxation

How well did you know this?
1
Not at all
2
3
4
5
Perfectly
353
Q

A pt. is on steroid therapy for a long time. sudden extraction is ill advised. why?
withdrawl effect

dec. Cortisol
inc. Cortisol
inc. ACTH
dec. ACTH

A

E) Reason

TOXICITY Iatrogenic Cushing syndrome—buffalo hump, moon facies, truncal obesity, muscle wasting, thin skin, easy bruisability, osteoporosis (treat with bisphosphonates), adrenocortical atrophy, peptic ulcers, diabetes (if chronic).Adrenal insufficiency (dec. ACTHg when drug stopped abruptly after chronic use.

How well did you know this?
1
Not at all
2
3
4
5
Perfectly
354
Q

CHF pt has developed edema. mechanism?
dec. hydrostatic pressure

inc. hydrostatic pressure
sec. oncotic pressure
inc. oncotic pressure
dec. plasma

A

B) Reason

Causes of Edema by Increased capillary hydrostatic pressure: Arteriolar dilationVenous constrictionIncreased venous pressureHeart failureExtracellular volume expansionStanding (edema in the dependent limbs)

How well did you know this?
1
Not at all
2
3
4
5
Perfectly
355
Q

histological feature of PCT
less mucosa

more mucosa

less muscles

villi

microvilli

A

E) Reason

The lateral borders of adjacent cells are extensively interdigitated. These characteristics are typical of cells involved in active transport. The lumen of the PCT is frequently clouded by microvilli which do not preserve well during the histologic preparation process.

How well did you know this?
1
Not at all
2
3
4
5
Perfectly
356
Q

Which of following cell has variable life
RBC

osteoblast

osteoclast

macrophages

lymphocytes

A

C) Reason

Osteoclasts are derived from cells of blood monocyte lineage, which ultimately come from hematopoietic stem cells. They have variable life span depending body Ca and PO4 levels.

How well did you know this?
1
Not at all
2
3
4
5
Perfectly
357
Q

Refractory period of cardiac Myocytes
0.8 sec

  1. 7 sec
  2. 5 sec
  3. 3 sec
  4. 1 sec
A

D) Reason

Absolute refractory period■ is the period during which another action potential cannot be elicited, no matter how large the stimulus, eg: 0.3 sec in cardiac cycle.■ coincides with almost the entire duration of the action potential.

How well did you know this?
1
Not at all
2
3
4
5
Perfectly
358
Q

SA node is pacemaker, reason
higher rate of firing

superior location

intrinsic property

less myofibrils

more myofibrils

A

A) Reason

Sinoatrial (SA) node ■ is normally the pacemaker of the heart due to its fast rate and automaticity■ has an unstable resting potential.■ exhibits phase 4 depolarization, or automaticity

How well did you know this?
1
Not at all
2
3
4
5
Perfectly
359
Q

prosopagnosia is inability to
recoganize color

recoganize face

recoganize objects

recoganize shapes

recoganize location

A

B) Reason

Among the higher mental functions of the cortex, the cerebrum, lesion manifestations are agraphesthesia (inability to recognize the shape of objects) and prosopagnosia is inability to recognize the face.

How well did you know this?
1
Not at all
2
3
4
5
Perfectly
360
Q

Regarding diabetes insipidus truth is?
dec. urine vol.

dec. serum Na
inc. serun K
inc. urinary Osmolarity
dec. urine Osmolarity

A

E) Reason

Diabetes insipidus Characterized by intense thirst and polyuria with inability to concentrate urine (decreased urine Osmolarity) due to lack of ADH. Has a central or nephrogenic cause.

How well did you know this?
1
Not at all
2
3
4
5
Perfectly
361
Q

A young lady took 10 tablets of paracetamol, what changes on liver on 3rd day
fibrosis

extensive hepatic necrosis

periportal fibrosis

periportal congestion

sinus congestion

A

B) Reason

Findings: mitochondrial abnormalities, extensive hepatic necrosis, fatty liver (microvascular fatty change), hypoglycemia, vomiting, hepatomegaly, coma.

How well did you know this?
1
Not at all
2
3
4
5
Perfectly
362
Q

A patient has chronic H/O gastric ulcer.Now he presented to SEW with severe epigastric pain and hypostension.Surgeon suspects gastric perfortion.Pain is carried in this situation by
lumber plexus

lumber splanchnic

great splanchnic

thoracic splanchnic

vagus

A

C) Reason

Great splanchnic nerve supply to the abdomenal viscera. Foregut and midgut (thoracic splanchnic nerves: T5-T12) and to the hindgut and pelvic viscera (lumbar splanchnic nerves: L1-L2) synapse in collateral ganglia.

How well did you know this?
1
Not at all
2
3
4
5
Perfectly
363
Q

block of artery supplying medulla will cause
deltoid paralysis

tongue paralysis

serratus anterior

serratus posterior

latismus dorsi

A

B) Reason

Caudal medulla—origin of hypoglossal nerve.Lesion causes Ipsilatral hypoglossal dysfunction (tongue deviates ipsilaterally

How well did you know this?
1
Not at all
2
3
4
5
Perfectly
364
Q

A patient can”t feel pain or touch sensation over third and fourth fingers.Which nerve is damaged in this case?
cutaneous nerve

axillary nerve

radial nerve

ulnar nerve

median nerve

A

D) Reason

Ulnar (C8-T1) Fracture of medial epicondyle of humerus “funny bone”.“Ulnar claw” on digit extension and loss of sensation of 3rd and 4th fingers.

How well did you know this?
1
Not at all
2
3
4
5
Perfectly
365
Q

A patient sustained injury to chest with a knife.How.many layers of chest wall will be damaged?
2

4

8

10

14

A

C) Reason

layers to be penetrted are skin, subcutaneous tissue, fat, muscle layers (2 layers), pleural ligament, pleural layers (2 layers) to lungs.

How well did you know this?
1
Not at all
2
3
4
5
Perfectly
366
Q

reason of decline in PaO2?
high altitude

deap sea

low drowning

sea level

Hypoxia

A

A) Reason

Adaptation to high altitude (Table 4.9) Alveolar Po2 is decreased at high altitude because the barometric pressure is decreased. As a result, arterial Po2 is also decreased (hypoxemia).

How well did you know this?
1
Not at all
2
3
4
5
Perfectly
367
Q

true about ABO blood group antigen
autosomal dominant

autosomal recessive

X-linked recessive

X-linked dominant

Y-linked

A

B) Reason

Autosomal recessive Traits:Albinism, ARPKD (formerly known as infantile polycystic kidney disease), cystic fibrosis, glycogen storage diseases, ABO blood group antigen, hemochromatosis, Kartagener syndrome.

How well did you know this?
1
Not at all
2
3
4
5
Perfectly
368
Q

which factor dec. in pregnancy
antithrombin 3

anti-trypsin

heparin

thrombin

fibrin

A

A) Reason

Pregnancy is an hypercoagulative state due to loss of antithrombin 3. Acquired deficiency of the Anti-thrombin III is related to the loss of factor in urine.

How well did you know this?
1
Not at all
2
3
4
5
Perfectly
369
Q

which is same Marker in pancreatic cancer, colon cancer and colangitis
CA 19-9

C-myc

CA 19

CA 125

CEA

A

E) Reason

CEA: CarcinoEmbryonic Antigen. Very nonspecific but produced by ∼ 70% of colorectal and pancreatic cancers; also produced by gastric, breast, and medullary thyroid carcinomas.

How well did you know this?
1
Not at all
2
3
4
5
Perfectly
370
Q

left shift in Hb-O2 curve is result of?
dec. K

dec. Na
dec. O2

increased pH

dec. pH

A

D) Reason

An increased in all factors (including H+) causes a shift of the curve to the right.A decreased in all factors (including H+) causes a shift of the curve to the left.

How well did you know this?
1
Not at all
2
3
4
5
Perfectly
371
Q

sphincter of gall bladder and intestine are relaxed by
sympathetic

autonomic

parasympathetic

lumber plexus

ascral plexus

A

C) Reason

Parasympathetic system is the system of rest and digest. Its activation causes increased GI motility, relaxed GI sphincters and increased GI secretions.

How well did you know this?
1
Not at all
2
3
4
5
Perfectly
372
Q

1st approved gene therapy was introduced through
plasmid

insulin

mice

doli sheep

horse

A

A) Reason

Plasmid Contains a variety of genes for antibiotic resistance, enzymes, toxins. It was used in recombinant technology for the first time in history.

How well did you know this?
1
Not at all
2
3
4
5
Perfectly
373
Q

A patient suddenly lifted heavy weight and subsequently developed sciatica.On examination,ankle jerk is absent.This is caused by damage of
L4

L3

S1

L5

L2

A

C) Reason

Heavy weight lifting is most of the time associated with damage to the Lumbosacral spine and its roots. Loss of ankle jerk (root value S1) mean lesion at S1 level.

How well did you know this?
1
Not at all
2
3
4
5
Perfectly
374
Q

which is not enzyme of pancrease?
glactase

lipase

amylase

mucous

protease

A

D) Reason

The acinar cells of the exocrine pancreas make up most of its weight.Secretion (serous) are secreted by the pancreatic acini.a. Acinar cells■ produce a small volume of initial pancreatic secretion, which is mainly Na+ and Cl

How well did you know this?
1
Not at all
2
3
4
5
Perfectly
375
Q

AV node delay is shortened by
parasympathetic

Ach

lidocain

autonomic

sympathetic

A

E) Reason

sympathetic stimulation causes increased in heart rate and hence cardiac cycle is decreased and sympathetic activity causes increased firing of nodal fibers and shortens the AV nodal delay.

How well did you know this?
1
Not at all
2
3
4
5
Perfectly
376
Q

in hemorrhagic shock urine output is decreased. reason?
low heart rate

high heart rate

dec. oncotic pressure

low arterial pressure

high arterial pressure

A

D) Reason

In hemmorhage, plasma volume decreases laeding to decreased arterial pressure and hence decreased GFR and low urine output.

How well did you know this?
1
Not at all
2
3
4
5
Perfectly
377
Q

ALT and AST are raised in hepatitis by which mechanism
their increase synthesis

decreased absorption

released from bile accumulation

none of these

released from destruction of hepatocytes

A

E) Reason

ALT and AST are the enzymes of the liver cells. In inflamation of the liver cells membrane is damaged leading to release of these enzymes. Hence their level increased in serum.

How well did you know this?
1
Not at all
2
3
4
5
Perfectly
378
Q

MOst active transport of sodium takes place at?
jejunum

dudenum

ileum

cecum

Colon

A

A) Reason

■ Examples are Na+-glucose cotransport in the small intestine (jejunum) and renal early proximal tubule and Na+K/2Cl cotransport in the renal thick ascending limb.

How well did you know this?
1
Not at all
2
3
4
5
Perfectly
379
Q

A patient with previous Compylobacter infection now developed paraplegia, cause.?
UMNL

LMNL

NM junction disorder

flaccid paralysis

GBS

A

E) Reason

Campylobacter infection can result in reactive arthritis, primarily in patients with HLA-B27. Other extraintestinal complications, including erythema nodosum and GuillainBarré syndrome, a flaccid paralysis caused by immunologically mediated inflammation of peripheral nerves, are not HLA-linked.

How well did you know this?
1
Not at all
2
3
4
5
Perfectly
380
Q

Which of the following structure is related medially and posteriorly to parotid and might be damaged during parotidectomy?
azygus vein

hemi azygus vein

internal jugular

carotid sheath

external jugular

A

D) Reason

• Posterolateral relation of the parotid gland are: The carotid sheath and its contents

How well did you know this?
1
Not at all
2
3
4
5
Perfectly
381
Q

which bone make floor of cranial fossa
maxillary sinus

mndibular bone

cervical spine

greater wing of sphenoid

lesser wing of sphenoid

A

D) Reason

Sphenoid Complex bone composed of a central body, and greater and lesser wings. Its greater wing forms the floor of the middle cranial fossa.

How well did you know this?
1
Not at all
2
3
4
5
Perfectly
382
Q

AB+ lady need blood postpartum. which group is apropriate?
O-

O+

AB+

B+

all of above

A

E) Reason

AB positive blood group is an universal accepter, moreover all groups given here all are compatible with AB positive blood group.

How well did you know this?
1
Not at all
2
3
4
5
Perfectly
383
Q

Which hormone causes smooth muscle contraction via IP3 ?
cortisol

Aldosterone

Prolactin

renin

ADH

A

E) Reason

Vasopressin [antidiuretic hormone (ADH)], IP3 and DAG■ is involved in the regulation of blood pressure in response to hemorrhage, but not in minute-to-minute regulation of normal blood pressure.■ Atrial receptors respond to a decrease in blood volume (or blood pressure) and cause the release of vasopressin from the posterior pituitary.■ Vasopressin has two effects that tend to increase blood pressure toward normal.

How well did you know this?
1
Not at all
2
3
4
5
Perfectly
384
Q

right hemiplegia with left sided mouth deviation, lesion is at..?
crona radiata

Midbrain

medulla

pons

internal capsule

A

E) Reason

internal capsule lesion: Contralateral hemiparesis/Crossed hemiplegia (contralatral mouth deviation)Common location of lacunar infarcts, 2° to unmanaged hypertension.

How well did you know this?
1
Not at all
2
3
4
5
Perfectly
385
Q

taste sensation from tractus solitarius goes to cortex via
hypothalamus

pons

thalamus

Midbrain

medulla

A

C) Reason

Solitarius Visceral Sensory information (e.g., taste, baroreceptors, gut distention). Sensations are Transmitted through thalamus to the cortex.

How well did you know this?
1
Not at all
2
3
4
5
Perfectly
386
Q

inhibitor of prostaglandins are
GH

insulin

corticosteroids

glucagon

naltrisone

A

C) Reason

orticosteroids cause neutrophilia, but eosinopenia and lymphopenia. Corticosteroids decrease the formation of prostagladins and decrease the activation of neutrophil adhesion molecules, impairing migration out of the vasculature to sites of inflammation.

How well did you know this?
1
Not at all
2
3
4
5
Perfectly
387
Q

All or none action potential is feature of
dendrites

axon

astrocytes

oligodendrocyte

Schwann cells

A

B) Reason

During the action potential of the neurons, it follows the all or none principle. All the fibers are excited or no one is excited during the action potential.

How well did you know this?
1
Not at all
2
3
4
5
Perfectly
388
Q

lidocaine is given in treatment of
WPS syndrome

ventricular tachycardia

atrial fibrillation

atrial flutter

ventriclar fibrillation

A

B) Reason

CLINICAL USE of Lidocain : Acute ventricular arrhythmias (especially postMI), digitalis-induced arrhythmias especially post-MI.

How well did you know this?
1
Not at all
2
3
4
5
Perfectly
389
Q

A female with recurrent UTI, Pseudomonas on culture, drug of choice
Ciprofloxacin

metronidazole

ampicillin

gentamicin

clavulunate

A

A) Reason

Treatment: aminoglycoside plus extended spectrum penicillin (e.g., piperacillin, ticarcillin, cefepime, imipenem, meropenem) and quinolones (UTI and pylonephritis).

How well did you know this?
1
Not at all
2
3
4
5
Perfectly
390
Q

Hyperextension of tibia anteriorly over knee joint will damage
patella

anterior cruciate ligament

lateral ligament

tibial condyl

fibular head

A

B) Reason

The anterior cruciate ligament is attached below tothe anterior intercondylar area of the tibia, most commonly injured in excessive anterior movementa of the knee joint.

How well did you know this?
1
Not at all
2
3
4
5
Perfectly
391
Q

squamocolumnar junction of esophagus is associated with
achlasia

squamous metaplasia

cuboidal Metaplasia

Barrett esophagus

CA esophagus

A

D) Reason

Barrett esophagus Glandular metaplasia—replacement of nonkeratinized (stratified) squamous epithelium with intestinal epithelium (nonciliated columnar with goblets cells) in the distal esophagus at squamocolumnar junction. Due to chronic acid reflux (GERD).

How well did you know this?
1
Not at all
2
3
4
5
Perfectly
392
Q

myelination of periphral nervous system
astrocyte

dendrites

axons

schwann cells

oligodendrite

A

D) Reason

Schwarm cells are glial cells that make myelin for PNS axons. Unlike oligodendrocytes, which make C S myelin, individual Schwann cells myelinate only a small part of a single axon.

How well did you know this?
1
Not at all
2
3
4
5
Perfectly
393
Q

Complete loss of arm abduction rsults from damage to
supraspinatus

axillary

supraspinatus and axillary

radial

median

A

C) Reason

Abduction of arm is the combined function if the Supraspinatus muscle and deltoid, supplied by supraspinatus and axillary nerve respectively.

How well did you know this?
1
Not at all
2
3
4
5
Perfectly
394
Q

serum Ca and phosphate are regulated by?
PTH

renal function

liver function

lung function

bone function

A

B) Reason

Level of serum calcium and serum Phosphate are regulated by renal function. Ca is decreased and phosphate is increased in impaired renal functions.

How well did you know this?
1
Not at all
2
3
4
5
Perfectly
395
Q

Quiet inspiration is carried out by
diaphragm

pleura

intercostal

thoracic splanchnic

Trapezius

A

A) Reason

The phrenic nerve (C3–5).Action• Muscle of inspiration: On contraction, the diaphragmpulls its central tendon down and increases the vertical diameter of the thorax

How well did you know this?
1
Not at all
2
3
4
5
Perfectly
396
Q

true about vibrio cholera
survive in alkaline

survive in acidic

susceptible to alkaline

live in high temperature

live in high pH

A

A) Reason

Vibrio cholerae Produces profuse rice-water diarrhea via enterotoxin that permanently activates Gs, inc. cAMP. Comma shaped, oxidase positive, grows in alkaline media. Endemic to developing countries. Prompt oral rehydration is necessary

How well did you know this?
1
Not at all
2
3
4
5
Perfectly
397
Q

Injury to the phrenic nerve at its root will lead to loss of
respiratory reflex

gastric reflex

sneezing reflex

cough reflex

pharyngeal reflex

A

A) Reason

Diaphragm is innervated by C3, 4, and 5 (phrenic nerve). Lesion of the nerve leads to respiratory distress and loss of respiratory reflex.

How well did you know this?
1
Not at all
2
3
4
5
Perfectly
398
Q

Thrombocytopenia is side effect of
loop diuretics

osmotic diuretics

thiazide

spironolactone

amiloride

A

C) Reason

TOXICITY Hypokalemic metabolic alkalosis, Thrombocytopenia, hyponatremia, hyperGlycemia, hyperLipidemia, hyperUricemia, and hyperCalcemia. Sulfa allergy

How well did you know this?
1
Not at all
2
3
4
5
Perfectly
399
Q

true about SLE
type 1 HSR

type 3 HSR

tyep 2 HSR

type 4 HSR

Delayed response

A

B) Reason

Type III:Polyarteritis nodosaPoststreptococcal glomerulonephritisSerum sickness, SLEArthus reaction (e.g., swelling and inflammation following tetanus vaccine)

How well did you know this?
1
Not at all
2
3
4
5
Perfectly
400
Q

a pt with pacemaker comes for ecg, finding shows absent P wave, normal qrs complex & T wave. location of pacemaker?
bundle of His

SA node

AV node

atria

ventricle

A

C) Reason

Pacemaker action:Occurs in the SA and AV nodes. Key differences from the ventricular action potential. Normally in SA node. When in AV node, there is no P wave, rest is normal.

How well did you know this?
1
Not at all
2
3
4
5
Perfectly
401
Q

fatty acids combines with glycerol to form fat in..?
RER

SER

golgi complex

nucleus

cell membrane

A

C) Reason

In golgi apparatus, Glyceraldehyde-3-phosphate to glycerol-3-phosphate (combines with fatty acids to make triglycerides

How well did you know this?
1
Not at all
2
3
4
5
Perfectly
402
Q

A boy had injury to knee, 8 month later he had hematoma and bone trabeculi, Dx..?
dysplasia

metaplasia

anaplasia

neoplasia

dystrophy

A

B) Reason

Metaplasia mean transformation of the one type of cells into another type of cells. Here in this scenario, injury caused soft tissue to develop bony (metaplstic) tissue.

How well did you know this?
1
Not at all
2
3
4
5
Perfectly
403
Q

in mitral stenosis which jvp is not found?
a wave

x wave

y desent

d wave

V wave

A

E) Reason

v wave—inc. right atrial pressure due to filling against closed tricuspid valve. It is absent in mitral stenosis.

How well did you know this?
1
Not at all
2
3
4
5
Perfectly
404
Q

cells responsible for maintenance and integrity of bone
fibroblasts

osteiod

osteocytes

osteoclasts

osteoblasts

A

E) Reason

Osteoblasts are specialized to synthesize and secrete the components of osteoid, type I collagen, and GA Gs. The unique structure of the osteoid, in particular the ordering of the collagen, promotes the formation of a crystalline structure.

How well did you know this?
1
Not at all
2
3
4
5
Perfectly
405
Q

Which structure will be damaged in case of penetrating injury to posterior triangle of the neck?
deltoid

trapezius

supraspinatus

teres major

seratus posterior

A

B) Reason

The posterior triangle is bounded posteriorly by the trapezius muscle, anteriorly by the sternocleidomastoid muscle,and inferiorly by the clavicle

How well did you know this?
1
Not at all
2
3
4
5
Perfectly
406
Q

Cause of amenorhea in married female with pregnancy test negative
ectopic pregnancy

molar pregnancy

no pregnancy

twin pregnancy

anovulation

A

E) Reason

Most common causes of anovulationPregnancy, polycystic ovarian syndrome, obesity, HPO axis abnormalities, premature ovarian failure, Anovulatory cycles, hyperprolactinemia, thyroid disorders, eating disorders, female athletes, Cushing syndrome, adrenal insufficiency.

How well did you know this?
1
Not at all
2
3
4
5
Perfectly
407
Q

When fibrinogen changes to fibrin what will be seen in blood
lipase

fibrinolase

Fibrinopeptidase A

Fibrinopeptidase B

Fibrinopeptide

A

C) Reason

Fibrinogen: It is Coagulation factor; converts to fibrin by FibrinopeptidaseA, promotes endothelial repair; correlates with ESR

How well did you know this?
1
Not at all
2
3
4
5
Perfectly
408
Q

A middle age man is suspected of lymphoma, it is diagnosed by
Electrophoresis

bone marrow biopsy

CBC

Peripheral smear

immuno-histochemistry

A

E) Reason

Lymphoma Discrete tumor masses arising from lymph nodes. Presentations often blur definitions. Diagnosed witn immunohistochemistry.

How well did you know this?
1
Not at all
2
3
4
5
Perfectly
409
Q

Patient has no cough, but diffuse fibrosis on chest X-ray, cause
silicon

hydrocarbon

asbestos

inorganic dust

berrylium

A

D) Reason

How well did you know this?
1
Not at all
2
3
4
5
Perfectly
410
Q

Keeping types of joints in mind, wrist joint is a
hing type

sesamoid

fixed joint

condyloid

Ellipsoid

A

E) Reason

Wrist joint is an example of Ellipsoid type. Reference B.D.

How well did you know this?
1
Not at all
2
3
4
5
Perfectly
411
Q

A 70kg male will have a urinary output of?
0.5 L

1 L

2 L

3 L

1.5 L

A

E) Reason

Normal urine output is about 1ml/kg per hour. So, in a person of 70kg it would be, Urine output= 1ml x 70 x 24 = 1680ml, app = 1.5L.

How well did you know this?
1
Not at all
2
3
4
5
Perfectly
412
Q

IV drug abuser with cough not responding to ATT, bilateral infiltrates on CXR, cause..n
M. tuberculosis

P. aerogenosa

Candida

P. jirovecii

M. bovis

A

D) Reason

Pneumocystis jirovecii:Causes Pneumocystis pneumonia (PCP), a diffuse interstitial pneumonia. Yeast (originally classified as protozoan). Inhaled. Most infections are asymptomatic. Immunosuppression (e.g., AIDS) predisposes to disease. Diffuse, bilateral CXR appearance. Diagnosed by lung biopsy or lavage. Disc-shaped yeast forms on methenamine silver stain of lung tissue .

How well did you know this?
1
Not at all
2
3
4
5
Perfectly
413
Q

primordial germ cells are formed at
2nd month

8th day

2nd week

3rd week

4th week

A

E) Reason

Primordial germ cells arise from the lining cells in the wall of the yolk sac. At week 4, primordial germ cells migrate into the indifferent gonad.

How well did you know this?
1
Not at all
2
3
4
5
Perfectly
414
Q

temp maintenance of skeleto-motor system is via?
vasoconstriction

shivering

vasodilation

piloerction

sheltering

A

B) Reason

Shivering is the most potent mechanism for increasing heat production. ■ Cold temperatures activate the shivering response, which is orchestrated by the posterior hypothalamus.

How well did you know this?
1
Not at all
2
3
4
5
Perfectly
415
Q

loop diuretic cause what
no effect on medularry osmolarity

inc. reabsorption
inc. secretion
inc. medullary interstitial osmolarity
dec. Medullary Osmolarity

A

E) Reason

It causes decreased osmolarity.

How well did you know this?
1
Not at all
2
3
4
5
Perfectly
416
Q

which structure is responsible for keeping flexor tone of muscles?
fasiculus gracilis

fasiculus cuneatus

stratum

uncus

red nucleus

A

E) Reason

Rubrospinal tract■ originates in the red nucleus and projects to interneurons in the lateral spinal cord.■ Stimulation of the red nucleus produces stimulation of flexors and inhibition of extensors.

How well did you know this?
1
Not at all
2
3
4
5
Perfectly
417
Q

which has role in tissue remodeling
macrophages

metalloproteins

fibroelasts

collagenase

lipase

A

D) Reason

Collegenase is protein enzyme that functions to remodeling if the repaired tissues. Its defects leads to abnormal shape of the repaired tissues like abnormal scars.

How well did you know this?
1
Not at all
2
3
4
5
Perfectly
418
Q

There is a triangle in neck which is formed by mandible, sternocledomastoid and midline.This traingle is called
posterior

lateral

anterior

inferior

superior

A

C) Reason

The anterior triangle is bounded above by the body of themandible, posteriorly by the sternocleidomastoid muscle,and anteriorly by the midline

How well did you know this?
1
Not at all
2
3
4
5
Perfectly
419
Q

Epithelium of urinary bladder is dervied in embryological life from
urogenital sinus

urogenital ridge

urogenital diaphragm

urogenital membrane

urachus

A

A) Reason

In the 3rd week the yolk sac forms the allantois, which extends into the urogenital sinus. Later on, urogenital sinus forms the epithelium of the Urinary bladder.

How well did you know this?
1
Not at all
2
3
4
5
Perfectly
420
Q

brain gets what %age of total cardiac output?
5

7

10

15

20

A

D) Reason

These are the well perfused organs. 25% of the cardiac output goes to the liver; 4-5% (225 ml/min) to the heart; 15% to the brain; 20% to the kidneys; and 100% to the lungs.

How well did you know this?
1
Not at all
2
3
4
5
Perfectly
421
Q

a pt experiences conduction aphasia. site of lesion is?
Arcuate faciculus

gracilis

faciculus cuneatus

red nucleus

pons

A

A) Reason

Conduction Aphasia There is a large fiber bundle connecting areas 22, 39, and 40 with Broca area in the frontal lobe, known as the superior longitudinal fasciculus (or the arcuate fasciculus). A lesion affecting this fiber bundle results in a conduction aphasia.

How well did you know this?
1
Not at all
2
3
4
5
Perfectly
422
Q

frontal lobe tumor with optic tract affected will cause
Homonemous hemianopia

Heteronymous hemianopia

ipsilatral loss

contralatral loss

upper quadrant loss

A

A) Reason

Bitemporal hemianopia (pituitary lesion, chiasm) Left homonymous hemianopia(lesion of ipsilatral optic tract). Left upper quadrantic anopia (right temporal lesion, MCA)

How well did you know this?
1
Not at all
2
3
4
5
Perfectly
423
Q

what leads to osmotic diarrhea?
fructose

galactose

sucrose

dextran

lactose

A

E) Reason

Lactose intolerance results from the absence of brush border lactase and, thus, the inability to hydrolyze lactose to glucose and galactose for absorption. Nonabsorbed lactose and H2O remain in the lumen of the GI tract and cause osmotic diarrhea.

How well did you know this?
1
Not at all
2
3
4
5
Perfectly
424
Q

Pt. can”t diff. between red and green having red color blindness, condition is called
deutronopia

tritononopia

protanopia

gluconopia

rubronopia

A

C) Reason

Color blindness are named as Protonopia as red blindness, Deutronopia as green blindness and tritononopia is blue color blindness.

How well did you know this?
1
Not at all
2
3
4
5
Perfectly
425
Q

A lady with fasting sugar 98 and random 122, post meal is 190, Dx..?
diabetes

Impaired glucose tolerance

pre-diabetes

no diabetes

type 2 diabetes

A

B) Reason

Diagnosis: fasting sugar greater than 110mg/dl, random level greater than 126mg/dl or post glucose greater than 200mg/dl are diagnostic ojf diabetes mellitus.

How well did you know this?
1
Not at all
2
3
4
5
Perfectly
426
Q

Dermatome for cholecystitis pain
C5

C6

C6-C8

C4

C7-9

A

D) Reason

dermatome is same as of shoulder tip.

How well did you know this?
1
Not at all
2
3
4
5
Perfectly
427
Q

which carry impulses away from cell body
dendrites

axon

oligodendrite

astrocytes

schwann cells

A

B) Reason

Signal-relaying cells with dendrites (receive input), cell bodies, and axons (send output). Cell bodies and dendrites can be stained via the Nissl substance (stains RER).

How well did you know this?
1
Not at all
2
3
4
5
Perfectly
428
Q

fatty meal leading to bloating at night. work of which hormone?
GIP

CCK

VIP

secretin

gastrin

A

B) Reason

Can also lead to biliary colic—neurohormonal activation (e.g., by CCK after a fatty meal) triggers contraction of the gallbladder, forcing a stone into the cystic duct. May present with bloating and with or without pain (e.g., in diabetics).

How well did you know this?
1
Not at all
2
3
4
5
Perfectly
429
Q

after shock managment what will be corected first?
BP

temperature

Hb

leukocytes

Heart rate

A

E) Reason

In hypovolumic shock, Blood volume decreases that causes decreased BP and increased heart rate. As the fluid therapy is started, heart rate slow down to normal and then BP also goes up to normal.

How well did you know this?
1
Not at all
2
3
4
5
Perfectly
430
Q

A lady going to Saudi Arabia has clear fluid filled lesion in mouth, Dx..?
HSV1

HHV

HHV-8

HIV

HCV

A

A) Reason

HSV-1: Gingivostomatitis, keratoconjunctivitis, temporal lobe encephalitis (most common cause of sporadic encephalitis in the United States), herpes labialis. Latent in trigeminal ganglia.Transmitted by respiratory secretions, saliva.

How well did you know this?
1
Not at all
2
3
4
5
Perfectly
431
Q

which NSAID is contraindicated in CLD
Clopidogrel

aspirin

ampicillin

gentamycin

metronidazole

A

B) Reason

TOXICITY Gastric ulceration, tinnitus (CN VIII). Chronic use can lead to acute renal failure, interstitial nephritis (contraindicated in CRF), and upper GI bleeding. Risk of Reye syndrome in children treated with aspirin for viral infection.

How well did you know this?
1
Not at all
2
3
4
5
Perfectly
432
Q

In thrombus formation, 1st step is
clot retraction

platelets adhesion

clot formation

endothelial injury

tissue reaction

A

B) Reason

In thrombus formation, platelets adhesion takes place over the injured endothelium making platelet plug that is site for thrombus formation.

How well did you know this?
1
Not at all
2
3
4
5
Perfectly
433
Q

A female is declared infertile yet prolactin is increased. reason?
adenoma

hyperplasia

hypertrophy

macroprolactinoma

microprolectinoma

A

E) Reason

Functional tumor (microprolactinoma) presentation is based on the hormone produced (e.g., prolactinoma: amenorrhea, galactorrhea, low libido, infertility; somatotropic adenoma: acromegaly).

How well did you know this?
1
Not at all
2
3
4
5
Perfectly
434
Q

heart contractility is affected by all except?
Ach

epinephrine

Norepinephrine

Cortisol

GH

A

A) Reason

Sympathetic system stimulation or secretion of any stress hormone causes increased contratility of the heart . Ach. is a parasympathetic hormone.

How well did you know this?
1
Not at all
2
3
4
5
Perfectly
435
Q

Cost effective way of judging vit.A deficiency in community
clinical

diet

pharmacological

supplements

Screenig

A

B) Reason

It is easy to assess and manage the nuitrional deficiencies in the in diet rather to do it with clinical or the pharmacological interventions that would be expensive and complicated as well.

How well did you know this?
1
Not at all
2
3
4
5
Perfectly
436
Q

which is type 2 hypersensitivity reaction
SLE

Polyarteritis nodosa

serum sickness

arthus reaction

ITP

A

E) Reason

Type II: Autoimmune hemolytic anemiaPernicious anemiaIdiopathic thrombocytopenic purpuraErythroblastosis fetalisAcute hemolytic transfusion reactionsRheumatic feverGoodpasture syndromeBullous pemphigoidPemphigus vulgaris.

How well did you know this?
1
Not at all
2
3
4
5
Perfectly
437
Q

staging is defined as
extent of severity

extent of lesion

differentiation

anaplasia

prognosis

A

B) Reason

Staging of tumor: Degree of localization/spread based on site and size of 1° lesion, spread to regional lymph nodes, presence of metastases. Based on clinical (c) or pathology (p) findings. Example: cT3N1M0

How well did you know this?
1
Not at all
2
3
4
5
Perfectly
438
Q

Hepatic gulconeogenesis is done by
glucagon

insulin

GH

LH

Cortisol

A

A) Reason

FUNCTION of Glucagon are: Catabolic effects of glucagon:Increases Glycogenolysis, gluconeogenesis(in liver)Inc. Lipolysis and ketone production

How well did you know this?
1
Not at all
2
3
4
5
Perfectly
439
Q

A patient was injured in RTA and sustained fracture of hip joint.Now when he stands up,his pelvis and left leg sinks down.This is caused by a lesion of
rectus femoris

quadricep femoris

adductor longus

left maximus

Right minimus

A

E) Reason

If gluteus minimus then the pelviswill sink downward on the opposite, unsupportedside. The patient is then said to exhibit a positiveTrendelenburg’s sign.

How well did you know this?
1
Not at all
2
3
4
5
Perfectly
440
Q

parameter responsible for preload calculation?
stroke volume

heart rate

plasma flow

Left ventricle EDV

viscosity

A

D) Reason

Preload: Preload approximated by ventricular EDV; depends on venous tone and circulating blood volume.

How well did you know this?
1
Not at all
2
3
4
5
Perfectly
441
Q

Damage to which of the following will lead to loss of ankle jerk?
S1

L4

L5

S2

L1

A

A) Reason

Heavy weight lifting is most of the time associated with damage to the spinal nerve roots. Here ankle jerk (root value of S1) loss mean lesion is at S1 level.

How well did you know this?
1
Not at all
2
3
4
5
Perfectly
442
Q

reason of TPR
smooth muscle

sphincters

arterioles

vasomotor tone

capillaries

A

D) Reason

Total peripheral resistance: maintained by vasomotor tone.Changes in TPR leads to altered CO at a given RA pressure; however, mean systemic pressure (x-intercept) is unchanged.

How well did you know this?
1
Not at all
2
3
4
5
Perfectly
443
Q

Which of the following vessels starts at the level of ear lobule and ends at sternoclavicular joint?
external jugular vein

internal jugular vein

internal carotid

external carotid

common carotid

A

B) Reason

The internal jugular vein is a large vein that drains bloodfrom the brain, face, scalp, and neck . It starts as acontinuation of the sigmoid venous sinus and leaves theskull through the jugular foramen and ends at the medial border of clavicle by forming bracheocephalic vein.

How well did you know this?
1
Not at all
2
3
4
5
Perfectly
444
Q

fibrocartilage is present in
knee

elbow

Intervertebral disc

sutures

TM joint

A

C) Reason

Fibrocartilage has type I collagen in addition to type II, giving it greater resistance to being stretched (tensile strength), and is the type of cartilage found in intervertebral disks of the vertebral column and the menisci of the knee.

How well did you know this?
1
Not at all
2
3
4
5
Perfectly
445
Q

most common cause of Hyperbilirubinemia in neonates
hepatitis

hemolysis

maternal alcohlism

conjugated Hyperbilirubinemia

Gilbert syndrome

A

B) Reason

Most common cause of unconjugated /indirect hyperbilirubinemia are: Hemolytic, physiologic (newborns), Crigler-Najjar, Gilbert syndrome

How well did you know this?
1
Not at all
2
3
4
5
Perfectly
446
Q

Virulence of Staph aureus is determined by
protease

plasmin

coagulase

cell wall

capsule

A

C) Reason

Coagulase is an enzyme that causes plasma to clot byactivating prothrombin to form thrombin. It is the main virulent factor of staph. aureus. Thrombin then catalyzes the activation offibrinogen to form the fibrin clot.

How well did you know this?
1
Not at all
2
3
4
5
Perfectly
447
Q

which of following is Hypertonic solution
10% dextrose

  1. 9% NaCl
  2. 2% NaCl

5% Dextrose

0.1% NaCl

A

A) Reason

All the fluids given in the list are either hypotonic to the body (0.1, 0.2 percent NaCl) or Isotonic such as (0.9 percent NaCl or 5 percent dextrose) except 10 percent Dextrous that is Hypertonic.

How well did you know this?
1
Not at all
2
3
4
5
Perfectly
448
Q

Recently married female with lower abdominal pain. Urine gives pus cells, Dx..?
E. coli

Pseudomonas

staph. aureus

N. miningitis

Shagilla

A

A) Reason

E. coli is the leading cause of community-acquired urinary tract infections.These infections occur primarily in women; causing frequent burning Micturation and suprapubic tenderness.

How well did you know this?
1
Not at all
2
3
4
5
Perfectly
449
Q

what change will be seen in lung compliance if surface tension is decreased?
decrease

increase

shift to left

shift to right

become zero

A

B) Reason

Compliance—change in lung volume for a given change in pressure; maintained by surfactant decreased in pulmonary fibrosis, pneumonia, and pulmonary edema and increased surfactant; increased in emphysema, low or none surfactantand normal aging

How well did you know this?
1
Not at all
2
3
4
5
Perfectly
450
Q

what replicates itself
mitochondria

RER

SER

Golgi complex

nuclei

A

A) Reason

They synthesize adenosine triphosphate (ATP ), contain their own double-stranded circular DNA, and make some of their own proteins and replicate themselves by virtue of their own genetic material.

How well did you know this?
1
Not at all
2
3
4
5
Perfectly
451
Q

Patient presents with constipation, muscle weakness and short T-wave, Dx..?
hyperkalemia

hypokalemia

hypernatremia

atrial flutter

atrial fibrillation

A

B) Reason

T wave—ventricular repolarization. T-wave inversion may indicate recent MI. While short T wave in hypokalemia and tall T wave in hyperkalemia.

How well did you know this?
1
Not at all
2
3
4
5
Perfectly
452
Q

which connects laminae in vertebral column
Ligamentum teres

Ligamentum flavum

round ligament

introssious ligament

transverse ligament

A

B) Reason

LIGAMENTSbetween 2 Vertebra:• Supraspinous ligament: Connects adjacent spines.• Interspinous ligament: Connects adjacent spines.• Ligamentum flavum: Connects adjacent laminae

How well did you know this?
1
Not at all
2
3
4
5
Perfectly
453
Q

In a young pt. high levels of GH are seen it will lead to deficiency of what?
amino acids

carbohydrates

protein

vitamins

fatty acids

A

E) Reason

FUNCTION Stimulates linear growth and muscle mass through IGF-1/somatomedin secretion. Increases lipolysis and insulin resistance (diabetogenic).

How well did you know this?
1
Not at all
2
3
4
5
Perfectly
454
Q

Trypsinogen changes to trypsin ?
gastrin

secretin

bile

enterokinase

protease

A

D) Reason

Trypsinogen: Converted to active enzyme trypsin by enterokinaseto activation of other proenzymes and cleaving of additional trypsinogen molecules into active trypsin (positive feedback loop)

How well did you know this?
1
Not at all
2
3
4
5
Perfectly
455
Q

Pt has mild jaundice that is recurrent and has unconjugated Hyperbilirubinemia, Dx..?
Hepatitis

Wilson disease

Criglar naggar

Dubin johnson

Gilbert

A

E) Reason

Gilbert syndrome: Mildly dec. UDP-glucuronosyltransferase conjugation activity dec. bilirubin uptake by hepatocytes. Asymptomatic or mild off and on jaundice. Elevated unconjugated bilirubin without overt hemolysis.

How well did you know this?
1
Not at all
2
3
4
5
Perfectly
456
Q

which one has highest conductance rate
SA node

AV node

bundle of His

left atrium

Purkingi fibers

A

E) Reason

Speed of conduction—Purkinje > atria > ventricles > AV node.Pacemakers—SA > AV > bundle of His/Purkinje/ventricles.

How well did you know this?
1
Not at all
2
3
4
5
Perfectly
457
Q

Ends of long bones are supplied by
nutrient artery

periosteal artery

metaphyseal artery

periphyseal artery

epiphyseal artery

A

E) Reason

Epiphysial arteries • These are derived from periarticular vascular arcades (circulus vasculosus) found on the nonarticular bony surface. These supply the metaohysis and epiohysis.

How well did you know this?
1
Not at all
2
3
4
5
Perfectly
458
Q

drug of choice for tape worm
metronidazole

ciprofloxacin

albendazole

ampicillin

gentamicin

A

C) Reason

Ingestion of larvae encysted in undercooked pork.Ingestion of eggs. Treated with Albendazole or ivermectin.

How well did you know this?
1
Not at all
2
3
4
5
Perfectly
459
Q

Difference between plasma and serum
clotting factors

proteins

cells

ions

RBCs

A

A) Reason

Plasma is part of blood minus blood cells while serum is the part of plasma minus clotting factors.

How well did you know this?
1
Not at all
2
3
4
5
Perfectly
460
Q

Receptors which adjust swiftly?
Pacinian

Meissner”s

tactile receptor

Murcel discs

GTO

A

A) Reason

Rapidly adapting, or phasic, receptors (pacinian corpuscle; light touch)■ show a decline in action potential frequency with time in response to a constant stimulus.■ primarily detect onset and offset of a stimulus

How well did you know this?
1
Not at all
2
3
4
5
Perfectly
461
Q

Choose the correct narrayive about renal fascia
sheet of muscle

encloses vessels

encloses aorta

encloses IVC

encloses suprarenal

A

E) Reason

Renal fascia: This is a condensation of areolar tissue outside the perirenal fat. It encloses the kidneys and thesuprarenal glands

How well did you know this?
1
Not at all
2
3
4
5
Perfectly
462
Q

which cells of intestine may carry antigens?
M cells

G cells

goblet cells

enterocytes

villi

A

A) Reason

Peyer patches: Unencapsulated lymphoid tissue found in lamina propria and submucosa of ileum. Contain specialized M cells that sample and present antigens to immune cells

How well did you know this?
1
Not at all
2
3
4
5
Perfectly
463
Q

most commonly raised enzyme in liver disease
AST

ALP

pseudocholinestrase

ALT

enterokinase

A

D) Reason

Aminotransferases (AST and ALT) (often called “liver enzymes”)Viral hepatitis (ALT > AST), viral hepatitis is more common than alcoholic one.Alcoholic hepatitis (AST > ALT)

How well did you know this?
1
Not at all
2
3
4
5
Perfectly
464
Q

Outer covering of single muscle is called as
epimysium

perimysium

neuromysium

exomysium

endomysium

A

E) Reason

Endomysium surrounds each muscle fibre separately. Perimysium surrounds bundles (fasciculi or myonemes) of muscle fibres of various sizes. Epimysium surrounds the entire muscle.

How well did you know this?
1
Not at all
2
3
4
5
Perfectly
465
Q

Diabetic with HLA1, undergone transplant 1 month ago, which cells involved in rejection
CD4 cell

Th1 cells

HLA 4

CD4 with MHC1

CD8 with MHC1

A

E) Reason

Febrile nonhemolytic transfusion reaction:Type II hypersensitivity reaction. Host antibodies against donor HLA antigens and leukocytes. through MHC.

How well did you know this?
1
Not at all
2
3
4
5
Perfectly
466
Q

substantia gelatinosa is related to
temperature

tactile

pressure

pain

vibration

A

D) Reason

This is particularly important in relation to thespinothalamic and spinoreticular pathways and the perception of pain.Presynaptic inhibition influences many, possibly all, primary afferent terminals. Amuch-investigated site ofpresynaptic effects is the substantia gelatinosa.

How well did you know this?
1
Not at all
2
3
4
5
Perfectly
467
Q

which phase of action potential leads to rise in pulse pressure?
phase 1

phase 2

phase 0

phase 3

phase 4

A

E) Reason

Positive chronotropic effect■ increases heart rate by increasing the rate of phase 4 depolarization.■ More action potentials occur per unit time because the threshold potential is reached more quickly and, therefore, more frequently.

How well did you know this?
1
Not at all
2
3
4
5
Perfectly
468
Q

difference between plasma and interstitial fluid
less lipids

more proteins

less proteins

less cells

less clotting factors

A

C) Reason

Interstitial fluid is three-fourths of the ECF. Thus, it is one-fourth of TBW (3/4 × 1/3).■ The composition of interstitial fluid is the same as that of plasma except that it has little protein. Thus, interstitial fluid is an ultrafiltrate of plasma.

How well did you know this?
1
Not at all
2
3
4
5
Perfectly
469
Q

Anticoagulant which is present naturally?
plasminogen

thrombin

plasmin

factor 4

Hagmann factor

A

A) Reason

Thrombolytics Directly or indirectly conversion of plasminogen (a natural Thrombolytic) to plasmin, which cleaves thrombin and fibrin clots. inc. PT, inc. PTT, no change in platelet count.

How well did you know this?
1
Not at all
2
3
4
5
Perfectly
470
Q

Fibers of the cilliary ganglion are carried through which branch
nasocilliary

optic

maxillary

labial

nasal

A

A) Reason

Some fibers from the nasocilliary branch go and synapse in the ciliary ganglionand reach the eyeball in the short ciliary nerves.

How well did you know this?
1
Not at all
2
3
4
5
Perfectly
471
Q

Doctor can develop better communication by
informed consent

non malficence

compitence

active listening

knowledge

A

D) Reason

A better communication skill is one of the important step of the relationship between a Doctor and a patient, it can be best developed by active listening and respect the views and ideas of the patients at their level.

How well did you know this?
1
Not at all
2
3
4
5
Perfectly
472
Q

Percentage of fat in normal diet
5 to 10

10 to 15

15-20

20-25

52-30

A

C) Reason

Normal contribution of the major nutrients in diet are as: Carbohydrates 45-65 percent, proteins 25-40 percent and fats 15-20 percent.

How well did you know this?
1
Not at all
2
3
4
5
Perfectly
473
Q

38 years male with fever, and chills, now presented with Hb of 8 and spleenomegaly, Dx..?
CML

ALL

malaria

CLL

lymphoma

A

C) Reason

Here in this scenario, all the features given are in favour of malarial diagnosis. While decreased Hb is the feature of hemolysis by the RBC burst and leading to spleenomegaly.

How well did you know this?
1
Not at all
2
3
4
5
Perfectly
474
Q

secondary center of ossification is associated with
epiphysis

diaphysis

metaphysis

shaft

metaphyseal plate

A

A) Reason

Epiphysis The ends and tips of a bone which ossify from secondary centres are called epiphyses.

How well did you know this?
1
Not at all
2
3
4
5
Perfectly
475
Q

clorpromazine toxicity
hypokalemia

hyponatremia

hyperkalemia

dystonia

CRF

A

D) Reason

TOXICITY: Highly lipid soluble and stored in body fat; thus, very slow to be removed from body.Extrapyramidal system side effects (e.g., dystonia and dyskinesias). Treatment: benztropine or diphenhydramine.

How well did you know this?
1
Not at all
2
3
4
5
Perfectly
476
Q

nerve damaged in Erb”s paralysis
C1-2

C2-3

C5-6

C4-5

C6

A

C) Reason

UPPER TRUNK LESIONS OF THE BRACHIALPLEXUS (ERB-DUCHENNE PALSY)Upper trunk (C5-6) lesions of the brachial plexus result fromdisplacement of the head to the opposite side and depression of the shoulder on the same side, as during fallson the shoulder or in infants during a difficult delivery

How well did you know this?
1
Not at all
2
3
4
5
Perfectly
477
Q

what is true about Antibodies in Hashimoto thyroiditis
anti mitochondrial

anti SM Igs

anti globulin

anti thyroxin

antithyroglobulin

A

E) Reason

Hashimoto thyroiditis: Most common cause of hypothyroidism in iodine-sufficient regions; an autoimmune disorder (anti-thyroid peroxidase, antithyroglobulin antibodies).

How well did you know this?
1
Not at all
2
3
4
5
Perfectly
478
Q

quatran malaria is by which specie
P. malarie

P. falciparum

P. vivax

P. ovale

P. hilus

A

A) Reason

Plasmodium Malarie is a specie of the Malaria causing parasites. P.Malarie fever spikes are there after every 3 days (Qurtran malaria).

How well did you know this?
1
Not at all
2
3
4
5
Perfectly
479
Q

side effect of thiazide diuretics
hypokalemia

hyperkalemia

hypernatremia

hypercalcemia

hypocalemia

A

D) Reason

TOXICITY Hypokalemic metabolic alkalosis, hyponatremia, hyperGlycemia, hyperLipidemia, hyperUricemia, and hyperCalcemia. Sulfa allergy.

480
Q

calcitonin is given in
hyperthyroidism

paget disease

akalosis

hypoparathyroidism

hypercalcemia

A

E) Reason

Calcitonin■ is synthesized and secreted by the parafollicular cells of the thyroid.■ secretion is stimulated by an increase in serum [Ca2+].■ acts primarily to inhibit bone resorption.■ can be used to treat hypercalcemia and hyperparathyroidism.

481
Q

example of end artery is?
RCA

mesenteric

spleenic

central retinal

hepatic

A

D) Reason

Central retinal artery supplies rest of the layers of the retina. It is a branch of the ophthalmicartery and is an end artery. It enters the optic nerve on its lower surface 15-20 mm behind theglobe. The normal artery : vein ratio is 2 : 3

482
Q

A middle age Pt. with bone pain but normal Ca and increased PO4, cause..?
hypercalcimia

Achondroplasia

multiple myeloma

paget disease

sarcoma

A

D) Reason

Clinical features 1. Bone pain- due to microfractures 2. Increasing hat size- Skull is commonly affected. 3. Hearing loss-impingement on cranial nerve 4. Lion-like facies- involvement of craniofacial bones 5. Isolated elevated alkaline phosphatase-most common cause of isolated elevated alkaline phosphatase in patients> 40 years old

483
Q

A Pt. developed reaction aftet 15 min of transfusion, it is a type of reaction
type 1 HSR

delayed response

type 3 HSR

type 2 HSR

type 4 HSR

A

D) Reason

Type II: Autoimmune hemolytic anemiaPernicious anemiaIdiopathic thrombocytopenic purpuraErythroblastosis fetalisAcute hemolytic transfusion reactionsRheumatic feverGoodpasture syndromeBullous pemphigoidPemphigus vulgaris

484
Q

post op pain killer to a diabetic
Nalbin

diclophenac

aspirin

Ketorolac

midazolam

A

D) Reason

As NSAIDs are contraindicated in chronic renal failure and for the best control in DM and its complication (as CRF) is the Ketorolac.

485
Q

which structure is most commonly damaged in anatomical snuff box
axillary nerve

radial artery

radial nerve

ulnar nerve

brachial artery

A

C) Reason

When the thumb is fully extended, a depression known as the anatomical snuffbox is seen on the lateralaspect of the wrist, immediately distal to the radial styloid process. Radial nerve is susceptible to injury here in this box.

486
Q

stress hormone is?
Cortisol

insulin

ADH

Aldosterone

Dopamine

A

A) Reason

Stress hormones are the hormones that cope with the stressful conditions. these include GH, Epinephrine, Norepinephrine, Cortisol and glucagon.

487
Q

Which Lymph Nodes are involved in stage 4 cervical cancer
external iliac

internal iliac

aortic

internal & external iliac

deep inguinal

A

D) Reason

Lymph Drainage:The Cervical cancer drains into the internal and theexternal iliac nodes.

488
Q

in cardiac potential which phase is plateu
1

2

3

4

5

A

B) Reason

Phase 2 = plateau—Ca2+ influx through voltage-gated Ca2+ channels balances K+ efflux. Ca2+ influx triggers Ca2+ release from sarcoplasmic reticulum and myocyte contraction.

489
Q

A pt. presented with high grade fever and chill is collapsed, Hb is 5, Dx..?
miningitis

cerebral malaria

encephalitis

epilepsy

hydrocephalus

A

B) Reason

All the features given in scenario are characteristics of malaria while H/O collapse indicates its CNS association that is cerebral Malaria.

490
Q

Submandibular gland surgery is performed & accidentally a nerve is damaged. most likely nerve is
facial

mandibular

marginal mandibular

maxillary

ophthalmic

A

C) Reason

Nerve SupplyParasympathetic secretomotor supply is from the facialnerve via the chorda tympani. Marginal mandibular branch is closely related to gland and is frequently injured in its surgery.

491
Q

adrenal cortex is derived from
ectoderm

endoderm

Mesoderm

meuroectoderm

yolk sac

A

C) Reason

drenal cortex and medulla: Adrenal cortex (derived from mesoderm) and medulla (derived from neural crest).

492
Q

A female with femur fracture now presented with limb swelling, what will be next pathology.?
DIC

DVT

sepsis

shock

fat embolism

A

B) Reason

Venous thrombosis arises most often in the deep veins of the lower extremities with extremely painful swollen limb. It is often associated with inflammation and is then termed thrombophlebitis.

493
Q

In a 43 year old ill looking man, gene/nuclear amplification is characteristic of which of following condition
myxoma

neuroblastoma

ewing sarcoma

hemangioma

medulloblastoma

A

B) Reason

Most common presentation is abdominal distension and a firm, irregular mass that can cross the midline (vs. Wilms tumor, which is smooth and unilateral). Homovanillic acid (HVA), a breakdown product of dopamine, inc.in urine. Bombesin +ve . Less likely to develop hypertension. Associated with overexpression of N-myc oncogene.

494
Q

poplitial fossa is medially bound to?
sartorius

semimembranosus

adductor longus

adductor brevis

quadricep

A

B) Reason

BOUNDARIES• Laterally: The biceps femoris muscle above and the lateralhead of the gastrocnemius and plantaris muscles below.• Medially: The semimembranosus and semitendinosusmuscles above and the medial head of the gastrocnemiusmuscle below

495
Q

which bone fracture will result in radial nerve damage?
humerus

radius

scapula

ulnar head

radial head

A

A) Reason

FRACTURES OF THE HUMERUS AND INJURYTO THE RADIAL NERVEThe radial nerve can be damaged where it lies in thespiral groove on the posterior surface of the shaft ofthe humerus under cover of the triceps muscle.

496
Q

adipose tissue are affected by insulin. How?
inc. glycogen synthesis

inc. gulconeogenesis
inc. lipolysis
inc. lipoprotein lipase

lipase

A

D) Reason

Anabolic effects of insulin:Inc. glucose transport in skeletal muscle and adipose tissueInc. glycogen synthesis and storageInc. triglyceride synthesisInc. lipoprotein lipase in adipose tissueInc. protein synthesis (muscles, proteins)Inc. cellular uptake of K+ and amino acids.

497
Q

hip extension and knee Flexion is carried out by
gracillus

adductor longus

semitendinosus

sartorius

quadriceps

A

C) Reason

• Muscles for Flexion at Knee: Biceps femoris, semitendinosus, and semimembranosus muscles.

498
Q

Which structure is found between bellies of digastric and mandible
parotid

Sublingual

sub maxillary

submental nodes

Submandibular gland

A

E) Reason

SUBMANDIBULAR GLANDThe submandibular gland consists of a mixture of serousand mucous acini. It lies beneath the lower border of thebody of the mandible and bellies of digastric. It is divided into superficial anddeep parts by the mylohyoid muscle.

499
Q

mucopolysacchride is abundantly found in
elastin

bone

collagen

cartilage

Tendon

A

D) Reason

Cartilage is a connective tissue composed of cells (chondrocytes) and fibres (collagen or yellow elastic) embedded in a firm, gel-like matrix which is rich in a mucopolysaccharide

500
Q

local anesthetic with rapid metabolism and less side effects
lignocaine

bupivacaine

prilocaine

mepivacaine

ropivacaine

A

C) Reason

There is considerable variation in the rate of liver metabolism of individual amide compounds, with prilocaine (fastest) > lidocaine > mepivacaine > ropivacaine ≈ bupivacaine and levobupivacaine (slowest).

501
Q

posterior triangle biopsy was taken, nerve likely to be affected
axillary

hypoglossal

thoracodorsal

long thoracic

spinal part of acessary

A

E) Reason

POSTERIOR TRIANGLEThe posterior triangle is bounded posteriorly by the trapezius muscle, anteriorly by the sternocleidomastoid muscle,and inferiorly by the clavicle. Two most important structures in it are the spinal part of acessary nerve and Subclavian vein.

502
Q

In a 42 year old man diabetic for 14 years, diabetic retinopathy is due to
macroangiopathy

hyperglycemia

osmotic pressure

microangiopathy

neuropathy

A

D) Reason

Small vessel disease (diffuse thickening of basement membrane) retinopathy (hemorrhage, exudates, microaneurysms, vessel proliferation) A, glaucoma, nephropathy (nodular sclerosis, progressive proteinuria, chronic renal failure, arteriolosclerosis leading to hypertension, Kimmelstiel-Wilson nodules)

503
Q

pH of filterate is high in PCT than DCT as H ions is…..in DCT
reabsorbed

secreted

filtered

double

become half

A

B) Reason

Early distal convoluted tubule (DCT)—actively reabsorbs Na+, Cl-. Makes urine hypotonic.PTH—↑ Ca2+/Na+ exchange → Ca2+reabsorption.Secrets H ions.5–10% Na+ reabsorbed.

504
Q

In Patient with mitral valve replacement, which infects artificial heart valve
staph epidermidis

staph aureus

staph mutan

strep viridan

strep Pneumoniae

A

A) Reason

Staphylococcus epidermidisInfects prosthetic devices and intravenous catheters by producing adherent biofilms. Component of normal skin flora; contaminates blood cultures. Novobiocin sensitive.

505
Q

prostate is drained by
internal iliac

external iliac

internal & external iliac

deep inguinal

aortic

A

C) Reason

Lymph DrainageThe lymph drains from the prostate goes into the internal iliac and external iliec nodes.

506
Q

On reaching mount everest a man developed severe dyspnea, reason.?
hypoxia

polycythemia

anemia

toxemia

pulmonary edema

A

E) Reason

Acute pulmonary edema. The cause of this is stillunknown, but a suggested answer is the following:The severe hypoxia causes the pulmonary arteriolesto constrict potently, but the constriction is muchgreater in some parts of the lungs than in otherparts, so that more and more of the pulmonaryblood flow is forced through fewer and fewer stillunconstricted pulmonary vessels.

507
Q

abductor of thigh is mediated by
gluteus maximus

Obturator internus

sartorius

pectineus

gluteus minimus

A

E) Reason

Gluteus minimus is preferred answer here.

508
Q

Regarding neoplasms, ameloblastoma is the tumor of
maxilla

mandible

vomer

ethmoid

palatine

A

B) Reason

Ameloblastoma (adamantinoma) is an epithel ial tumor arising from precursor cells of the enamel organ. (1) Most frequently, this tumor occurs in the mandible. It usually appears in individuals younger than 35 years of age

509
Q

floculonodular lobe is associated with
uncus

substantia nigra

amygdala

hippocampus

cerebellum

A

E) Reason

Medial lesions—Lesions involving midline structures (vermal cortex, fastigial nuclei) and/or the flocculonodular lobe result in truncal ataxia, nystagmus, and head tilting.

510
Q

A patient with chronic toothaches, after few days of 3rd molar extraction, patient developed submandibular swelling, organism involved
staph aureus

strep. mutans

staph epidermis

strep virridan

staph saprophyticus

A

B) Reason

Viridans streptococci are α-hemolytic. They are normal flora of the oropharynx and cause dental caries (Streptococcus mutans) andsubacute bacterial endocarditis at damaged valves (S. sanguinis)

511
Q

A middle age female diabetic presents with femer fracture on X-ray, osteoporotic changes seen, etiology can be confirmed by?
DEXA scan

CT scan

bone scan

Serum Ca

PO4

A

A) Reason

Osteoporosis Trabecular (spongy) bone loses mass and interconnections despite normal bone mineralization and lab values (serum Ca2+ and PO43-). Diagnosis by a bone mineral density test (DEXA) with a T-score of ≤ -2.5. Can be caused by long-term exogenous steroid use.Can lead to vertebral crush fractures—acute back pain, loss of height, kyphosis

512
Q

What can be histological changes on liver biopsy in chronic HCV
regeneration

portal fibrosis

portal congestion

sinus congestion

generalized fibrosis

A

E) Reason

Cirrhosis is a descriptive term for chronic liver disease characterized by generalized disorganization of hepatic architecture with scarring and nodule formation. Liver cell damage, regenerative activity, and generalized fibrosis reSUlting in a nodular pattern are also characteristic.

513
Q

which is abundant in collagen and amorphous substance
hyaline cartilage

fibrocartilage

elastic cartilage

collagen type 5

elastin

A

B) Reason

Fibrocartilage has type I collagen in addition to type II, giving it greater resistance to being stretched (tensile strength), and is the type of cartilage found in intervertebral disks of the vertebral column and the menisci of the knee, and may form the attachment of ligaments and tendons to bone

514
Q

myocardial muscles are affected by norepinephrine by?
sec. strength

inc. rate
inc. stroke volume
inc. contraction
inc. atrial contraction

A

D) Reason

Release of norepinephrine from a sympathetic nerve ending is modulated by norepinephrine itself, acting on presynaptic α2-autoreceptors, angiotensin II, and other substances. It has positive chronotropic effect on heart.

515
Q

P wave has no relation with QRS complex in which cardiac state?
atrial flutter

1st degree block

2nd degree block

mobitz type 1

3rd degree block

A

E) Reason

3rd degree (complete):The atria and ventricles beat independently of each other. Both P waves and QRS complexes are present, although the P waves bear no relation to the QRS complexes. The atrial rate is faster than the ventricular rate.

516
Q

peptic ulcer pain is carried by
small gastric

great splanchnic nerve

thoracic splanchnic

vagus

short gastric

A

B) Reason

Branches from the Great Splanchnic nerves supply the viscera of the Abdominal cavity. Sympathetics to the foregut and midgut (thoracic splanchnic nerves: T5-T12) and to the hindgut and pelvic viscera (lumbar splanchnic nerves: L1-L2) synapse in collateral ganglia

517
Q

Before waking cortisol levels are at its peak in body. Cause?
circadian rhythm

stress

deep sleep

circannual

morning stress

A

A) Reason

Glucocorticoid secretion ■ oscillates with a 24-hour periodicity or circadian rhythm.■ For those who sleep at night, cortisol levels are highest just before waking (ª8 am) and lowest in the evening (≈12 midnight).

518
Q

perineal region is derived from which dermatome
S1-2

S2-3

S3-4

S4-5

L5

A

C) Reason

L1—at the inguinal ligament.L4—includes the kneecaps.S2, S3, S4—erection and sensation of penile and anal zones

519
Q

what can be the gross appearance of Fibroadenoma
desmoplasia

Metaplasia

hypoplasia

dysplasia

hyperplasia

A

A) Reason

FIBROADENOMA A. Tumor of fibrous tissue and glands by desmoplasia B. Most common benign neoplasm of the breast; usually seen in premenopausal women C. Presents as a well-circumscribed, mobile marble-like mass

520
Q

Sudden up and back shoulder movement, later unable to abduct arm, reason is?
deltoid rupture

bursitis

cuff tendinitis

supraspinatus rupture

fatigue

A

C) Reason

ROTATOR CUFF TENDINITISThe rotator cuff is a common site of tendinitis and cancause severe pain in the shoulder region.

521
Q

A patient is at end stage renal disease, and is ready for dialysis has Low Hb, MCV 76 and low MCHC, anemia type
microcytic

normocytic hypochromic

microcytic hypochromic

macrocytic hypochromic

microcytic normochromic

A

B) Reason

Anemia of chronic disease: Is an example of normocytic hypochromic anemia, Inflammation inc. hepcidin (released by liver, binds ferroportin on intestinal mucosal cells and macrophages, thus inhibiting iron transport) dec. release of iron from macrophages.

522
Q

treatment for cutaneous Larvae migrans
ciprofloxacin

ampicillin

Doxycyline

gentamycin

rifampin

A

C) Reason

Common in northeastern United States.Initial symptoms—erythema chronicum migrans B, flu-like symptoms, +/- facial nerve palsy.Later symptoms—monoarthritis (large joints) and migratory polyarthritis, cardiac (AV nodal block), neurologic (encephalopathy, facial nerve palsy, polyneuropathy).Treatment: doxycycline, ceftriaxone.

523
Q

CSF formation begins from
ultrafiltration of K

ultrafiltration of Na

ultrafiltration of Cl

ultrafiltration of H

ultrafiltration of Ca

A

B) Reason

A total of 400-500 cc of CSF is produced per day; most important step is the first one that is ultrafiltration of Na; ventricles and subarachnoid space contain 90-150 cc, so all of CSF is turned over 2-3 times per day.

524
Q

right kidney is related anteriorly to
hepatic flexure

1at part of duedenum

spleenic vein

pancreas head

tail of pancreas

A

A) Reason

Important Relations, Right Kidney■ Anteriorly: The suprarenal gland, the liver, the second part of the duodenum, and the right colic flexure ■ Posteriorly: The diaphragm; the costodiaphragmatic recess of the pleura; the 12th rib; and the psoas, quadratus lumborum, and transversus abdominis muscles.

525
Q

A patient in ER with atypical chest pain, on ECG, ST elevation in lead 4, artery involved
marginal artery

RCA

LCA

LXA

LAD

A

E) Reason

Left anterior descending artery (LAD)—supplies anterior 2/3 of interventricular septum, anterior papillarymuscle, and anterior surface of left ventricle (Lead 4 & 5 are lateral leads).

526
Q

10 months old child, dehydration should be assessed by
lips

eyes

tongue

occiput

bregma

A

E) Reason

In neonates, due to immature sutures, their fontanelle become prominent in dehydrated states, that is why, bregma (anterior fontanelle) best to assess dehydration status.

527
Q

A polyp is seen in endocervix with squamous epithelium, condition is..?
Metaplasia

dysplasia

Metaplasia

hypoplasia

anaplasia

A

A) Reason

Cervical polyps are inflammatory proliferations of cervical mucosa; they are not true neoplasms. Most common of these polyps are the squamous by Metaplasia of the cervical epithelium.

528
Q

Which is not found at free margin of lesser omentum
hepatic artery

portal vein

potal vein & CBD

CBD

spleenic artery

A

E) Reason

BOUNDARIES OF THE EPIPLOIC FORAMEN• Anteriorly: Free border of the lesser omentum, the bileduct, the hepatic artery, and the portal vein.• Posteriorly: Inferior vena cava.• Superiorly: Caudate process of the caudate lobe of theliver.• Inferiorly: First part of the duodenum.

529
Q

No. of muscle layers in sole
1

2

4

5

6

A

C) Reason

Sole of FootThe muscles of the sole are usually described in four layers (from inferior to superior).

530
Q

lytic lesion on scalp X-ray, confirmatory test..?
Urine complete

Bone scan

Protein Electrophoresis

CT scan

PCR

A

C) Reason

Associated with:Inc. susceptibility to infectionPrimary amyloidosis (AL)Punched-out lytic bone lesions on x-ray AM spike on serum protein electrophoresisIg light chains in urine (Bence Jones protein) Rouleaux formation (RBCs stacked like poker chips in blood smear)

531
Q

primary ossification of humerus starts at which week
2

4

8

10

12

A

C) Reason

The primary centre for the shaft appears near its middle in the eighth week of intrauterine life, andgradually extends towards the ends.

532
Q

Anterior two third of tongue taste sensation is innervated by
facial

mandibular

auricular

auriculotemporal

glossopharyngeal

A

A) Reason

SENSORY INNERVATION• Anterior two thirds: The lingual nerve (general sensation) and chorda tympani (taste).• Posterior third: The glossopharyngeal nerve (general sensation and taste).

533
Q

stratified squamous non keratinized epithelium is found in
sclera

conjunctiva

skin

cornea

lungs

A

D) Reason

The sclera is the external layer and continues anteriorly as the cornea, which is transparent (stratified squamous non keratinized epithelium) and allows light to enter the eye. The intermediate choroid layer is highly vascularized and pigmented.

534
Q

lymphoid tissue with respiratory epithelium
lymph nodes

spleen

liver

suprarenal

thymus

A

E) Reason

THYMUS:The thymus is a lymphoid tissue with respiratory epithelium, is divided into a distinct cortex and medulla. The thymus serves as a site of maturation ofT-lymphocyte precursors after they exit from the bone marrow and arrive at the thymus via blood circulation.

535
Q

most common metastatic malignancy in 2nd decade
SCC

HCC

ewing sarcoma

HPV

gastric CA

A

C) Reason

Ewing sarcoma:Boys < 15 years old.Commonly appears in diaphysis of long bones, pelvis, scapula, and ribs.

536
Q

In a patient with multiple myloma, cause of polyurea is
hyperkalemia

hyponatremia

acidosis

hypokalemia

hypocalcemia

A

D) Reason

Clinical features include Bone pain with hypokalemia and hypercalcemia-Neoplastic plasma cells activate the RANK receptor on osteoclasts, leading to bone destruction. Lytic, “punched-out” skeletal lesions are seen on x-ray, especially in the vertebrae and skull; increased risk for fracture. Presents with polyuria, bone pain and lytic lesions.

537
Q

After surgery, Pt. is with hypotension and shock, why?
hypovolumic

dehiscence

Toxemic

burst abdomen

shock

A

C) Reason

All the features given in scenario are characteristics of shock (low BP and tachycardia) while H/O surgery points to the Toxemic shock.

538
Q

epicondyle ossification in female
4 yr

5 yr

6 yr

7 yr

8 yr

A

A) Reason

Ossification begins in the medial epicondyle in the fourth year in females, sixth in males, and in the lateralepicondyle about the 12th year.

539
Q

crus cerebri are closely related to
cerebellum

uncus

amygdala

hippocampus

substantia nigra

A

E) Reason

For descriptive purposes, the midbrain may be divided into a dorsaltectum and right and left cerebral peduncles, each of which is further divided into a ventral crus cerebriand a dorsal tegmentum by a pigmented lamina, the substantia nigra.

540
Q

resting tremors, expressionless face, lesion is at
striatum

substantia nigra

cerebellum

putamin

globus palidus

A

B) Reason

Parkinson —Tremor (at rest—e.g., pill-rolling tremor), cogwheel Rigidity, expressionless face,Akinesia (or bradykinesia), Postural instability and Shuffling gait.

541
Q

In patient with acute chest pain, 5mm ST elevation in lead 2, artery involved
LXA

posterior branch

LCA

RCA

LAD

A

D) Reason

Posterior descending/interventricular artery (PDA)—supplies posterior 1/3 of interventricular septum (lead 1-4 are the anterior chest leads) and posterior walls of ventricles.

542
Q

Tumor on anterior 2/3 of tongue, Lymph Nodes involved
submental

Sublingual

Submandibular

parotid

anterior cervical

A

C) Reason

LYMPH DRAINAGEf• Tip: Submental lymph nodes.• Sides of anterior two thirds: Submandibular lymphnodes.• Posterior third: Deep cervical lymph nodes.

543
Q

which ligament is triangle shaped
pubovesical

pubofemoral

puborectalis

pectineal

perineal membrane

A

B) Reason

Pubofemoral LigamentThe pubofemoral ligament is triangular in shapeThe base is attached above to the superior ramus of the pubis, and the apex is attached below to the lower end of theintertrochanteric line.

544
Q

Lactation is sustained by
LH

FSH

GH

Prolactin

Oxytoxin

A

D) Reason

Prolactin—induces and maintains lactation and reproductive function. Oxytocin—assists in milk letdown; also promotes uterine contractions

545
Q

Most potent antioxidant
vit. A

vit C

vit. D
vit. B2
vit. E

A

E) Reason

Vitamin E (tocopherol/tocotrienol)FUNCTION: Antioxidant (protects erythrocytes and membranes from free radical damage).

546
Q

mechanism of aqueous humor production
filtration

secretion

absorption

outflow

ultrafiltration

A

E) Reason

AQUEOUS HUMOR:The aqueous humor is a clear fluid formed by ultrafiltration that fills the anterior andthe posterior chambers.

547
Q

Which is not boundary of lateral arch of foot
cuneform

cuboid

4th metatarsal

1st metatarsal

5th metatarsal

A

D) Reason

Lateral Longitudinal ArchThe lateral longitudinal arch is formed by the calcaneum,the cuboid, and the fourth and the fifth metatarsal bones.

548
Q

IVC gives anterior branch known as?
spleenic vein

portal vein

IVC

hepatic vein

great splanchnic vein

A

D) Reason

TRIBUTARIES• Two anterior visceral tributaries (the hepatic veins).• Three lateral visceral tributaries: the right suprarenal vein(the left vein drains into the left renal vein), renal veins,and right testicular or ovarian vein (the left vein drains intothe left renal vein).

549
Q

Which of these is responsible fro Quiet expiration?
diaphragm

intercostal

abdominal muscles

recoil of lungs

pleura

A

D) Reason

Quiet ExpirationQuiet expiration is a passive process accomplished by theelastic recoil of the lungs and the relaxation of the intercostal muscles and diaphragm.

550
Q

veinous blood hematoma is?
epidural

subdural

subarachnoid

Interventricular

intracranial

A

B) Reason

A subdural hematoma results from head trauma that tears superficial (“bridging”) cerebral veins at the point where they enter the superior sagittal sinus. A subdural hemorrhage occurs between the meningeal dura and the arachnoid.

551
Q

which measures variability among values
sensitivity

specificity

standard deviation

chi square test

controlled trials

A

C) Reason

Standard deviation = how much variability exists from the mean in a set of values.Standard error of the mean = an estimation of how much variability exists between the sample mean and the true population mean.

552
Q

Regarding aspesis, Disinfectant for skin is
alcohol

formalin

H2O2

methylene

phenol

A

A) Reason

Ethanol is widely used to clean the skin before immunization or venipuncture. It actsmainly by disorganizing the lipid structure in membranes, but it denatures proteins aswell. Ethanol requires the presence of water for maximal activity (i.e., it is far moreeffective at 70% than at 100%).

553
Q

What should not be absorbed without digestion?
glucose

glycerol

glactose

fructose

sucrose

A

E) Reason

All the carbohydrates given in the list are monosacchrides and need no further digestion being simplest except Sucrose that is discchride and must be digested further to a mono sugar before absorption.

554
Q

A patient with long term history of raised B.P. and Creatinine, and dec. erythropoitin, defect..?
JG cells

glomerulus

DCTs

loop of Henle

interstium

A

A) Reason

Features in scenario are of CRF in which JG cells are damaged as well. JG cells (modified smooth muscle of afferent arteriole) and the macula densa (NaCl sensor, part of the distal convoluted tubule). JG cells secrete renin in response to dec. renal blood pressure, dec. NaCl delivery to distal tubule, and inc. sympathetic tone (β1)

555
Q

Acrosome is formed in
RER

golgi complex

SER

cell membrane

nucleus

A

B) Reason

The acrosome, which is located over the anterior half of the nucleus, is derived from the Golgi complex of the spermatid and contains several hydrolytic enzymes such as hyaluronidase, neuraminidase, and acid phosphatase; these enzymes dissociate cells of the corona radiata and digest the zona pellucida of the recently produced secondary oocyte.

556
Q

A young boy in Village developed severe respiratory infection, which is facultative anaerobe causing infection with unpasteurized dairy products
Pseudomonas

Actinomycetes

spirochete

L. monocytogenes

N. miningitis

A

D) Reason

Listeria monocytogenes:Facultative intracellular microbe; acquired by ingestion of unpasteurized dairy products and deli meats, via transplacental transmission, or by vaginal transmission during birth.

557
Q

Role of pneumotaxic center
expiration center

limits inspiration

inc. respiration

limits expiration

control hypoxia

A

B) Reason

A pneumotaxic center, located dorsally in the nucleusparabrachialis of the upper pons, transmits signals tothe inspiratory area. The primary effect of this centeris to control the “switch-off” point of the inspiratoryramp, thus controlling the duration of the filling phaseof the lung cycle

558
Q
function of , corticospinal tract
voluntary movement

coordination

balance

involuntary movement

gaze control

A

A) Reason

• The corticospinal tract is involved in the voluntary contraction of skeletal muscle, especially in the distal extremities. This pathway consists of 2 neurons, an upper motor neuron, and a lower motor neuron

559
Q

Which of these is neurohypophysis
adrenal medulla

anterior pituitary

median lobe

hypothalamus

posterior pituitary

A

E) Reason

Posterior pituitary (neurohypophysis)Secretes vasopressin (antidiuretic hormone, or ADH) and oxytocin, made in the hypothalamus and shipped to posterior pituitary via neurophysins (carrier proteins). Derived from neuroectoderm

560
Q

A female has breast masses bilaterally, which Cancer has most potentail to involve breasts bilaterally
lobular in situ

ductal CA

Fibroadenoma

phyllodes

papilloma

A

A) Reason

LOBULAR CARCINOMA IN SITU (LCIS) A. Malignant proliferation of cells in lobules with no invasion of the basement membrane B. LCIS does not produce a mass or calcifications and is usually discovered incidentally on biopsy.

561
Q

Pt has nystagmus, site of lesion
vermis

striatum

peduncles

floculonodular lobe

median lobe

A

D) Reason

Medial lesions—Lesions involving midline structures (vermal cortex, fastigial nuclei) and/or the flocculonodular lobe result in truncal ataxia, nystagmus, and head tilting.

562
Q

Control of defecation reflex is by
lumber splanchnic

sacral plexus

pelvic splanchnic

internal sphincter

pelvic diaphragm

A

C) Reason

The internal anal sphincter is circular smooth muscle that surrounds the anal canal. The sympathetics (lumbar splanchnics) increase the tone of the muscle and the parasympathetics (pelvic splanchnics) relax the muscle during defecation.

563
Q

A patient has got burnt accidently and developed edema, cause of edema in burn patient
plasma loss

shock

hypokalemia

hypoalbuminemia

hyponatremia

A

D) Reason

In a burn patient, ECF and plasma is lost along with the plasma proteins especially the albumin. This state of hypoalbuminemia causes edema.

564
Q

A patient with positive family history, has fasting blood sugar of 140, what is your diagnosis
pre diabetes

poor control

abnormal glucose tolerance

diabetes

no diabetes

A

D) Reason

TESTS: Fasting serum glucose of >126mg/dl is diagnostic , oral glucose tolerance test, HbA1c (reflects average blood glucose over prior 3 months).

565
Q

Pt. knowledge about disease and drug side effects will decide
informed consent

communication

prognosis

effectiveness

compliance

A

E) Reason

Complience is defined as the adherence of patients to the treatment and it largely depends upon knowledge of pt about disease and drug and its side effects.

566
Q

part of urethra passes through urogenital diaphragm is?
prostatic

membranous

bulbar

spongium

cavurnus

A

B) Reason

MEMBRANOUS URETHRAThe membranous urethra passes through the urogenital diaphragm and is surrounded by the sphincter urethrae. It is the shortest and least dilatable part of the urethra.

567
Q

epiploic foramin is formed inferiorly by?
colon

1st part of duodenum

spleen

hepatic flexor

3rd part of duodenum

A

B) Reason

BOUNDARIES OF THE EPIPLOIC FORAMEN• Anteriorly: Free border of the lesser omentum, the bileduct, the hepatic artery, and the portal vein.• Posteriorly: Inferior vena cava.• Superiorly: Caudate process of the caudate lobe of theliver.• Inferiorly: First part of the duodenum.

568
Q

A chronic diabetic Pt with wound on thigh, now presented with hypotension amd tachycardia, cause
DIC

ITP

sepsis

hypovolumic shock

septic shock

A

E) Reason

All the features given in scenario are suggestive of shock (hypotension and tachycardia) while H/O wound previouly fevers the condition of being sepsis.

569
Q

barorecptors act & detect alteration in?
venous pressure

diastolic pressure

TPR

arterial pressure

chemical response

A

D) Reason

Aortic arch barorecptors respond to inc but not to decreased while carotid sinus receptors dectect increase as well deceased in arterial pressure

570
Q

Drainage of left gastric vein
spleenic vein

IVC

hepatic vein

splanchnic vein

portal vein

A

E) Reason

Veinsof Stomach:The veins drain into the portal circulation. The right and leftgastric veins drain into the portal vein. The short gastric and the left gastroepiploic veins drain into the splenic vein,and the right gastroepiploic vein drains into the superiormesenteric vein.

571
Q

cachexia is mediated by
IL 1

TNF alpha

IL2

CRP

IL3

A

B) Reason

Cachexia Weight loss, muscle atrophy, and fatigue that occur in chronic disease (e.g., cancer, AIDS, heart failure, TB). Mediated by TNF-α (nicknamed cachectin), IFN-γ, and IL-6

572
Q

All are secretions of ant. pituitary except
FSH

LH

ADH

GH

TSH

A

C) Reason

Anterior pituitary (adenohypophysis)Secretes FSH, LH, ACTH, TSH, prolactin, GH, melanotropin (MSH). Derived from oral ectoderm (Rathke pouch)

573
Q

CSF is present in which space
subdural

epidural

sub Arachnoid

Arachnoid

spinal canal

A

C) Reason

• Subarachnoid space lies between the arachnoid and pia mater containing CSF: site of subarachnoid hemorrhage (described later).

574
Q

Placenta normally acts as……for fetal blood?
channel

barrier

filter

facilitator

exchanger

A

B) Reason

The placenta permits exchange of nutrients and removal of waste products between maternal and fetal circulations. The fetal component consists of the chorionic plate and villi. The maternal component is decidua basalis. Maternal blood is separated from fetal blood by the cytotrophoblast and syncytiotrophoblast.

575
Q

which lymphoid tissue lies posterior to nasopharynx
pharyngeal tonsil

palatine tonsil

thymus

Lymph Nodes

adenoid

A

A) Reason

The pharyngeal tonsil is an aggregate of nodular and diffuse lymphatic tissue within the posterior wall of the nasopharynx.

576
Q

Glucose & sodium are actively absorbed in intestine. which part is main contributor?
duodenum

ilium

cecum

colon

jejunum

A

E) Reason

Examples are Na+-glucose cotransport in the small intestine especially jejunum and renal early proximal tubule and Na+–K+ –2Cl– cotransport in the renal thick ascending limb.

577
Q

foot is everted by
sartorius

peroneus longus

adductor longus

soleus

popliteus

A

B) Reason

• Eversion (opposite movement of the foot so that the solefaces laterally): Peroneus longus, peroneus brevis, peroneus tertius, and lateral tendons of extensor digitorumlongus muscles.

578
Q

In expiration, breathing lowers because of
diaphragm

abdominal muscles

intercostal

pleura

recoil of lungs

A

B) Reason

Forced ExpirationForced expiration is an active process accomplished bycontraction of the muscles of the anterior abdominal wall(forcing the relaxed diaphragm upward by raising intraabdominal pressure) and contraction of the quadratuslumborum (pulling the twelfth rib downward).

579
Q

femoral artery begins at
adductor canal

femoral artey

femoral Ligament

inguinal ligament

adductor longus

A

D) Reason

Femoral ArteryThe femoral artery is a continuation of the external iliac artery. It begins behind the inguinal ligament, where itlies midway between the anterior superior iliac spine and thesymphysis pubis (the site for taking a femoral pulse).

580
Q

A patient developed chest pain and cough, pain not related to respiration
muscular

ribs pain

myocardial

pericardial

pleural

A

C) Reason

A chest pain that doesn”t change with any body movements or respiration is a pain of myocardium While pain of muskulosketal origin changes with movements.

581
Q

hyperventilation is accompanied by tetanic spasm. why?
dec. O2

dec. pH

lactic acidosis

fatigue

Ca squestration

A

E) Reason

In hyperventilation, CO2 is washed out leading to respiratory alkalosis. Conc. of H ions decreases and then plasma proteins use Ca for binding in place of H laeding hypocalcemia (and tetanic spasms).

582
Q

In dehydrated pt. K is given. how will K enter through?
Na/K pump

H/K pump

H/Cl pump

H/glucose pump

K/Ca exchange

A

A) Reason

Na+, K+ -ATPase (or Na+–K+ pump) in cell membranes transports Na+ from intracellular to extracellular fluid and K+ from extracellular to intracellular fluid; it maintains low intracellular [Na+] and high intracellular [K+].

583
Q

femoral triangle is laterally bound by
sartorius

gastricnemius

solius

popliteus

quadriceps

A

A) Reason

Its boundaries are as follows:• Superiorly: The inguinal ligament.• Laterally: The sartorius muscle.• Medially: The adductor longus muscle

584
Q

Pt. utters few words, damage is to
Wernicke

global

fronatal

temporal lobe

Broca”s area

A

E) Reason

Broca”s Area lesion: Nonfluent aphasia with intact comprehension.Broca area—inferior frontal gyrus of frontal lobe.

585
Q

facial nerve arises from
hypoglossal canal

facial canal

maxillary foramen

mastiod foramen

stylomastoid foramin

A

E) Reason

he facial nerve enters the internal acoustic meatus and thenthe facial canal. It emerges through the stylomastoid foramen, and it enters the parotid salivary gland.

586
Q

difference of ECF to ICF
less Na

more Na

less HCO3

less proteins

less Osmolarity

A

B) Reason

Extracellular fluid (ECF)■ is one-third of TBW.■ is composed of interstitial fluid and plasma. The major cation of ECF is Na+.■ The major anions of ECF are Cl- and HCO3.

587
Q

1st defense against microbes
skin

spleen

mucous membrane

neutrophils

lymphocytes

A

A) Reason

Innate defenses can be classified into three major categories: (1)physical barriers, such as intact skin and mucous membranes; (2) phagocytic cells,such as neutrophils, macrophages, and natural killer cells; and (3) proteins, such ascomplement, lysozyme, and interferon

588
Q

Which of the following is a potassium sparing diuretic?
Amiloride

dorzolamide

ethazolamide

triamitrine

brinzolamide

A

A) Reason

K+-sparing diuretics: Spironolactone and eplerenone; Triamterene, and Amiloride. CLINICAL USE: Hyperaldosteronism, K+ depletion, CHF.

589
Q

hypoglycemia brings about ecg changes. which one?
Tall T wave

small T wave

dec. QT interval

Inc. QT interval

bizzare complexs

A

D) Reason

Causes of Long QT interval:1. Bradycardia 2. Hypocalcemiaor hypoglycemia 3. Acute MI 4. Acute myocarditis 5. Cerebrovascular accident 6. Hypertrophic cardiomyopathy 7. Hypothermia

590
Q

Measure of Cardiac thermodilution mechanism
dye method

Inulin method

NH4 method

dilution method

Fick method

A

D) Reason

When asked technique mark dilution technique. When asked law then mark Ficks law.

591
Q

in superior mediastinum, on right side trachea is related to?
vagus nerve

azygus vein

hemi azygus vein

inferior thyroid artery

inferior thyroid vein

A

A) Reason

Relation of Trachea in Superior mediastinum: • Right side: Azygos vein, right vagus nerve, and pleura.• Left side: Arch of the aorta, left common carotid, left subclavian arteries, left vagus nerve, left phrenic nerve, andpleura.

592
Q

Arterial branch of celiac trunk:
spleenic artery

pancreatic artery

right gastric artery

cystic artery

hepatic artery

A

E) Reason

Celiac Artery (Trunk):The celiac artery is a short, large artery that arises from thefront of the abdominal aorta as it emerges through the diaphragm. It has three terminal branches: the leftgastric, the splenic, and the hepatic arteries.

593
Q

In a liver transplant pt, Drug causing neurotoxicity-nephrotoxicity but doesnt cause bone marrow suppression.
cyclophosphamide

6-MP

Cyclosporine

Methotrixate

Ethambutol

A

C) Reason

MoA: Calcineurin inhibitor; binds cyclophilin. Blocks T cell activation by preventing IL-2 transcription. USES: Transplant rejection prophylaxis, psoriasis, rheumatoid arthritis. TOXICITY: Nephrotoxicity,hypertension, hyperlipidemia, hyperglycemia, tremor, hirsutism, gingival hyperplasia.

594
Q

Folia are present in:
corpus callosum

basal ganglia

cerebellum

vermis

pons

A

C) Reason

The cerebellum consists of a midline vermis and 2 lateral cerebellar hemispheres. The cerebellar cortex consists of multiple parallel folds that are referred to as folia. The cerebellar cortex contains several maps of the skeletal muscles in the body

595
Q

A 42 year old female patient with septic shock and bruises and d-dimer were raised possible cause..?
vWB disease

septic shock

Hypercoagulable state

thrombosis

DIC

A

E) Reason

DIC: Widespread activation of clotting leads to a deficiency in clotting factors, which creates a bleeding state.Causes: Sepsis (gram-negative), Trauma, Obstetric complications, acute Pancreatitis, Malignancy, Nephrotic syndrome, Transfusion (STOP Making New Thrombi).Labs: schistocytes, increased fibrin split products (d-dimers), decreased fibrinogen, decreased factors V and VIII.

596
Q

To correct hypokalemia, IV K is given. K enters the cell through:
H/K exchange

Na/K ATPase

K/Cl exchange

K/glucose symport

K/HCO symport

A

B) Reason

Na/K ATPase (or Na+/K+ pump) in cell membranes transports Na+ from intracellular to extracellular fluid and K+ from extracellular to intracellular fluid; it maintains low intracellular [Na+] and high intracellular [K+].

597
Q

which vasoactive amines are demolished at lungs:
histamine

Angiotensin

epinephrine

dopamine

serotonin

A

E) Reason

Serotonin (5-hydroxytryptamine)■ causes arteriolar constriction and is released in response to blood vessel damage to help prevent blood loss.It is destroyed at lungs.■ has been implicated in the vascular spasms of migraine headaches.

598
Q

What causes Plateau of action potential?
Ca in

K out

Ca in Na out

Ca in K out

Ca in Cl out

A

D) Reason

Cardiac muscle action potential has a plateau, which is due to Ca2+ influx and K+ efflux; myocyte contraction occurs due to Ca2+-induced Ca2+ release from the sarcoplasmic reticulum

599
Q

dermatome from lower end of clavicle to sternal angle:
C1

C2

C3

T1

T2

A

D) Reason

Dermatomes: C2—posterior half of a skull “cap.”C3—high turtleneck shirt.C4—low-collar shirt.T1–Lower end of clavicle to sternumT4—at the nipple.T7—at the xiphoid process.T10—at the umbilicus (important for early appendicitis pain referral).L1—at the inguinal ligament.L4—includes the kneecaps.S2, S3, S4—erection and sensation of penile and anal zones.

600
Q

2 x 2 table is used for
standard deviation

Sensitivity

specificity

positive productive value

chi square test

A

E) Reason

Chi-square (χ²) Checks difference between 2 or more percentages or proportions of categoricaloutcomes (not mean values). It uses 2 x 2 table.

601
Q

Patient with decrease salivation submandibular damage. Which nerve damage?
maxillary nerve

mandibular nerve

CN V

CN VI

CN VII

A

E) Reason

The submandibular ganglion is a parasympathetic ganglionthat lies deep to the submandibular salivary gland and is attached to the lingual nerve by small nerves. Preganglionicparasympathetic fibers reach the ganglion from the facialnerve via the chorda tympani and the lingual nerves.Postganglionic secretomotor fibers pass to the submandibular and the sublingual salivary glands.

602
Q

A pt. undergoes gastric antral removal. what deficiency will occur eventually
rapid emptying

dec. gastric complience

delayed emptying

malabsorption

Dumping syndrome

A

B) Reason

Pyloric antrum: This extends from the incisura angularisto the pylorus. Its removal can lead to decreased complience of the stomach.

603
Q

Consequences of deudenal resection
malnutrition

delayed emptying

GERD

Ulcerations

Rapid stomach emptying

A

E) Reason

The duodenum begins at the pyloric sphincter ofthe stomach, and it ends by becoming continuous with the jejunum. It smoothens the gastric emptying by sphincter. It removal as in cases of Whipple Procedure, can lead to rapid gastric emptying.

604
Q

Chlorpromazine side effect:
depression

psychosis

rigidity

heat intolerance

dystonia

A

E) Reason

Drugs that block the activity ofdopaminergic pathways include older antipsychotics (eg, chlorpromazine, haloperidol), which maycause dystonia and parkinsonian symptoms.

605
Q

atmospheric pressure is 700mmhg, what will be gases content
PaO² 47, CO² 53

PaO² 153, paCO² 147

PaO2 will be 147 and Nitrogen will be 553 mmHg

PaO² 147, pa N 153

PaO² 147, Pa N 253

A

C) Reason

O2 percentage in air is 21, so 700 x 21/100=147Nitrogen percentage is 79, so 700 x 79/100=553

606
Q

Thiazide causing hypotension by:
dec. Na

inc. Na
dec. K
inc. H²O
inc. renin

A

A) Reason

MECHANISM Thiazide diuretic. Inhibits NaCl reabsorption in early distal tubule, decreased diluting capacity of the nephron. Decreased Ca2+ excretion.

607
Q

Most potent antioxidant:
vit. D

B3

B6

vit. C
vit. E

A

E) Reason

Remember the sequence Glutathione is most potent anti oxident if given in options then Vit. E (correct here) and then Vit. C

608
Q

Cushing triad
inc. ICP, tacchycardia, hypotension

HTN, dec. Na & inc. K

hyperpigmentation, HTN, dec. HR

Inc ICP, HTN, Bradycardia

hypoglycemia, HTN, brasycardja

A

D) Reason

FINDINGS Hypertension, weight gain, moon facies, truncal obesity B, buffalo hump, hyperglycemia (insulin resistance), skin changes (thinning, striae), osteoporosis, amenorrhea, and immune suppression.

609
Q

Man with gunshot wound in spine at L1 has Loss of pain and temp of right Side of lower limb. Which pathway has been damaged?
right cuneatus

left cuneatus

right gracilis

medial laminiscus

Left spinothalmic tract

A

E) Reason

pain and temperature sensation is carried by opposite side spinothalmic tract at L1 level.

610
Q

Non consanguineous marriage, child was born who was short stature due to short limbs, trunk was of normal size. What can be the mode of inheritance:
X-linked dominant

X-linked recessive

autosomal recessive

autosomal dominant

Y-linked

A

D) Reason

Pseudohypoparathyroidism (Albright hereditary osteodystrophy)—autosomal dominant unresponsiveness of kidney to PTH. Hypocalcemia, shortened 4th/5th digits, short stature.

611
Q

A 2 year old baby with sickle cell anemia with Hb 6 and mild jaundice, cause is
Hemolytic crisis

hypoxic crisis

recent infection

inc. reticulocytes
dec. reticulocytes

A

A) Reason

Complications in sickle cell disease (SS):1. Aplastic crisis (due to parvovirus B19).2. Autosplenectomy (Howell-Jolly bodies) 3. risk of infection with encapsulated organisms; early splenic dysfunction occurs in childhood.4. Splenic sequestration crisis.5. Salmonella osteomyelitis.

612
Q

Basophilia of cell is due to:
Golgi appratus

RER

SER

nucleus

ribosomes

A

E) Reason

Hematoxylin is a blue dye which stains basophilic substrates that are the acidic cellular components such as DNA, RNA and ribosomes. Hematoxylin stains nuclei blue, and may tint the cytoplasm of cells with extensive mRNA in their cytoplasm.

613
Q

wasting of thenar eminence, nerve responsible?
ulnar

radial

median

axillary

musculocutaneous

A

C) Reason

Median nerve lesions commonly result from supracondylar fractures of the humerus and from wounds just proximal to the flexor retinaculum. Among the clinical signs, the muscles of the thenar eminence areparalyzed and wasted so that the eminence is flattened and the thumb is laterally rotated and adducted. The hand looks apelike.

614
Q

Policeman got shot and lost knee jerk reflex where will be the lesion:
L3

L4

L5

L2-3

L4-5

A

B) Reason

Values of the different deep tendon reflexes are as: Bicep C5-6, Tricep C6-7, Jaw jerk CN V, and knee jerk as L4 and ankle jerk as S1.

615
Q

A 32 year old female diabetic pt, draining abscess from jaw. What is the finding on microscopy?
Granuloma with abcess

granuloma

abscess

Fibrinoid necrosis

caseous necrosis

A

A) Reason

Mononuclear cell and fibroblast mediated; characterized by persistent destruction and repair. Associated with blood vessel proliferation, fibrosis. Granuloma: nodular collections of epithelioid macrophages and giant cells. Acute on chronic inflamation can lead to abcess formation.

616
Q

Upper slips of external oblique muscle interdigitate with:
serratus posterior

serratus anterior

latismus dorsi

internal oblique

facia transversalis

A

B) Reason

The upper slips of muscle merge and fuse with the slips of Serratus anterior. The inferior slips interdigitate with Latissimus dorsi.

617
Q

A 5 years old girl with Hx of failure to thrive, anemia. Hb electrophoresis showed maximum amount of 2 alpha and 2 gamma chains. Diagnosis?
HbA²

HbF

HbA

HbS

M-spike

A

B) Reason

Newborns are initially asymptomatic because of Increased HbF and decreased HbS.Heterozygotes (sickle cell trait) have resistance to malaria.

618
Q

A 52 Year old man got gunshot in thigh, after few days he developed hypotension. On examination of wound found foul smelling greenish discharge, what is your dx:
DIC

hypovolumic shock

Toxemic shock

anaphylactic shock

neurogenic shock

A

C) Reason

Obligate anaerobes give foul smelling discharge, Examples include Clostridium, Bacteroides, and Actinomyces. They lack catalase and/or superoxide dismutase and are thus susceptible to oxidative damage. Generally foul smelling (short-chain fatty acids), are difficult to culture, and produce gas in tissue (CO2 and H2).

619
Q

Aspirin overdose side effect
acidosis

Alkalosis

Thrombocytopenia

decreased BT

decreased PT

A

B) Reason

TOXICITY Gastric ulceration, tinnitus (CN VIII). Chronic use can lead to acute renal failure, interstitial nephritis, and upper GI bleeding. Reye syndrome in children with viral infection. Overdose causes respiratory alkalosis initially, which is then superimposed by metabolic acidosis.

620
Q

Artery contributing in formation of circle of Willis except
Basilar artery

spinal artery

anterior communicating artery

anterior cerebral

posterior cerebral

A

B) Reason

Arteries contributing to Circle of Willis are: anterior communicating,anterior cerebral, internal carotid, posterior communicating, posterior cerebral, and basilar (formed by the junctionof the two vertebral arteries).Cortical and central branches arise from thecircle and supply the brain.

621
Q

Triad microtubules are present in:
Centrioles

cilia

flagella

spindle

dynein

A

A) Reason

Centrioles are the eukaryotic organelles that are composed of 9 circular arrangement of clusters of triad of microtubules and function as mitotic spindle formation

622
Q

A patient gone for colostomy & was on parentral nutrition. What metabolic abnormality will be present:
hypernatremia

hypokalemia

hyperglycemia

hypoglycemia

hypomagnesemia

A

C) Reason

Complications of total parenteral nutrition (TPN) include sepsis, thrombosis, hyponatraemia, hyperglycaemia and liver damage. To minimize sepsis, the central venous catheter is tunnelled with a subcutaneous Dacron cuff at the exit site to reduce the risk of line infection.

623
Q

Fibrocartilage contain which fibers
type 1 collagen

Reticular Fiber

fibronectin

type 3 collegen

type 5 collegen

A

A) Reason

Fibrocartilage has type I collagen in addition to type II, giving it greater resistance to being stretched (tensile strength), and is the type of cartilage found in intervertebral disks of the vertebral column and the menisci of the knee, and may form the attachment of ligaments and tendons to bone

624
Q

Conduction system of heart is found in
pericardium

myocardium

epicardium

endocardium

sub endocardium

A

E) Reason

The cardiac conduction system located subendocardially is a specialized group of myocardial cells that initiates the periodic contractions of the heart due to their ability to depolarize at a faster rate than other cardiac myocytes.

625
Q

sympathetic supply of Lower respiratory bronchioles
T1-2

T3-4

T2-3

T4-5

T6

A

B) Reason

Each lung is supplied by the pulmonary plexus. Theplexus is formed by branches of the sympathetic trunk (T3-T4) andreceives parasympathetic fibers from the vagus nerve (CN X).

626
Q

A normal diet should contain protein up to:
15-20%

20-25%

25-40%

10-15%

0.4

A

C) Reason

Normal percentage of the major nutrients in a normal diet are Carbohydrates 45-65 %, Proteins 25-40% , Lipid as 15-20%.

627
Q

Drug that acts on AV node?
amlodipine

propranolol

carvedilol

Verapamil

metoprolol

A

D) Reason

MECHANISM: Block voltage-dependent L-type calcium channels of cardiac and smooth muscle, thereby reduce muscle contractility.Vascular smooth muscle—amlodipine = nifedipine > diltiazem > verapamil.Heart—verapamil > diltiazem > amlodipine = nifedipine (verapamil = ventricle).

628
Q

If Heart rate is doubled cardiac cycle duration will become?
0.8 sec

  1. 6 sec
  2. 2 sec
  3. 4 sec
  4. 6 sec
A

D) Reason

Cardiac Cycle= 1 minute/ number of beats= 60 sec/ 72= 0.8 sec. As heart rate doubles the cycle will become half e.g: cardiac cycle= 60/144= 0.4 sec.

629
Q

Which parameter doesn’t change in restrictive lung disease:
FEV1

FEV1/RV

FEV1/FEV

FEV1/ FRC

FEV1/FVC

A

E) Reason

in restrictive lung disease FEV1/FVC can be increased or normal.

630
Q

Botulinum toxin cause
toxic shock

necrosis

muscular weakness

respiratory muscle paralysis

muscle rigidity

A

D) Reason

Flaccid paralysis, floppy baby; toxin prevents release of stimulatory (ACh) signals at neuromuscular junctions leading to flaccid paralysis and can cause respiratory muscle paralysis.

631
Q

Stage of Theta waves on EEG?
REM sleep

NON-REM

Full wake state

REM 1st stage

REM 2nd stage

A

D) Reason

  1. Awake (eyes open) Alert, active mental concentration Beta (highest frequency, lowest amplitude)2. Awake (eyes closed) Alpha3. Non-REM sleepStage N1 (5%) Light sleep- theta waves
632
Q

A 32 old female with low grade fever, hypercalcemia stones with B/L lymphadenopathy\ histopathological findings will be?
Granuloma

Granuloma with asteroid bodies

abscess

Fibrinoid

Cassius

A

B) Reason

Sarcoidosis is Associated with restrictive lung disease (interstitial fibrosis), erythema nodosum, lupus pernio, Bell palsy, epithelioid granulomas containing microscopic Schaumann and asteroid bodies, uveitis, and hypercalcemia (due to increased 1α hydroxylase–mediated vitamin D activation in macrophages). Treatment: steroids.

633
Q

Function of Pneumotaxic center
control apnea

cause apnea

control respiration

control expiration

limit inspiration

A

E) Reason

A Pneumotaxic Center Limits theDuration of Inspiration and Increasesthe Respiratory Rate: A pneumotaxic center, located dorsally in the nucleusparabrachialis of the upper pons, transmits signals tothe inspiratory area.

634
Q

A player has lateral malleolar fracture, what other findings should be present in this case?
loss of dorsal sensation of foot

loss of toe sensation

loss of ankle jerk

loss of sole sensation

upgoing planters

A

A) Reason

The sensory supply from most of the dorsal surface of the foot is provided by the superficial fibular nerve, except between the great and second toes, which is supplied by the deep fibular nerve.

635
Q

In diarrhea metabolic acidosis and hypokalemia are observed, from where fluid loss is happening
Colon

ilium

rectum

jejunum

dudenum

A

A) Reason

Water and electrolytes are reabsorbed in large intestine esp. the colon. When there is any abnormality of this absorption mechanism, it leads to dirrohea and electrolytes loss

636
Q

If Posterior cord of brachial plexus is damaged it will result in loss of action of:
teres major

teres minor

tricep

bicep

Deltoid

A

E) Reason

Posterior cord: Upper subscapular nerveThoracodorsal nerve, Lower subscapular nerveAxillary nerve (C5, 6)..supplies deltoid and teres minor muscles.Radial nerve (C5, 6, 7, 8; T1)

637
Q

AV node is supplied by
LCA

RCA

LXA

LAD

Posterior Interventricular artery

A

B) Reason

SA and AV nodes are usually supplied by RCA. Infarct may cause nodal dysfunction (bradycardia or heart block)

638
Q

Old age bed ridden male pt having c/o burning micturition. Urine sample was taken for culture, on microscopy there was greenish growth on culture, what drug will you prescribe him?
ampicillin

ciprofloxacin

metronidazol

moxifloxacin

albendazole

A

B) Reason

Clinical Uses of Fluroquinolones: Gram-negative rods of urinary and GI tracts (including Pseudomonas), especially Ciprofloxacin, Neisseria, some gram-positive organisms.

639
Q

after 2 hours of surgery pt. developed hypotension, tachycardia and abdominal distension, due to?
sepsis

wound leakage

Hypovolumia

wound dehiscence

stress response

A

C) Reason

Abdominal distention might be due to intra-abdominal bleed after surgery leading to hypovolemic shock. Moreover, sepsis takes time.

640
Q

A 35 years old woman patient with raised BP and erythropoietin was decreased creatinine raised , possible site of defect
interstitium

nephron

collecting tubules

glumerulus

JG cells

A

E) Reason

JG cells (modified smooth muscle of afferent arteriole) and the macula densa (NaCl sensor, part of the distal convoluted tubule). JG cells secrete renin in response to decreased renal blood pressure, decreases NaCl delivery to distal tubule, and increases sympathetic tone (β1).

641
Q

Distinctive character of olfaction:
cells are replaced continually

No true neurons

permanent cells

mixed nerve

without epithelium

A

A) Reason

The olfactory nerves arise from olfactory nerve cells in theupper part of the nasal mucous membrane that is replaced continually.

642
Q

In hypoxia, mechanism of cellular swelling?
inc. water entry from surrounding

inc. entry from lymph
inc. entry from tissues
dec. drainage to lymph

increase entry from plasma

A

A) Reason

Decreased cardiac output leads to Hypoxia. This supplement the baroreceptor mechanism by increasing sympathetic outflow to the heart and blood vessels. This increased outflow causes cellular edema.

643
Q

What is secreted by Outflow tract of basal ganglia?
Glycine

glutamate

Aspartate

lysine

GABA

A

E) Reason

Excitatory pathway—cortical inputs stimulate the striatum, stimulating the release of GABA, which disinhibits the thalamus via the GPi/SNr (increasess motion).

644
Q

In a young female with 3rd or 2nd degree burn pt was in emergency, due to electrolyte imbalance what could cause low grade voltage potential:
hypokalemi

hylercalcemia

hyponatremia

hyperglycemia

hypermagnecemia

A

C) Reason

Causes of Low voltage QRS (less than 5 mm in leads I, II, III and <10 mm in chest leads):1. Improper standardization2. Obesity or thick chest wall3. Pericardial effusion4. Emphysema5. Chronic constrictive pericarditis6. Hypothyroidism7. Hypothermia

645
Q

Renal plasma flow in kidney is calculated by?
PAH

Inulin

PABA

Creatinine

urea

A

A) Reason

Measurement of renal plasma flow (RPF)—clearance of para-aminohippuric acid (PAH)■ PAH is filtered and secreted by the renal tubules.■ Clearance of PAH is used to measure RPF.

646
Q

Nutmeg liver, site of congestion
portal congestion

periportal congestion

periportal fibrosis

sinosidal congestion

centrilobular

A

E) Reason

Nutmeg liver Due to backup of blood into liver. Commonly caused by right-sided heart failure and Budd-Chiari syndrome. The liver appears mottled like a nutmeg. If the condition persists, centrilobular congestion and necrosis can result in cardiac cirrhosis

647
Q

Which of the following has maxmimum value upon standing at the apex of lung?
inc. V/Q value

dec. V/Q value

increased Ventilation

decreased perfusion

increased profusion

A

A) Reason

Ideally, ventilation is matched to perfusion (i.e., V/Q = 1) in order for adequate gas exchange.Lung zones:1. Apex of the lung—V/Q = 3 (wasted ventilation)2. Base of the lung—V/Q = 0.6 (wasted perfusion)Both ventilation and perfusion are greater at the base of the lung than at the apex of the lung

648
Q

Prominent feature of malignancy
pleumorphism

anaplasia

anrchy

metastasis

dysplasia

A

D) Reason

Neoplastic progression Hallmarks of cancer—evasion of apoptosis, growth signal self-sufficiency, anti-growth signal insensitivity, sustained angiogenesis, limitless replicative potential, tissue invasion, and metastasis. Metastasis is especially prominent in malignant lesions.

649
Q

Serous acinus gland, interlobular septa, connective tissue and striated ducts are feature of
Submandibular

Sublingual

minor salivary gland

Parotid

pancreatic acini

A

D) Reason

The parotids are entirely serous salivary glands with septa and striated ducts that drain inside each cheek through Stensen”s ducts which open above the second upper molar tooth

650
Q

Cardiac muscle work as syncytium due to:
SA node

AV node

gap junction

tight junction

hemidesmosomes

A

C) Reason

Gap junctions:■ are present at the intercalated disks.■ are low-resistance paths between cells that allow for rapid electrical spread of action potentials.■ account for the observation that the heart behaves as an electrical syncytium.

651
Q

which clinical feature helps in diagnosing cardiac temponade?
pulsus alternans

Pulsus paradox

water hammer pulse

hypotension

jerky pulse

A

B) Reason

Pulsus paradoxus decrease in amplitude of systolic blood pressure by ≥ 10 mmHg during inspiration. Seen in cardiac tamponade, asthma, obstructive sleep apnea, pericarditis, and croup.

652
Q

Propanalol acts mainly on:
alpha 1 receptor

alpha 2 receptor

beta 1 receptor

beta 2 receptor

nicotinic receptor

A

C) Reason

All of the β blockers used clinically are competitive pharmacologic antagonists. Propranolol is theprototype. Drugs in this group are usually classified into subgroups on the basis of 1 selectivity.

653
Q

How does insulin act on adipose tissue:
sec. lipoprotein lipase

inc. lipoprotein lipase
inc. glucose uptake
dec. glucose uptake

no effect

A

B) Reason

Insulin-dependent glucose transporters:GLUT-4: adipose tissue (increases lipoprotein lipase) and skeletal muscle.

654
Q

Nor Epinephrine is given to increase heart rate. It will act on which receptors?
beta 1 receptor

beta 2 receptor

alpha 1 receptor

alpha 2 receptor

central acting

A

A) Reason

Norepinephrine effects are α1 > α2 > β1 ( β1 increase heart rate, increase contractility, increase renin release, increase lipolysis) Clinical use are Hypotension (but decreases renal perfusion)

655
Q

mechanism of CO2 transport from tissues to lungs
uptake of HCO3

exchange of K

uptake of HCO3 in exchange of Cl

uptake of HCO3 in exchange of K

uptake of K in exchange forCl

A

C) Reason

HCO3- leaves the RBCs in exchange for Cl- (chloride shift) and i is transported to the lungs in the plasma. HCO3- is the major form in which CO2 is transported to the lungs.

656
Q

If needle is passed just 2 cm below and lateral to pubic tubercle which nerve will be pierced?
tibial nerve

peroneal

femoral

Obturator

sciatic

A

D) Reason

The posterior division descends through the obturatorexternus muscle and passes behind the adductor brevis andin front of the adductor magnus muscles

657
Q

A 46 year old female was operated, hysterectomy was done, after week she developed SOB and hemoptysis, her calf muscles are tender, what can be possible dx:
Pulmonary embolism

fat embolism

DIC

ARDS

toxic shock

A

A) Reason

Pulmonary embolism: V/Q mismatch leading to hypoxemia and cause respiratory alkalosis. Sudden-onset dyspnea, chest pain, tachypnea. May present as sudden death

658
Q

Supply of Upper 1/3rd of esophagus
superior thyroid artery

middle thyroid artery

inferior thyroid artery

bracheocephalic trunk

internal carotid

A

C) Reason

• The inferior thyroid artery ascends to reach the posterior surface of the thyroid gland, where it is closely relatedto the recurrent laryngeal nerve. The inferior thyroidartery supplies the thyroid, upper 1/3rd of esophagus and the inferior parathyroidglands.

659
Q

2ndary peristalsis of esophagus is caused by?
distention of esophagus

by gravity

by food particles

by saliva

by acrdiac sphincter

A

A) Reason

A primary peristaltic contraction creates an area of high pressure behind the food bolus. The peristaltic contraction moves down the esophagus and propels the food bolus along. Gravity accelerates the movement.d. A secondary peristaltic contraction by the dilatation of esophagus clears the esophagus of any remaining food.

660
Q

A 6 year old young boy developed shortness of breath during PT class. He has 2-3 weeks back previous history of sore throat and joint pain; what test will you advise him:
Echocardiography

Chest X-ray

Blood culture

ECG

ASO titer

A

E) Reason

Associated with Aschoff bodies (granuloma with giant cells [blue arrows in ]), Anitschkow cells (enlarged macrophages with ovoid, wavy, rod-like nucleus [red arrow in A]), increased ASO titers. Immune mediated (type II hypersensitivity); not a direct effect of bacteria. Antibodies to M protein cross-react with self antigens.

661
Q

What drug Enhance alpha 2 responses at presynaptic cleft
Methyldopa

amolodipine

Clonidine

prazosin

Timolol

A

C) Reason

Clonidine MoA: It enhance alpha 2 responses at presynaptic cleft. Clinical: Hypertensive urgency (limited situations); does not decrease renal blood flowADHD, severe pain, and a variety of off label indications (e.g., ethanol and opioid withdrawal)

662
Q

In a 36 years married women cervical dysplasia is caused by:
nulliparity

dysplasia

multiparty

Metaplasia

atrophy

A

C) Reason

Risk factors for Cervical dysplasia : multiple sexual partners (#1), smoking, early sexual intercourse, multiparty, HIV infection.

663
Q

A 5 year old child having 6 g/dl proteinuria where will be the defect:
Basement membrane

interstium

nephron

tubules

glomerulus

A

A) Reason

Nephrotic syndrome includes a group of conditions characterized by increased basement membrane permeability, permitting the urinary loss of plasma proteins, particularly low weight proteins, such as albumin.

664
Q

Glans penis is drained by?
superficial inguinal

deep inguinal

inferior hypogastric

internal illeic

external iliec

A

B) Reason

The glans penis drains into the deep inguinal and the external iliac nodes. The skin of the remainder of the organ drains into the superficial inguinal nodes. The erectile tissuedrains into the internal iliac nodes.

665
Q

which component of blood recovers last after massive hemorrhage?
Heart rate

Blood pressure

blood volume

CBC

LFTs

A

D) Reason

After blood loss, fisrt of all, the hemodynamics are corrected due to fast and slow acting mechanisms. Hematopoisis takes larger time and hence CBC is corrected last.

666
Q

Pt is having nystagmus where will be the lesion:
vermis

striatum

floculonodular

cerebellar peduncles

cerebellar cortex

A

C) Reason

Medial lesions—Lesions involving midline structures (vermal cortex, fastigial nuclei) and/or the flocculonodular lobe result in truncal ataxia, nystagmus, and head tilting. These patients also may have a wide-based (cerebellar) gait and deficits in truncal coordination.

667
Q

Muscle causing heat released from contraction?
radiant heat

contraction heat

initial heat

secondary heat

tertiary heat

A

C) Reason

Shivering is the most potent mechanism for increasing heat production. ■ Cold temperatures activate the shivering response (initial heat production), which is orchestrated by the posterior hypothalamus.

668
Q

Which drug is given in Pregnancy in a patient with Graves disease
Propylthiouracil

propranolol

iodine

radiation

chemotherapy

A

A) Reason

CLINICAL USE: Hyperthyroidism. PTU blocks Peripheral conversion, used in Pregnancy because it can”t cross placenta.

669
Q

Chondrocytes present in?
immature cartilage

mature cartilage

bone

epiphyseal plate

metaphysis

A

B) Reason

Chondroblasts (cartilage forming) and chondrocytes (mature cartilage) produce the type II collagen in cartilage. Cartilage covers the articular surfaces of bones, and is found in the ears, nose, larynx, and tracheobronchial tree.

670
Q

A 47 years old man with Chronic hepatitis C microscopic finding?
cirrhosis

periportal congestion

portal congestion

fibrosis

portal hypertension

A

D) Reason

Cirrhosis—diffuse fibrosis and nodular regeneration destroys normal architecture of liver , risk for hepatocellular carcinoma (HCC).

671
Q

A pt with MCV 86fL, HB: 7.6g/dl, malabsorption with change in RBC shape and size. The area affected in GIT Is?
duodenum

jejunum

iIium

colon

cecum

A

C) Reason

Intrinsic factor is Vitamin B12–binding protein (required for B12uptake in terminal ileum). Damage to Intrinsic factor or malabsorption due ilial defects causes pernicious anemia.

672
Q

A 35 years old man with Hodgkin lymphoma develops fever headache and neck rigidity responsible organism?
Mycobacterium

Pseudomonas

cryptococcus

S. aureus

Miningococcus

A

C) Reason

Cryptococcus neoformans:Cryptococcal meningitis (sometimes associated with lymphoma), cryptococcosis. Heavily encapsulated yeast. Not dimorphic. Found in soil, pigeon droppings. Acquired through inhalation with hematogenous dissemination to meninges

673
Q

Benzodiazepine cause significant hypotension if given to?
septic patient

hypovolumic patient

hypertensive

diabetic

COPD patient

A

B) Reason

High doses of conventional sedative-hypnotics, especially alcohols and barbiturates, can causedepression of medullary neurons in deblitated or hypovolumic patients , leading to respiratory arrest, hypotension, and cardiovascularcollapse. These effects are the cause of death in suicidal overdose

674
Q

Tabes dorsalis causes?
atonic bladder

autonomous bladder

UV prolapse

tumling gait

loss of vibration

A

A) Reason

Associated with Charcot joints, shooting pain, atonic bladder, Argyll Robertson pupils (small bilateral pupils that further constrict to accommodation and convergence, not to light)

675
Q

Relation of steady pressure ?
Pacinian

free nerve endings

Ruffni”s

tactile

murckle discs

A

C) Reason

Ruffini corpuscle are the Encapsulated receptors that carry Pressure sensations and are Slowly adapting receptors.

676
Q

Patient post GA in recovery room stops breathing and is cyanosed. What drug caused this?
halothane

ketamine

pancurium

isoflurane

fentinyl

A

C) Reason

The action of full doses of crurae NM blockers leads directly to respiratory paralysis. Ifmechanical ventilation is not provided, the patient will asphyxiate.

677
Q

In a young female, Most probable cause of negative Montoux test?
prior exposure

acute infection

chronic infection

past TB

immunosuppressant

A

E) Reason

PPD positive: if current infection, past exposure, or BCG vaccinated.PPD negative: if no infection or anergic (steroids, malnutrition, immunocompromise) and in sarcoidosis.

678
Q

Free end of small intestine lies in?
left upper quadrant

right upper quadrant

Righ iliac fossa

left iliac fossa

bellow diaphragm

A

B) Reason

Small intestine is attached to the mesentery of the small intestine and is placed at its place exept the right hypochondriac region where it is free.

679
Q

In which of the following maximum chemotaxis occur?
acute inflammation

chronic inflammation

granuloma

bleeding

wound dehisence

A

A) Reason

Acute inflammation is rapid onset (seconds to minutes), lasts minutes to days by the chemotaxis of the acute inflammatory cells to the site. Outcomes include complete resolution, abscess formation, andprogression to chronic inflammation

680
Q

prime key to measure GFR?
Creatinine

urea

PAH

Inulin

PABA

A

D) Reason

Best way to measure GFR =inulin ClearanceBest way to measured GFR clinically by =creatinineBest way to estimate GFR = creatinine clearance

681
Q

During male gametogenesis, what inhibits FSH?
LH

Progesterone

Aldosterone

estrogen

inhibin

A

E) Reason

Sertoli cells (non–germ cells):Secrete inhibin that inhibit FSHSecrete androgen-binding protein that maintain local levels of testosterone.

682
Q

Pt with lower backache pain radiating to back of thigh cause?
lesion at L5

lesion at L5-S1

lesion at S1-2

lesion at L4-5

lesion at L3-4

A

B) Reason

Herniation usually occurs in the lower lumbar (L4/LS or LS/Sl) when pain radiates to posterior of thigh. or lower cervical (CS/C6 or C6/C7) when pain radiates to arm parts of the vertebral column.

683
Q

main difference b/w ECF & ICF?
water concentration

organic cation

inorganic cation

organic anion

inorganic anions

A

E) Reason

The major anions of ICF are protein and organic phosphates. While anions of the ECF are the inorganic such as Cl and HCO3.

684
Q

Mannitol given with caution in?
CHF

MI

CLD

CKD

cirrhosis

A

A) Reason

In a patient with heart failure, Mannitol should be given with caution because of risk of precipitating pulmonary edema

685
Q

Association of blood transfusion reaction?
hemoglobinurea

Myoglobinurea

urticaria

WBC casts

RBC casts

A

A) Reason

Blood transfusion risks include infection transmission (low), transfusion reactions, iron overload, hypocalcemia (citrate is a calcium chelator), and hyperkalemia (RBCs may lyse in old blood units leading to hemoglobinurea and hypokalemia)

686
Q

Chief inhibitory hormone from hypothalamus is?
ACTH

TRH

TSH

Prolactin

Dopamine

A

E) Reason

Prolactin is regulated by Dopamine and TRH. Prolactin in turn inhibits its own secretion by increased dopamine synthesis and secretion from hypothalamus.

687
Q

Which cell type decreases in HIV patient commonly?
CD4-Th2

CD8

CD4-Th1

CD8-Th1

CD8-Th2

A

C) Reason

AIDS diagnosis ≤ 200 CD4+ cells/mm3(normal: 500–1500 cells/mm3). HIV-positive with AIDS-defining condition (e.g., Pneumocystis pneumonia) or CD4 percentage < 14%.

688
Q

Radiation cause damage via?
cellular protein

cellular lipid

cellular carbohydrates

chromosome

cell wall

A

A) Reason

Ionizing radiation is a classic cause of chronic inflammation and cancer as these damage geneetic material and cellular proteins, as exemplified by the increased incidence of cancers in those exposed to radiation.

689
Q

Polyp in Endocervix shows stratified squamous epithelia?
Dysplasia

Metaplasia

hyperplasia

hypertrophy

anaplasia

A

A) Reason

Dysplasia and carcinoma in situ:Disordered epithelial growth; begins at basal layer of squamocolumnar junction (transition zone) and extends outward. Classified as CIN 1, CIN 2, or CIN 3 (severe dysplasia or carcinoma in situ), depending on extent of dysplasia.

690
Q

Automatic bladder caused?
at S5

lesion above S1

at S4

at S4-5

at S3

A

B) Reason

Atonic bladder results from lesions to the sacral spinal cord segments or the sacral spinal nerve roots. Loss of pelvic splanchnic motor innervation with loss of contraction of the detrusor muscle results in a full bladder with a continuous d ribble of urine from the bladder.

691
Q

After a few days of typhoid, the bacili reside in?
spleen

payers patches

liver

ilium

jejunum

A

B) Reason

In typhoid and other enteric fevers, infection begins in the small intestine, but few gastrointestinal symptoms occur. Then after few days organisms enter, multiply in themononuclear phagocytes of Peyer’s patches, and then spread to the phagocytes of theliver, gallbladder, and spleen

692
Q

stress hormone of utmost value?
glucagon

cortisol

GH

insulin

FSH

A

B) Reason

FUNCTION: It is an important stress hormone and functions to increase Blood pressure (upregulates α1-receptors on arterioles and increases sensitivity to norepinephrine and epinephrineIncreases nsulin resistance (diabetogenic)Increases luconeogenesis, lipolysis, and proteolysisDecreases Fibroblast activity (causes striae)Decreases Inflammatory and Immune responses

693
Q

Sinus venosum develops into?
AV node

coronary sulcus

coronary sinus

pulmonary trunk

aorta

A

C) Reason

The sinus venosus at the venous end of the heart tube will develop into the coronary sinus and the smooth part of the right atrium.

694
Q

Food poisoning in 10 people, it is
infectivity

infection rate

poisoning rate

virulence

infectious rate

A

D) Reason

Virulence is a quantitative measure of pathogenicity and is measured by thenumber of organisms required to cause disease. As many people are affected, means more virulent is the organism or its poison.

695
Q

Crus cerebri closely related to?
cerebellum

striatum

substantia nigra

globus pallidus

uncus

A

C) Reason

The substantia nigra is the largest nucleus of the midbrain. It appears black to dark brown in the freshly cut brain because nigral cells contain melanin pigments. It is closely related to crus cerebri.

696
Q

Antibodies in TB are?
inside cell

inside nucleus

bound to cell

inside Golgi complex

inside vacules

A

C) Reason

Tuberculosis is an example of type 4 delayed HSR and is associated with chronic infection. Its antibodies are attached to the cells.

697
Q

Which organ compensates its blood supply if the blood supply is compromised?
Intestine

liver

spleen

heart

kidney

A

A) Reason

Submucosal plexus (Meissner plexus):■ primarily controls secretion and blood flow by virtue of which it can compensates its blood supply in case of hemmorhage or hypovolumia.

698
Q

thoracic part of esophagus is supplied by?
arch of aorta

left gastric

bronchial artery

ascending aorta

Internal thoracic branch of descending aorta

A

E) Reason

Blood Supplyof the Esophagus:• Upper third: Inferior thyroid artery.• Middle third: Branches from the descending thoracicaorta (internal thoracic artery).• Lower third: Left gastric artery.

699
Q

Standard deviation is calculated along mean because it shows
average

above values

bellow values

central tendency

variability

A

E) Reason

Standard deviation = how much variability exists from the mean in a set of values.Standard error of the mean = an estimation of how much variability exists between the sample mean and the true population mean.

700
Q

Adrenergic response on muscarinic receptors of heart are by…
epinephrine

nicotinic

Norepinephrine

muscrine

Dopamine

A

C) Reason

Norepinephrine released from a sympathetic nerve ending is modulated by norepinephrine itself, acting on presynaptic α2-autoreceptors, angiotensin II, and other substances.

701
Q

1st phase in the CSF production in choroid plexus?
filtration of K

filtration of Na

secretion

protein dilution

ultrafiltration of Na

A

E) Reason

B. Formation of CSF by the choroid plexus epithelium:■ Ultrafiltration of Na ions followed by the Lipid-soluble substances (CO2 and O2) and H2O freely cross the blood–brain barrier and equilibrate between blood and CSF.■ Other substances are transported by carriers in the choroid plexus epithelium. They may be secreted from blood into the CSF or absorbed from the CSF into blood.

702
Q

Highest percentage of protein is in?
VLDL

LDL

HDL

chylomicron

adipose tissue

A

C) Reason

In lipoproteins, as the percentage of lipids increases the percentage of protein decreases and vice versa. So, protein conc. being highest in HDL and lowest in VLDL.

703
Q

Direct activation of immune system by?
bacterial membrane

bacterial coat

Bacterial cell wall CHO

bacterial wall proteins

bacterial capsule

A

C) Reason

Induced to kill when exposed to a nonspecific activation signal on target cell and/or to an absence of class I MHC on target cell surface (carbohydrates on cell wall).

704
Q

continous phase of contraction without relaxing
tetnus

summation

adding

tetany

rhythmicity

A

D) Reason

Sometimes, spontaneous discharge occurs without relaxation in someperipheral nerves, often causing muscle “tetany.”

705
Q

18th month’s baby, surgeon needs to do a tracheostomy which vascular structure should he save from injury?
carotid artery

internal carotid artery

external jugular vein

recurrent laryngeal nerve

internal jugular vein

A

E) Reason

Recurrent laryngeal nerves, inferior thyroid veins, carotid arteries & internal jugular veins could be damaged this is a real risk in obese or paediatric patients.

706
Q

Staging of tumor is based on?
extent of severity

Extent of invasion

anaplasia

metastasis

prognosis

A

B) Reason

Degree of localization/spread based on site and size of 1° lesion, spread to regional lymph nodes, presence of metastases. Based on clinical (c) or pathology (p) findings. Example: cT3N1M0

707
Q

UV light causes?
glaucoma

cataract

sudden vision loss

retinal detachment

conjunctivitis

A

B) Reason

Cataract:Painless, often bilateral, opacification of lens , decreased in vision. Risk factors: increasing age, smoking, EtOH, excessive sunlight, UV light, prolonged corticosteroid use, classic galactosemia, galactokinase deficiency, diabetes (sorbitol), trauma, infection.

708
Q

In a 56 year old male with Colon cancer surgery done, no mets present. Colon cancer marker?
CEA

RET

Rb

CA 19

CA 19-9

A

A) Reason

Diagnosis: Screen patients > 50 years old with colonoscopy or stool occult blood test.“Apple core” lesion seen on barium enema x-ray.CEA tumor marker: good for monitoring recurrence, not useful for screening

709
Q

If parasympathetic is lost which of these will be affected mostly
muscle of GIT

muscle of heart

kidney

liver

suprarenal gland

A

A) Reason

This is a major parasympathetic nucleus of the brain stem, and it supplies preganglionic fibers innervating terminal ganglia in the thorax and the foregut and midgut parts of the gastrointestinal tract. It damage mainly affects the gastrointestinal tract muscles.

710
Q

Phase of Pharyngeal swallowing ?
raising of larynx

pulling larynx

upward palate

down the larynx

close glottis

A

A) Reason

During pharyngeal swallowing larynx and the laryngeal part of the pharynx are pulled upward by the contraction of the stylopharyngeus,the salpingopharyngeus, the thyrohyoid, and the palatopharyngeus muscles.

711
Q

The part of developing long bone which develops after birth from a secondary center of ossification is known as
metaphyseal plate

diaphysis

metaphysis

epiphyseal plate

epiphysis

A

E) Reason

Later in development, secondary (or late) centers of ossification form separately through a process of cartilage hypertrophy, ingrowth of vessels from outside the cartilage template, and replacement of cartilage by new trabecular bone and marrow. This takes place at the ends (epiphyses) of long bones.

712
Q

In Sarcoidosis, Which cell types?
T-cells

B-cells

macrophages

mast cells

eosinoplils

A

A) Reason

Sarcoidosis Characterized by immune-mediated, widespread noncaseating granulomas containing T-cells and elevated serum ACE levels. Common in black females. Often asymptomatic except for enlarged lymph nodes.

713
Q

receptors which adapt readily?
touch receptor

pain receptors

pacinian

murkle disc

Ruffini”s

A

A) Reason

Rapidly adapting, or phasic, receptors (pacinian corpuscle; light touch)■ show a decline in action potential frequency with time in response to a constant stimulus.■ primarily detect onset and offset of a stimulus.

714
Q

Parasympathetic secretomotor fibers to submandibular salivary gland are innervated by?
lacrimal

Cilliary ganglion

auricular

auriculotemporal

corda tympani

A

E) Reason

chorda tympani nerve: it supplies the mucous membrane of the anterior two thirds of the tongue and the floor of the mouth. It also gives off preganglionic parasympatheticsecretomotor fibers to the submandibular ganglion

715
Q

Plasmodium found in which form during the dormant phase:
oozyte

merozoite

haptozoite

haplozoite

hypnozoites

A

E) Reason

P. vivaxand P. ovale produce a latent form (hypnozoite) in the liver; this form is the causeof relapses seen with vivax and ovale malaria.

716
Q

Sudden protection after injury is given by?
blood vessels

skin

barorecptors

renin Angiotensin

sympathetic

A

A) Reason

The compensatory responses to acute blood loss are as follows: First of all vasoconstriction followed by A decrease in blood volume produces a decrease in venous return. As a result, there is a decrease in both cardiac output and arterial pressure

717
Q

Ventricular rate less than atria on ECG. Diagnosis?
atrial fibrillation

WPS syndrome

1st degree block

3rd degree block

2nd degree block

A

E) Reason

Progressive lengthening of the PR interval until a beat is “dropped” so that ventricular rate is less than that of atria (a P wave not followed by a QRS complex).

718
Q

Which of the following remains normal in chronic respiratory acidosis?
plasma H conc.

plasma HCO3

plasma Cl

plasma K

plasma Ca

A

A) Reason

■ In chronic respiratory alkalosis, renal compensation (decreased HCO3− reabsorption) has occurred. Thus, arterial pH is decreased toward normal (i.e., a compensation). Symptoms of hypocalcemia (e.g., tingling, numbness, muscle spasms) may occur because H+ and Ca2+ compete for binding sites on plasma proteins. So, plasma H concentration remains almost unchanged.

719
Q

Plasmodium with longest incubation period?
P. falciparum

M. malariae

P. Ovale

P. vivax

P. Ovax

A

B) Reason

P. malariae infects onlymature red cells; therefore, they produce much lower levels of parasites in theblood and these have the longest incubation period of than other Plasmodium species.

720
Q

Dorsal column medial leminiscus conducts ?
touch

pain

temperature

vibration

pressure

A

D) Reason

Function of Dorsal column-medial lemniscal system is to carryDiscriminative touch, conscious proprioception, vibration, pressure

721
Q

Consequence of decrease carotid sinus pressure ?
inc. heart rate

inc. heart rate
inc. BP
inc. stroke volume
inc. cardiac output

A

A) Reason

Decreased pressure on carotid sinus decreased stretch leads to decreased afferent baroreceptor firing that leads to decreased AV node refractory period and ultimately increases HR.

722
Q

A 4 years old child presented with progressive pallor, HB: 4.9g/dl MCV 56fl, hepatosplenomegaly. Diagnosis?
sickle cell

sideroblastic anemia

Thalasemia major

iron deficiency anemia

Thalasemia minor

A

C) Reason

β-thalassemia major (homozygote):β chain is absent causing severe anemia requiring blood transfusion (2° hemochromatosis).Marrow expansion (“crew cut” on skull x-ray) skeletal deformities. “Chipmunk” facies. Extramedullary hematopoiesis (leads to hepatosplenomegaly).

723
Q

to preserve blood volume what alters in kidney?
inc. Angiotensin

inc. H secretion
inc. ADH
dec. ADH
inc. renin

A

E) Reason

Renin–angiotensin–aldosterone system■ is a slow, hormonal mechanism.■ is used in long-term blood pressure regulation by adjustment of blood volume by increasing Aldosterone and hence plasma volume.■ Renin is an enzyme.■ Angiotensin I is inactive.■ Angiotensin II is physiologically active.

724
Q

Ability of a test to produce repeated results
reliability

sensitivity

specificity

PPV

NPV

A

A) Reason

When the results of a test are same repeatedly, the confidence of the professional and reliability of the test increases.

725
Q

lateral side of wrist is derived from which dermatome
C3

C4

C5

C6

C7

A

D) Reason

Landmark dermatomes C2—posterior half of a skull “cap.”C3—high turtleneck shirt.C4—low-collar shirt. C5–Deltoid, C6– lateral side of wrist

726
Q

Which site must be damaged in brain for short term memory loss?
Uncus

agygdala

Reticular substance

pineal gland

hippocampus

A

E) Reason

Hippocampus: Bilateral lesions lead to inability to consolidate short-term to long-term memory.

727
Q

in mediolateral episiotomy which muscle will be injured
bulbourethral

bulbospongiosus

rectal ani

Perianal membrane

Levator ani

A

B) Reason

Root of the clitoris is attached to the undersurface of the urogenital diaphragm and is covered by the bulbospongiosus muscles. It the same muscle that is cut in mediolateral episiotomy.

728
Q

what must be done to prevent complications of an intervention?
informed consent

non malificence

good communication

beneficence

professional competence

A

E) Reason

In order to have better results and least complications, it is an important aspect to fulfill the best level of professional competence.

729
Q

which of these consists of clara cells
terminal bronchioles

trachea

bronchi

tertiary bronchus

alveoli

A

A) Reason

The epithelial lining of the airway is composed of ciliated cells (goblet and basal cells are absent in the terminal bronchioles) and an additional type called the Clara cell.

730
Q

A 47 years old Pt with HCC has abdominal distension, tumor has spread to
portal vein

spleenic vein

hepatic vein

IVC

gastric vein

A

A) Reason

Findings: jaundice (Hyperbilirubinemia), tender hepatomegaly, ascites (portal hypertension) , and anorexia. Spreads hematogenously.Diagnosis: inc. α-fetoprotein; ultrasound or contrast CT.

731
Q

which mechanism helps in activity of smooth muscles of alveoli?
O2

Ca/H

O2/K

K/Cl

Na/H

A

C) Reason

Lung stretch receptors:■ are located in the smooth muscle of the airways.■ When these receptors are stimulated by differential concentration of alveolar O2 and K and distention of the lungs, they produce a reflex decrease in breathing frequency (Hering–Breuer reflex).

732
Q

Among the following, most common method to detect tumor in blood
staging

grading

receptor status

metastasis

tumor markers

A

E) Reason

Tumor markers should not be used as the 1° tool for cancer diagnosis in blood. They may be used to monitor tumor recurrence and response to therapy, but definitive diagnosis can be made only via biopsy.

733
Q

A 46 year old man Diabetic and HTN patient presents with nose bleed and orbital invasion of cellulitis, organism
Actinomycetes

mucormycosis

Pseudomonas

spirochete

S.aureus

A

B) Reason

Mucor and Rhizopus spp:Mucormycosis. Disease mostly in ketoacidotic diabetic and leukemic patients. Fungi proliferate in blood vessel walls when there is excess ketone and glucose, penetrate cribriform plate, and enter brain. Rhinocerebral,assocaited with frontal lobe abscesses. Headache, facial pain, black necrotic eschar on face; may have cranial nerve involvement.Treatment: amphotericin B.

734
Q

increased H secretion in exchange with Na or K in distal tubules, is the Mechanism of action of which drug
thiazide

loop diuretic

Acetazolamide

amiloride

manitol

A

C) Reason

AcetazolamideMECHANISM Carbonic anhydrase inhibitor. Causes self limited NaHCO3 diuresis and dec. total-body HCO3- stores.

735
Q

bicuspid aortic valve associated with
Rheumatic fever

Coarctation of aorta

mitral stenosis

mitral regurgitation

aortic regurgitation

A

B) Reason

Coarctation of the aorta:Associated with bicuspid aortic valve.Infantile type: Aorta narrowing is proximal to insertion of ductus arteriosus (preductal). Associated with Turner syndrome. Can present with closure of the ductus arteriosus (reverse with PGE2).

736
Q

Pt with genotype 46XX and ambiguous genitalia, Dx..??
true hermaphrodite

CAH

klinefilter syndrome

Turner syndrome

Patau syndrome

A

B) Reason

Clinical features CAH include:I. Salt wasting with hyponatremia, hyperkalemia, and hypovolemia due to lack of aldosterone. 2. Life-threatening hypotension due to lack of cortisol. 3. Clitoral enlargement (females) or precocious puberty (males) due to excess androgens

737
Q

projection fibers of cerebrum present in
striatum

putamin

internal capsule

globus pallidus

cerebellum

A

C) Reason

The white matter of the cerebral hemisphere are of three types:1.commissural fibers(The commissural fibers interconnect the corresponding regions of the two cerebral hemispheres eg Corpus callosum).2.association fibers(These fibers connect the various cortical regions of a cerebral hemisphere eg short and long association fibers).3.projection fibers(These fibers connect the cerebral cortex with lower part of the brain or brainstem and the spinal cord, in both directions eg Internal capsule).

738
Q

PDGF are derivatives of?
mast cells

eosinophils

platelets

Basophils

fibroelasts

A

C) Reason

PDGF Secreted by activated platelets and macrophages Induces vascular remodeling and smooth muscle cell migration Stimulates fibroblast growth for collagen synthesis

739
Q

tumor compressing anterior surface of sternocleidomastoid, vessel engorged will be
common carotid

internal jugular vein

external jugular vein

internal carotid artery

Subclavian vein

A

C) Reason

The muscle is covered superficially by skin, fascia, the platysma muscle, and the external jugular vein. It is the common site of compression of external jugular vein.

740
Q

Activity of osteoblasts is accelerated with?
Cortisol

LH

FSH

GH

ADH

A

D) Reason

FUNCTION: Stimulates linear growth (activating osteoblasts)and muscle mass through IGF-1/somatomedin secretion. inc. insulin resistance (diabetogenic)

741
Q

role of sternocleidomastoid
turn head to opposite

turn head to same

rotate neck

overhead abduction

circumduction

A

A) Reason

STERNOCLEIDOMASTOID MUSCLEWhen the sternocleidomastoid muscle contracts,it appears as an oblique band crossing the side of the neckfrom the sternoclavicular joint to the mastoid process of theskull. Its main function is to move the head to opposite side.

742
Q

dorsalis pedis is absent in child, foot will be supplied by
acessary peroneal

sural

great saphinous

tibial

fibular

A

A) Reason

Here in this scenario when dorsalis pedis artery absent, the perforating branch of the peroneal artery (acessary peroneal artery) will supply the foot.

743
Q

labia majora is drained by
superficial inguinal

deep inguinal

para aortic

Pudendal

pelvic

A

A) Reason

LYMPH DRAINAGEThe upper half of the anal canal drains into the pararectalnodes and then into the inferior mesenteric nodes. Thelower half including the labia majora drain into the medial group of superficial inguinal nodes.

744
Q

length of ureter in cm?
25

22

21

20

18

A

A) Reason

UretersThe two ureters are muscular tubes that extend from the kidneys to the posterior surface of the urinary bladd. Each ureter measures approximately 10 inch (25 cm) inlength and has an upper expanded end called the renalpelvis.

745
Q

major blood vessels present in which space
subdural

epidural

subarachnoid

intra canal

Arachnoid space

A

C) Reason

• Subarachnoid space lies between the arachnoid and pia mater containing CSF: contains blood vessels; site of subarachnoid hemorrhage.

746
Q

loss of movement of thumb and index finger due to gunshot but no sensory loss, nerve responsible?
superficial brand of ulnar

deep branch radial

Deep branch of Ulnar n.

Superficial brand of ulnar

Median Nerve

A

B) Reason

In damage of superficial branch of radial nerve there is sensory loss as well, but in case of deep nerve damaged, there is motor loss only.

747
Q

Which lesion will lead to lumber pain with loss of ankle jerk
S1

L5

S2

L4

L1

A

A) Reason

Ankle jerk has the root value of S1. Moreover Lumbosacral pain is associated with spine injury mostly. Ankle jerk lost mean lesion at S1.

748
Q

very long chain fatty acids are catalyzed in
glyoxisomes

peroxisomes

golgi complex

RER

SER

A

B) Reason

Peroxisome Membrane-enclosed organelle involved in catabolism of very-long-chain fatty acids, branched-chain fatty acids, and amino acids.

749
Q

common peroneal nerve damage will result in
rotation

planter flexion

dorsiflexion

inversion

eversion

A

E) Reason

Eversion: (opposite movement of the foot so that the solefaces laterally): caused by peroneus longus, peroneus brevis, peroneus tertius, and lateral tendons of extensor digitorumlongus muscles. It is lost in lesions of common peroneal nerve.

750
Q

relation of lateral fornix of vagina
Fallopian tube

ureter

Pudendal nerve

Pudendal artery

sciatic nerve

A

B) Reason

Uterine Artery thenpasses above the lateral fornix of vagina just close to the ureter to reach theuterus, where it ascends between the layers of the broadligament along the lateral margin of the uterus. It ends byfollowing the uterine tube laterally, where it anastomoseswith the ovarian artery.

751
Q

For what affinity of taut hemoglobin is reduced?
CO

O2

CO2

HCO3

K

A

B) Reason

Hemoglobin Hemoglobin (Hb) is composed of 4 polypeptide subunits (2 α and 2 β) and exists in 2 forms:T (taut) form has low affinity for O2.R (relaxed) form has high affinity for O2

752
Q

where is mostly pulsating abdominal mass found?
T10

T9

T8

T7

T12

A

E) Reason

Aortic aneurysms: Localized pathologic dilation of the aorta. Presents with pulsating mass most commonly over T10 level. May cause pain, which is a sign of leaking, dissection, or imminent rupture.

753
Q

1st route of spread of retinoblastoma out of optic cavity is
retinal artery

optic nerve

retinal vein

soft tissues

orbital fat

A

B) Reason

Retinoblastoma is a malignant retinal tumor of childhood. 1. The tumor is sporadic in approximately 60% of cases; sporadic cases are unilateral and mono centric in origin. 2. It is familial in approximately 40% of cases; familial cases are frequently bilateral and multicentric in origin. It involves the Optic nerve and spread through it to the extraocular tissues.

754
Q

Regarding neoplasia most common tumor after radiation
lymphoma

SCC

BCC

miningioma

HCC

A

D) Reason

MENINGIOMA A. Benign tumor of arachnoid cells, most of the time associated with radiation exposure.B. Most common benign CNS tumor in adults More commonly seen in women; rare in children C. May present as seizures; tumor compresses, but does not invade, the cortex. D. Imaging reveals a round mass attached to the dura.

755
Q

embryonic tumor of childhood
neuroblastoma

meningioma

nephroblastoma

medulloblastoma

astrocytoma

A

C) Reason

Wilms tumor (nephroblastoma) is the most common renal malignancy of early childhood (ages 2–4). Contains embryonic glomerular structures. Presents with huge, palpable flank mass and/or hematuria.

756
Q

1b fibers innervate
free nerve endings

Pacinian

murkle disc

GTO

muscle spindles

A

D) Reason

1b fibers inervate golgi tendon organ (GTO), While 1a fibers inervate the muscle spindles.

757
Q

febrile blood reaction after transfusion occurs. Best managment?
Adrenaline

Nor adrenaline

steroids

antipyretics

antihistamines

A

D) Reason

Non Hemolytic febrile reaction is just a febrile manifestation and is most of the time self remitting condition, can be managed by antipyretics.

758
Q

which drug used in headache is contraindicated in peripheral vascular disease
Ketorolac

NSAID

diclophenac

ergotamine

fluxetine

A

D) Reason

Ergotamine causes severe vasoconstriction of the periphral vasculature leading to adverse complications.

759
Q

A 30 year old man with bloody diarrhea, petachea and schistocytes on peripheral smear,diagnosis is?
ITP

TTP

DIC

HUS

sepsis

A

D) Reason

Hemolytic uremic syndrome, causes bloody diarrhea and hemolytic manifestations of petechiai, which typically is associated with enterohemorrhagic Escherichia coli (EHEC), also may occur after infection with shigellae that secrete Shiga toxin.

760
Q

A 29 year old man with abdominal pain and dirrohea, amoeba most commonly affects which part
cecum

jejunum

Illium

colon

rectum

A

A) Reason

Entamoeba histolytica:Amebiasis: bloody diarrhea (dysentery), usually involves cecum, liver abscess (“anchovy paste” exudate), RUQ pain (histology shows flask-shaped ulcer if submucosal abscess of colon ruptures)

761
Q

intermediate mesoderm develop into
urinary system

genital system

genitourinary system

Mesoderm

Endoderm

A

C) Reason

• The indifferent gonad begins development in a column of intermediate mesoderm called the urogenital ridge during week 4, and develops into the genitourinary system.

762
Q

ischiorectal fossa is related medially to
Uterus

bulbourethral

bulbospogiosus

internal sphincter

external anal sphincter

A

E) Reason

The ischiorectal fossa is a wedge-shaped space on each sideof the anal canal. The base of the wedge is superficial and is formed by the skin. The edge of the wedge is formed by the junction of the medial and the lateral walls.The medial wall is formed by the sloping levator ani muscle, external anal sphincter and the anal canal.

763
Q

relation of lesser sac
stomach & spleen

stomach & pancreas

stomach & GB

Stomach & lesser omentum

A

D) Reason

The lesser sac lies behind the stomach and the lesseromentum. It extends upward as far as the diaphragm and downward between the layers of the greater omentum

764
Q

A pt took antibiotics 2weeks back, now developed pancytopenia, Dx..?
sideroblastic anemia

iron deficiency anemia

G6PD deficiency

Aplastic anemic

Myelodysplastic disorders

A

D) Reason

Not a single features given in scenario is suggestive of Myelodysplastic disorders (pancytopenia with blast cells and ring sideroblasts), so most probable diagnosis js the Aplastic anemic.

765
Q

mucinous and serous staining is feature of which gland
parotid

Sublingual

Submandibular

maxillary

submaxillary

A

C) Reason

Submandibular gland is mixed, and Parotid is pure serous while Sublingual is pure mucous.

766
Q

Large intestine is removed. which deficiency is likely to occur?
water

electrolytes

minerals

vitamins

water and electrolytes

A

E) Reason

Large intestinal motility:■ Fecal material moves from the cecum to the colon (i.e., through the ascending, transverse, descending, and sigmoid colons), to the rectum, and then to the anal canal. Water and electrolytes are absorbed from colon.

767
Q

A 36 year old male patient with neck swelling and yellowish discharge, organism..?
Pseudomonas

spirochete

shegella

Actinomycetes

gardenella

A

D) Reason

Pigment-producing bacteria:Actinomyces israelii—yellow “sulfur” granules, which are composed of filaments of bacteria.Israel has yellow sand.S. aureus—yellow pigment. aureus (Latin) = gold.Pseudomonas aeruginosa—blue-green pigment. Aerugula is green.Serratia marcescens—red pigment. Serratia marcescens—think red maraschinocherries.

768
Q

prenatal diagnosis of Thalasemia is done by
genetic coding

CVS

USG

amnioscentesis

genotyping

A

B) Reason

β-thalassemia minor (heterozygote):β chain is underproduced.Usually asymptomatic.Diagnosis confirmed by inc. HbA2 (> 3.5%) on electrophoresis. Prenatal diagnosis by Chorionic Villous sampling (CVS).

769
Q

Tumor at outer upper quadrant, lymph drainage is by
anterior axillary

posterior axillary

apical

superior

lateral wall

A

A) Reason

Lymphatic Drainage:The lateral quadrants of the breast drain into the anterior axillary or the pectoral node. The medial quadrantsdrain into the internal thoracic nodes. A few lymph vesselsalso drain posteriorly into the posterior intercostal nodes,and some communicate with the lymphatic vessels of theopposite breast and with those of the anterior abdominalwall.

770
Q

Tumor at outer upper quadrant, lymph drainage is by
anterior axillary

posterior axillary

apical

superior

lateral wall

A

A) Reason

Lymphatic Drainage:The lateral quadrants of the breast drain into the anterior axillary or the pectoral node. The medial quadrantsdrain into the internal thoracic nodes. A few lymph vesselsalso drain posteriorly into the posterior intercostal nodes,and some communicate with the lymphatic vessels of theopposite breast and with those of the anterior abdominalwall.

771
Q

adrenal medulla derived from
ectoderm

mesoderm

endoderm

neural crest cell

yolk sac

A

D) Reason

Adrenal cortex (derived from mesoderm) and medulla (derived from neural crest)

772
Q

Cause of neurogenic shock
volume loss

sepsis

CVA

CRF

vasomotor tone loss

A

E) Reason

Shock occasionally results without any loss of bloodvolume. Instead, the vascular capacity increases somuch that even the normal amount of blood becomesincapable of filling the circulatory system adequately.One of the major causes of this is sudden loss of vasomotor tone throughout the body, resulting especially inmassive dilation of the veins. The resulting conditionis known as neurogenic shock.

773
Q

Type A agglutinins on cell membrane of RBCs, blood group???
B

AB

0+

A+

0-

A

D) Reason

Blood group A is one which contains antigen A on Red blood cells and antibodies B in serum.

774
Q

Thrombus in which vessel will lead to infarction of base of heart
LCA

LAD

LXA

marginal branch of RCA

posterior branch

A

D) Reason

Acute marginal artery—A branch of RCA: supplies right ventricle and hence the base of the heart. Its occlusion can lead to infarction of the base of the heart.

775
Q

Levator ani is innervated by
pelvic

Pudendal

sciatic

femoral

sacral plexus

A

B) Reason

Nerve Supply Pudendal nerve, Function: Supports the pelvic viscera;sphincter to the anorectal junction and vagina

776
Q

prostate is drained by
internal inguinal

external inguinal

para aortic

external iliac

internal iliac

A

E) Reason

The lymph drains into the internal iliac nodes.Nerve SupplyInferior hypogastric plexuses supply the prostate.

777
Q

structure found between extra peritoneal fat and transverse abdominus
internal oblique

transversalis facia

oblique fascia

facia lata

conjoint tendon

A

B) Reason

Layers of the anterior abdominal wall from outer to inside are:1. Skin 2. Superficial fascia a. Camper (fatty) b. Scarpa (fibrous) 3. External oblique 4. I nternal oblique 5. Transversus abdominis 6. Transversalis fascia 7. Extra peritoneal 8. Parietal peritoneum

778
Q

true about diphtheria produces
endotoxin

Both endotoxin and exotoxin

none of these

factor A2

exotoxin

A

E) Reason

Corynebacterium diphtheriae:Produces exotoxinInactivate elongation factor (EF-2)Manifestations: Pharyngitis with pseudomembranes in throat and severe lymphadenopathy (bull neck).

779
Q

Of the following list, bleeding time is normal in
hemophilia A

hemophilia B

VW disease

DIC

vit. k deficiency

A

A) Reason

Hemophilia A or B — Intrinsic pathway coagulation defect but bleeding time is normal A: deficiency of factor VIII, inc. PTT.B: deficiency of factor IX, inc. PTT.Macrohemorrhage in hemophilia—hemarthroses (bleeding into joints), easy bruising, inc. PTT. Treatment: recombinant factor VIII (in hemophilia A).

780
Q

smallest of the RNAs is
nuclear RNA

mitochondrial RNA

rRNA

mRNA

tRNA

A

E) Reason

tRNA Structure: 75–90 nucleotides, 2º structure, cloverleaf form, anticodon end is opposite 3′ aminoacyl end. All tRNAs, both eukaryotic and prokaryotic, have CCA at 3′ end along with a high percentage of chemically modified bases.

781
Q

deep sleep can be specified by?
beta wave

theta wave

gamma wave

delta wave

alpha wave

A

D) Reason

Deepest non-REM sleep (slow-wave sleep); when sleepwalking, night terrors, and bedwetting occur.Delta (lowest frequency, highest amplitude) is the charecteristic wave of deep sleep.

782
Q

lateral surface of foot is drained by
superficial inguinal

deep inguinal

para aortic

exrenal iliac

popliteal

A

E) Reason

Popliteal Lymph NodesSituated in the popliteal fossa, the popliteal lymph nodes receive the superficial lymph vessels that accompany thesmall saphenous vein from the lateral side of the foot andfrom the back and the lateral side of the calf. They also receive lymph from the deep structures from lateral side of foot.

783
Q

sciatic nerve damaged but still sensation over dorsum of foot is intact, reason
sural nerve

tibial nerve

femoral nerve

saphenus nerve

sciatic nerve

A

D) Reason

Saphnous nerve is the continuation of the femoral nerve, and supplies the medial side of foot and ball of big toe.

784
Q

lymph drainage of right thumb
trochenteric

axillary

internal thoracic

supraclavicular

infraclavicular

A

E) Reason

Infraclavicular (deltopectoral) nodes are not strictlyaxillary nodes, because they are located outside the axilla.They lie in the interval between the deltoid and the pectoralis major muscles. They receive lymph from the superficial vessels of the lateral side (thumb) of the hand, the forearm, and the arm; the lymph vessels accompany thecephalic vein.

785
Q

ovary is drained by
para aortic

internal iliec

external illeic

deep inguinal

superficial inguinal

A

A) Reason

Lymph Drainageof The Ovaries:The lymph drains into the para-aortic nodes at the level ofthe first lumbar vertebra.

786
Q

referal system is for
informed consent

experimental

better management

educational purposes

training purposes

A

C) Reason

Referral system is established to provide the better management to the patient from a center with less facilitates to the one with better facilitates.

787
Q

Among the following, the most common complication of DM
wet gangrene

Ischemic ulcer

dry gangrene

neuropathy

cataract

A

C) Reason

Large vessel complications of DM: atherosclerosis, CAD, peripheral vascular occlusive disease, and gangrene causing limb loss, cerebrovascular disease. MI most common cause of death.

788
Q

when vagus stimulates myocardium which phenomenon occurs?
AV node

bundle of His

Purkingi fibers

atria

SA node

A

E) Reason

Parasympathetic effects on heart rate and conduction velocity■ The SA node, atria, and AV node have parasympathetic vagal innervation, but the ventricles do not. The neurotransmitter is acetylcholine (ACh), which acts at muscarinic receptors.

789
Q

pt with rta sustained multiple fractures, low b.p. 1st line of treatment will be?
secure airway

close reduction

open reduction

tie bleeding

IV fluid

A

E) Reason

According to the standard ATLS protocol for the accident and emergency management, ABCDE is followed, while in an hypotensive bleeding pt. IV fluids are the 1st management in ATLS protocol.

790
Q

wound contraction is prevented with help of?
steroids

chloroquin

vit. C

Cu

Fe

A

A) Reason

FUNCTION: inc. Blood pressure (upregulates α1-receptors on arterioles dec. sensitivity to norepinephrine and epinephrineInc. Insulin resistance (diabetogenic)Inc. Gluconeogenesis, lipolysis, and proteolysisDec. Fibroblast activity (causes striae) Dec. Inflammatory and Immune responses, infalmatory and wound contraction.

791
Q

SA nodal branch in 60 percent is branch of?
LCA

RCA

LAD

LXA

LDA

A

B) Reason

Right-dominant circulation = 85% = PDA arises from RCA.Left-dominant circulation = 8% = PDA arises from LCX.Codominant circulation = 7% = PDA arises from both LCX and RCA

792
Q

Hypoxic injury affects most?
alveoli

Myocytes

nephron

osteoblasts

neuron

A

E) Reason

Neurons are one of the example of the permanent tissues that don”t regenerate usually. Neurons are the cells that bears least anoxia.

793
Q

most common complication of epidural anesthesia
hypotension

urinary retention

headache

pelvic pain

anal incontinence

A

B) Reason

Epidural anesthesia can often cause injury to the lower spinal nerves that often cause its manifestations most common of which is the atonic bladder or the urinary retention.

794
Q

glucose transport is enhanced by
inc. carrier

dec. carrier
inc. glycogen
inc. glucose conc.
inc. temperature

A

A) Reason

As glucose transport is an example of the facilitated transport, that is why its transport increases by increasing the carrier proteins (GLUT).

795
Q

In a 46 year old male with productive cough, cells of inflamation in lobar Pneumonia goes to
alveoli

bronchus

terminal bronchioles

alveolar ducts

alveolar sacs

A

A) Reason

Lobar Pneumonia: Causative organisms are S. pneumoniae most frequently, also Legionella, KlebsiellaFindings: Intra-alveolar exudate leading to consolidation; may involve entire lung.

796
Q

bifurcation of common illiac artry is crossed by
Fallopian tube

vesical urethra

ureter

vesical artery

pelvic splanchnic nerves

A

C) Reason

URETERIC CONSTRICTIONS AND STONESUreteric stones may be arrested at the following sites:• Where the renal pelvis joins the ureter.• Where the ureter is kinked as it crosses the pelvicbrim along the bifurcation of common iliac artry to enter the pelvis.• Where the ureter pierces the bladder wall.

797
Q

which of following inhibits thrombus formation
thromboxan

prostacyclin

GGA1

PG E2

PGE1

A

B) Reason

Prostaglandins■ Prostacyclin is a vasodilator in several vascular beds and it inhibits the formation of thrombus.■ E-series prostaglandins are vasodilators.

798
Q

common cause of liver cirrhosis in our country
Hep. B

Hep. D

Hep. A

Hep. C

Wilson disease

A

D) Reason

In our country, being a Muslim state, there is less cases with alcoholic cirrhosis. But due to the overuse of contaminated objects, cases of hep. C are more common.

799
Q

myelination in peripheral nerves is caused by
astrocytes

oligodenrocytes

glial cells

Ependymal cells

Schwan cells

A

E) Reason

Myelination increases conduction velocity of signals transmitted down axons. Results in saltatory conduction of action potential between nodes of Ranvier, where there are high concentrations of Na+channels. CNS—oligodendrocytes; PNS—Schwann cells

800
Q

hypertensive patient on diuretics, presents in ER in coma and K level of 6, cause..?
ARF

ATN

nephrotic syndrome

Acute GN

CKD

A

A) Reason

Acute kidney injury(acute renal failure)In normal nephron, BUN is reabsorbed (for countercurrent multiplication), but creatinine is not. Presentation can be azotemia or commatose state.

801
Q

deep inguinal ring opens at?
external oblique

internal oblique

transversalis facia

tranversus abdomen

conjoint tendon

A

C) Reason

The deep inguinal ring is an oval opening in the fascia transversalis and lies approximately 0.5 in. (1.3 cm)

802
Q

1 glucose molecule yeilds how many ATPs in O2
30

38

32

28

12

A

B) Reason

Completeoxidation of glucose through glycolysis pluscitric acid cycle will yield a net 38 ATPs .

803
Q

A 43 year old male Pt. has long standing GERD, endoscopy show esophagitis and goblet cells with columner cell
Carcinoma

dysplasia

anaplasia

Barrett esophagus

hyperplasia

A

D) Reason

Barrett esophagus Glandular metaplasia—replacement of nonkeratinized (stratified) squamous epithelium with intestinal epithelium (nonciliated columnar with goblets cells) in the distal esophagus. Due to chronic acid reflux (GERD)

804
Q

What kind of joints are intervertebral joints
primary Cartilagenous

secondary cartilaginous

Symphysis

gomphosis

Synchondrosis

A

B) Reason

Intervertebral disc forms secondary cartilaginous joint between the bodies of the vertebrae. If the nucleus pulposus part of the disc gets protruded backwards, it may press on the spinal nerve leaving out from the intervertebral foramina. The condition is known as herniation of the disc

805
Q

obiliteration of uterine cavity occurs due to fusion of decidua parietalis with
basalis

temporalis

occipitalis

centralis

capsularis

A

E) Reason

The part covering the conceptus is the decidua capsularis; that betweenthe conceptus and the uterine muscular wall is the decidua basalis (where the placenta subsequentlydevelops); and that which lines the remainder of the body of the uterus is the decidua parietalis. Uterus is obiliterated by these 2 decidua.

806
Q

After trauma cervical spine injury will result in
mydriasis

accommodation

miosis

cataract

retinal detachment

A

C) Reason

Horner syndrome Sympathectomy of face:Ptosis (slight drooping of eyelid: superior tarsal muscle)Anhydrosis (absence of sweating) and flushing (rubor) of affected side of faceMiosis (pupil constriction)Associated with lesion of spinal cord above T1

807
Q

primary afferent terminates in
medial leminiscus

spinothalamic tract

latral leminiscus

spinobulbar

rubrospinal

A

B) Reason

Anterolateral (spinothalamic tract) system:The anterolateral system carries pain, temperature, and crude touch sensations from the extremities and trunk.

808
Q

cataract occur due long term use of which drug
rifampin

INH

pyrazenamide

chloroquin

ethambutol

A

D) Reason

Cataract:Painless, often bilateral, opacification of lens dec. in vision. Risk factors: incresing age, smoking, EtOH, excessive sunlight, drugs (prolonged corticosteroid use, chloroquin) classic galactosemia, galactokinase deficiency, diabetes (sorbitol), trauma, infection.

809
Q

mood and emotions are brought about by which neurotransmitter?
epinephrine

Norepinephrine

serotonin

glutamate

Dopamine

A

E) Reason

Mood disorder is characterized by an abnormal range of moods or internal emotional states and loss of control over them. Caused by imbalance of Dopamine, neurotransmitter for mood.Severity of moods causes distress and impairment in social and occupational functioning.

810
Q

left sided weakness with tongue deviation to right, lesion at medulla, artery involved..?
MMA

ACA

vertebral

PCA

posterior communicating

A

C) Reason

Contralateral hemiparesis—upper and lower limbs.sec. contralateral proprioception. Ipsilateral hypoglossal dysfunction (tongue deviates ipsilaterally).Stroke commonly bilateral.Medial medullary syndrome— caused by infarct of paramedian branches of ASA and vertebral arteries

811
Q

hormone released during stress?
LH

FSH

androgen

TSH

ACTH

A

E) Reason

Stress hormones are those that are secreted in in stressful conditions. These include ACTH, GH, Cortisol, epinephrine, Norepinephrine and glucagon.

812
Q

In an old 57 year male, Oat cell CA causes what
Down syndrome

Carcinoid

asthma

cushing syndrome

Sarcoidosis

A

D) Reason

Small cell (oat cell) carcinoma:Central location, Undifferentiated and very aggressive.May produce ACTH (Cushing syndromeg) , ADH, or Antibodies against presynaptic Ca2+ channels (Lambert-Eaton myasthenic syndrome).

813
Q

Hemostasis begins with?
platelet activation

vasoconstriction

Vasodilation

thrombus formation

clotting

A

B) Reason

Whenever there is an injury that would stimulate the endothelium, then vasoconstriction is the 1st response for the Hemostasis.

814
Q

stony dull percusion is character of
air in plueral cavity

fluid in pleural cavity

fibrosis

Pneumoniae

COPD

A

B) Reason

Whenever there is a fluid in the the pleural cavity, the percussion note will be dull,normal note is tympanic in presence of normal pleural cavity.

815
Q

A man sweats 2L, then drinks 2L pure water, what will happen/
inc. both

inc. ICF Osmolarity
inc. ECF

causes dehydration

inc. ICF

A

E) Reason

There will be increase in ICF.

816
Q

all will cause edema except
dec. proteins

Vasodilation

CRF

vasoconstriction

inc. oncotic pressure

A

E) Reason

increase oncotic pressure is most correct option here.

817
Q

anatomical snuff box injury leads to damage of?
radial nerve

ulner nerve

radial artery

ulnar artery

median nerve

A

C) Reason

The cephalic vein can be seen in the proximal roof of the snuffbox, and the pulsation of the radial arterycan be felt in its depth. The snuffbox is bounded on the radial side by the tendons of abductor pollicislongus laterally and extensor pollicis brevis medially; these tendons lie close to each other

818
Q

pap smear of a 39 year old lady show hyperchromasia and koilocytes, Dx..?
anaplasia

dysplasia

Metaplasia

hyperplasia

hypertrophy

A

B) Reason

Dysplasia and carcinoma in situDisordered epithelial growth; begins at basal layer of squamocolumnar junction (transition zone) and extends outward. Classified as CIN 1, CIN 2, or CIN 3 (severe dysplasia or carcinoma in situ), depending on extent of dysplasia. Associated with HPV 16 and HPV 18, which produce both the E6 gene product (inhibits p53 suppressor gene) and E7 gene product (inhibits RB suppressor gene).

819
Q

p53 mutation causes cancer by which mechanism
inc. cell survival

inc. apoptosis
sec. necrosis
dec. replication
inc. replication

A

A) Reason

Tumor suppressors: p53 and hypophosphorylated Rb normally inhibit G1-to-S progression; mutations in these genes result in unrestrained cell division and increased cell survival(e.g., Li-Fraumani syndrome).

820
Q

artery arising from posterior of bifurcation of aorta
median sacral

lateral sacral

pudendal

ureteric

vesical

A

A) Reason

Three terminal arteries: two common iliac arteries and themedian sacral artery. Two are the continuation and third originates from posterior of the bifurcation of aorta

821
Q

role of renin?
alpha 1

alpha 2

beta 2

β 1

M2

A

D) Reason

Renin Secreted by JG cells in response to inc. renal arterial pressure and inc. renal sympathetic discharge (β1 effect).

822
Q

measurment of renal plasma flow is by?
Inulin

PABA

Creatinine

Hipporate

urea

A

D) Reason

Measurement of renal plasma flow (RPF)—by clearance of para-aminohippuric acid (PAH)■ PAH is filtered and secreted by the renal tubules.■ Clearance of PAH is used to measure RPF.■ Clearance of PAH measures effective RPF and underestimates true RPF by 10%.

823
Q

Regarding plasma K what is true?
Major ECF cation

usually moves out

usually moves in

98% in ECF

98% in ICF

A

E) Reason

Intracellular fluid (ICF)■ is two-thirds of TBW.■ The major cations of ICF are K+ and Mg2+.■ The major anions of ICF are protein and organic phosphates

824
Q

A ptient presents with malnutrition, edema, and dec. albumin, clinical condition is called as?
marasmus

kwashirkow

nephrotic syndrome

Down syndrome

patau syndrome

A

B) Reason

Kwashiorkor:Protein malnutrition resulting in skin lesions, edema, liver malfunction (fatty change due to decreased apolipoprotein synthesis). Clinical picture is small child with swollen belly

825
Q

pelvic splanchnic nerves are
parasympathetic

sympathetic

pelvic

somatic

involuntary

A

A) Reason

The preganglionic cell bodies of the parasympathetic nervous system are found in the CNS in one of 2 places: • Gray matter of brain stem associated with cranial nerves III, VII, IX, and X, or • Spinal cord gray in sacral segments S2 3, and 4 (pelvic splanchnics)

826
Q

where does entry of glucose is increased with insulin?
muscles

adipose tissue

muscle & adipose tissue

lense

Myocytes

A

C) Reason

Anabolic effects of insulin:inc. glucose transport in skeletal muscle and adipose tissueinc. glycogen synthesis and storageinc. triglyceride synthesisinc. Na+ retention (kidneys)

827
Q

which is necessary for development of Seminiferous tubules
FSH

LH

androgen

FSH & androgen

testosterone

A

D) Reason

Within each lobule are 1 -4 tubes, seminiferous tubules (developed under the action of FSH and androgen), where spermatozoa are produced. Each tubule is a coiled, non-branching closed loop that is 1 50-200 µm in diameter and 30-70 cm in length.

828
Q

enzyme act by
dec. Ea

inc. temperature
inc. pH
inc. Ea
inc. rate of reaction

A

A) Reason

Enzymes are the specific type of the functional proteins that speed up the reaction by decreasing the acitivation energy.

829
Q

hypothalamus releases inhibitory factor to control which hormone?
ACTH

Prolactin

LH

FSH

TSH

A

B) Reason

REGULATION Prolactin secretion from anterior pituitary is tonically inhibited by dopamine from hypothalamus. Prolactin in turn inhibits its own secretion by dec. dopamine synthesis and secretion from hypothalamus. TRH inc.prolactin secretion.

830
Q

Dicrotic notch is formed because of
aortic diastole

atrial systole

atrial diastole

aortic systole

aortic closure

A

E) Reason

The “blip” in the aortic pressure tracing occurs during aortic cystole after closure of the aortic valve and is called the dicrotic notch, or incisura.

831
Q

decrease in surfactant in lungs will result in?
dec. complience

dec. CO affinity
inc. O2 affinity
inc. complience

cause Hypoxia

A

A) Reason

Surfactant deficiency inc. surface tension decreases lung compliance and increased chances of alveolar collapse. A lecithin:sphingomyelin ratio < 1.5 in amniotic fluid is predictive of neonatal respiratory distress syndrome.

832
Q

Sample from 51 yearcold male with lymphadenopathy, epitheloid cells in TB granuloma are derived from
lymphocytes

NK cells

monocytes

macrophages

monocytes

A

C) Reason

Granuloma: nodular collections of epithelioidmacrophages (derived from monocytes) and giant cells. Outcomes include scarring and amyloidosis.

833
Q

A 58 year old male with DM for last 23 year, most common complication of DM in eye
cataract

Cilliary spasm

keratitis

retinopathy

conjunctival redness

A

D) Reason

Small vessel disease (diffuse thickening of basement membrane) retinopathy (hemorrhage, exudates, microaneurysms, vessel proliferation) , glaucoma, nephropathy (nodular sclerosis, progressive proteinuria, chronic renal failure, arteriolosclerosis leading to hypertension, Kimmelstiel-Wilson nodules)

834
Q

which one of these is part of innate immunity only?
NK cells

lymphocytes

monocytes

mast cells

macrophages

A

A) Reason

Natural killer cells Use perforin and granzymes to induce apoptosis of virally infected cells and tumor cells.Activity enhanced by IL-2, IL-12, IFN-β, and IFN-α.

835
Q

chief parameter for measuring GFR
PAH

PABA

urea

Inulin clearance

Creatinine

A

D) Reason

Measurement of GFR—clearance of inulin■ Inulin is filtered, but not reabsorbed or secreted by the renal tubules.■ The clearance of inulin is used to measure GFR

836
Q

cause of dysphagia to liquids?
LMN LESION

UMN LESION

cerebellar lesion

frontal lesion

NM disorder

A

E) Reason

liquids don”t have any physical hinderence to the swallowing, if there is Neuromuscular disorders that cause dysphagia to liquids as well.

837
Q

which epithelium has no basement membrane
simple cuboidal

urothelium

simple columner

simple squamous

stratified columner

A

B) Reason

The urethra extends from the urinary bladder to the tip of the penis. The prostatic urethra is composed of transitional epithelium or Urothelium and the distal urethra of stratified epithelium. It has no basement membrane.

838
Q

A 56 year old male with Hx of hypertensive with family history of Medullary CA thyroid, Dx..?
aortic stenosis

pheochromocytoma

renal stenosis

Cushing syndrome

conn syndrome

A

B) Reason

Medullary carcinoma From parafollicular “C cells”; produces calcitonin, sheets of cells in an amyloid stroma B. Associated with MEN 2A (especially phaeochromocytoma) and 2B (RET mutations).

839
Q

Maong the following, infection common in 3rd trimester in a low socioeconomic lady
P. carinii

aspergilus

A. histolytica

candida

Histoplasma

A

D) Reason

Candida albicans: alba = white.Systemic or superficial fungal infection. Oral and esophageal thrush in immunocompromised (neonates, steroids, pregnancy especially 3rd trimester, diabetes, AIDS), vulvovaginitis (diabetes, use of antibiotics), diaper rash, endocarditis in IV drug users, disseminated candidiasis (to any organ), chronic mucocutaneous candidiasis.

840
Q

Which nerve supplies middle of face?
ophthalmic nerve

mandibular nerve

maxillary nerve

facial nerve

Trigeminal nerve

A

C) Reason

Face is supplied by the Trigeminal nerve. Upper 3rd of face is supplied by the Ophthalmic nerve, middle by Maxillary nerve, and lower 3rd by the mandibular division of Trigeminal nerve.

841
Q

drug given for short term anesthesia of episiotomy
lignocain

prilocain

bupivacain

ropivacain

procain

A

E) Reason

CLINICAL USE: Minor surgical procedures especially procain, spinal anesthesia. If allergic to esters, give amides.TOXICITY:CNS excitation, severe cardiovascular toxicity (bupivacaine), hypertension, hypotension, and arrhythmias (cocaine).

842
Q

A 43 year old male has cecal mass, true about it
RET MUTATION

K-RAS mutation

Rb mutation

PAS mutation

gene loss

A

B) Reason

When APC gene is detected. the colon will be at risk of tumor formation. Further when K-RAS is mutated, there is risk of adenoma (as in this scenario) and finally p53 mutation causes cancer (molecular progression)

843
Q

forceful expiration is a feature of
quadratus lumborum

rectus sheet

rectus abdominis

external oblique

internal oblique

A

C) Reason

Forced ExpirationForced expiration is an active process accomplished bycontraction of the muscles of the anterior abdominal wall i.e. the transversus abdominis(forcing the relaxed diaphragm upward by raising intraabdominal pressure) and contraction of the quadratuslumborum (pulling the twelfth rib downward).

844
Q

What stabilizes TM joint
quadrate ligament

mandibular ligament

stylomandibular Ligament

hyoid ligament

condilyoid ligament

A

C) Reason

Stylomandibular Ligament:The stylomandibular ligament is the main support of the TM joint, it is attached to the styloid process above and to the angle of the mandible below.

845
Q

A 45 years old male with malignant melanoma, patient”s biopsy will show which cells
monocytes

lymphocytes

macrophages

leukocytes

NK cells

A

E) Reason

Malignant Melanoma: Common tumor with significant risk of metastasis. S-100 tumor marker and NK cells on histology. Associated with sunlight exposure; fair-skinned persons are at risk. Depth of tumor correlates with risk of metastasis.

846
Q

cell with large dark nucleus
eosinophils

monocytes

Basophils

lymphocytes

mast cells

A

C) Reason

Basophils: Mediates allergic reaction. Densely (dark)basophilic granules A containing heparin (anticoagulant), histamine (vasodilator), and leukotrienes.

847
Q

How is hip flexed over a long range?
rectus femoris

quadricep femoris

sartorius

adductor longus

poplieus

A

A) Reason

MOVEMENTThe hip joint has a wide range of movement.• Flexion: Iliopsoas, rectus femoris (strongest of flexors), sartorius, and adductormuscles.• Extension (posterior movement of the flexed thigh):Gluteus maximus and hamstring muscles

848
Q

Planter flexion lost, nerve responsible is/
tibial

common peroneal

sural

great Saphinus

fibular

A

A) Reason

Plantar flexion: (toes pointing downward): affected by tibial nerve lesion. Muscle causing movements are Gastrocnemius, soleus, plantaris, peroneus longus, peroneus brevis,tibialis posterior, flexor digitorum longus, and flexor hallucis longus muscles.

849
Q

Supply of dura of middle cranial fossa
CN VIII

CN VII

CN VIII

CN IX

CN V

A

E) Reason

The trigeminal nerve is the largest cranial nerve .It leaves the anterior aspect of the pons as a small motor rootand a large sensory root, and it passes forward, out of theposterior cranial fossa, to reach the middle cranial fossa, where it gives meningeal branches to its dura.

850
Q

fibrin clot is stabilized by?
factor 8

factor 9

factor 10

factor 13

factor 11

A

D) Reason

Factor X Stuart factor; Stuart-Prower factorFactor XI Plasma thromboplastin antecedent (PTA); antihemophilic factor CFactor XII Hageman factorFactor XIII Fibrin-stabilizing factorPrekallikrein Fletcher factor

851
Q

A 54 year old man travel to Pakistan 1st time, developed high grade fever and chills, later he lost conscious level, Dx..?
CVA

Rikketsia

P. malarie

P. vivax

falciparum malaria

A

E) Reason

P. falciparum—severe; irregular and high gradefever patterns; parasitized RBCs occlude capillaries in brain (cerebral malaria), kidneys, lungs.

852
Q

intracellular proteins that maintain protein folding are
elastins

Proreasomes

chaperons

reticulins

fibrins

A

C) Reason

Chaperone protein: Intracellular protein involved in facilitating and/or maintaining protein folding. In yeast, some are heat shock proteins (e.g., Hsp60) that are expressed at high temperatures to prevent protein denaturing/misfolding

853
Q

water is impermeable at which segment of nephron?
PCT

DCT

loop of Henle

thick ascending limb

collecting ducts

A

D) Reason

■ Thick ascending limb of loop of Henle is impermeable to water. Thus, NaCl is reabsorbed without water. As a result, tubular fluid [Na+] and tubular fluid osmolarity decrease to less than their concentrations in plasma (i.e., TF/PNa+ and TF/Posm < 1.0). This segment, therefore, is called the diluting segment.

854
Q

how many ATPs are produced in 1 cycle of TCA by 1 acetyl CoA
2

4

6

10

12

A

D) Reason

The TCA cycle produces 3 NADH, 1 FADH2, 2 CO2, 1 GTP per acetyl-CoA = 10 ATP/acetyl-CoA (2× everything per glucose). TCA cycle reactions occur in the mitochondria.α-ketoglutarate dehydrogenase complex requires the same cofactors as the pyruvate dehydrogenase complex (B1, B2, B3, B5, lipoic acid).

855
Q

contraindication to systemic antifungal
vaginal thrush

systemic candidiasis

oral candidiasis

aspergilosis

Histoplasmosis

A

C) Reason

Treatment: topical azole for vaginal; fluconazole or caspofungin for oral/esophageal; fluconazole, amphotericin B, or caspofungin for systemic.

856
Q

Regarding appendix what is true?
short vessels

anterior position

opens to colon

own Mesentary

has no Mesentary

A

D) Reason

It has a completeperitoneal covering, which is attached to the mesentery ofthe small intestine by a short mesentery of its own called the mesoappendix. The mesoappendix contains the appendicular vessels and nerves.

857
Q

Which lesion will result in loss of bladder control?
lumber plexus

sacral plexus

pelvic nerve

L5-S1 lesion

S2-4 lesion

A

E) Reason

The afferent impulses enter the second, third, and fourthsacral segments of the spinal cord, these control the bladder tone.Efferent impulses leavethe cord from the same segments and pass via the parasympathetic preganglionic nerve fibers in the hypogastricplexuses to the bladder wall, where they synapse with postganglionic neurons.

858
Q

A pt complain of water discharge from eye while eating. Which nerve is compromised
CN V

CN VIII

CN VII

CN IX

CN VI

A

C) Reason

Crocodile tears (gustatory lacrimation). There is unil ateral lacrimation with mastication. This is due to faulty regeneration of parasympathetic fibres of facial nerve which now supply lacrimal gland instead of the salivary glands.

859
Q

which muscle will not be damaged if left little finger is damaged?
extensor digitorum

extensor indicis

extensor hallucis

dorsal interossei

flexor digitorum profundus

A

D) Reason

It spares during injury of little finger.

860
Q

In a blood bank which anticoagulant is best to store blood?
CPDA

citrate

heparin

warfarin

Clopidogrel

A

A) Reason

Any substance that deionizes the blood calcium willprevent coagulation.The negatively charged citrate ionis especially valuable for this purpose, mixed withblood usually in the form of sodium, ammonium, orpotassium citrate (CPDA). The citrate ion combines withcalcium in the blood to cause an un-ionized calciumcompound, and the lack of ionic calcium preventscoagulation

861
Q

Rickettsia takes which stain
Gram staining

ZN staining

Geimsa

silver stain

sudan red

A

C) Reason

Giemsa:stains Chlamydia, Borrelia, Rickettsiae, Trypanosomes, Plasmodium.

862
Q

In kids which structure will be injured while passing IV line?
femoral vein

saphenous nerve

anterior tibial artery

great saphenous vein

great saphenous artery

A

B) Reason

Saphenous nerve, which descends through thefemoral triangle and the adductor canal and crosses thefemoral artery. The nerve emerges on the medial side ofthe knee joint between the tendons of the sartorius and thegracilis muscles, and it accompanies the great saphenousvein down the medial side of the leg and in front of the medial malleolus. It passes along the medial border of thefoot as far as the ball of the big toe. It is the commonally injured nerve in venopunture of children.

863
Q

What structure plays a role of station during labor?
Ischial spine

ischium

pubis

pubic tubercle

true pelvis

A

A) Reason

The ischiorectal fossa and the ischial spines can bepalpated laterally in the pelvis, Ischial spine palpated here is the station for fetus descent in labour

864
Q

hyperosmolar over hydration
Cushing syndrome

conn syndrome

phaechromocytoma

acute bronchitis

acute dirrohea

A

A) Reason

FINDINGS of Cushing syndrome: Hypertension due to hyperosmolar over hydration leading to inv. volume), weight gain, moon facies, truncal obesity B, buffalo hump, hyperglycemia (insulin resistance), skin changes (thinning, striae), osteoporosis, amenorrhea, and immune suppression.

865
Q

effect of Acetazolamide
osmotic diuresis

NaCl loss

K loss

Ca loss

NaCO3 diuresis

A

E) Reason

AcetazolamideMECHANISM Carbonic anhydrase inhibitor. Causes self limited NaHCO3 diuresis and dec. total-body HCO3- stores.

866
Q

2nd part of dudenum has anterior relation of
appendix

hepatic flexor

mesocolon

right suprarenal

spleenic vein

A

C) Reason

TRANSVERSE COLONThe transverse colon is approximately 15 in. (38 cm) inlength and passes across the abdomen, occupying the umbilical and the hypogastric region. It begins atthe right colic flexure below the right lobe of the liver andhangs downward, suspended by the transverse mesocolonfrom the pancreas. It then ascends from anterior to the dudenum to the left colic flexurebelow the spleen.

867
Q

Which cells migrate to epidermis during 3rd month of gestation.?
fibroelasts

lymphocytes

keratinocytes

melanocytes

mast cells

A

D) Reason

In pregnancy, anterior pituitary hormones including MSH are increased. In 3rd trimester, these excessive melanocytes come under the eidermis and are reason for the increased pigmentation during pregnancy.

868
Q

histology of macula densa
no basal lamina

no basement membrane

no basal layer

deep basal cells

deep basal lamina

A

E) Reason

The macula densa is formed by tall cuboidal cells with deep basal lamina in the wall of the DCT which detect sodium levels in the tubular fluid.

869
Q

extra embrionic coelom develops from
hypoblast

epiblast

mesoblast

yolk sac

Mesoderm

A

A) Reason

The inner cell mass of the first week differentiates into the epiblast and hypoblast cells and forms a bilaminar embryonic disk. An amniotic cavity develops from the epiblast and the primary yolk sac replaces the blastocyst cavity. Hypoblast then forms the embrionic coelom.

870
Q

A patient presents with bone mass, Giant cell tumor appearance on X-ray
soap bubbles

starry sky

onion peel

Reticular

apple core

A

A) Reason

Giant cell tumor:20–40 years old.Epiphyseal end of long bones.Locally aggressive benign tumor often around knee.“Soap bubble” appearance on x-ray.Has multinucleated giant cells.

871
Q

carcinoma has following intermediate filaments
elastin

caspase

fibrin

collegen II

keratin

A

E) Reason

Carcinomas have keratin in their structure. First step in the invasion of malignant cells through an epithelium results from a loss of expression of cadherins that weakens the epithelium.

872
Q

supply of bronchopulmonary segment
primary bronchus

secondary bronchus

tertiary bronchus

terminal bronchioles

alveolar sacs

A

D) Reason

BRONCHOPULMONARY SEGMENTSBronchopulmonary segments are the anatomic, functional,and surgical units of the lungs. Each lobar (secondary)bronchus, which passes to a lobe of the lung, gives offbranches called segmental (tertiary) bronchi .

873
Q

In a burn patient, most common post burn non bacterial infection
Histoplasma

aspergilus

S. hematobium

candida

S.mansoni

A

D) Reason

Candida albicans: alba = white.Systemic or superficial fungal infection. Oral and esophageal thrush in immunocompromised (neonates, Burns , steroids, diabetes, AIDS), vulvovaginitis (diabetes, use of antibiotics), diaper rash, endocarditis in IV drug users, disseminated candidiasis (to any organ), chronic mucocutaneous candidiasis.

874
Q

which of following is intraarticular tendon
sartorius

popliteus

adductor logus

gastrocnemius

soleus

A

B) Reason

Oblique Popliteal LigamentThe oblique popliteal ligament is a tendinous expansion ofthe semimembranosus muscle. It strengthens the backof the capsule.

875
Q

MCP joints are flexed mainly via
lumbricals

extensor digitorum

flexor digitorum

extensor hallucis

extensor indicis

A

A) Reason

• Flexion: Lumbricals and interossei muscles, assisted byflexor digitorum superficialis and profundus muscles.• Extension: Extensor digitorum, extensor indicis, and extensor digiti minimi muscles.

876
Q

which segment of nephron absorbs bicarbonate?
loop of Henle

DCT

thick ascending limb

collecting duct

PCT

A

E) Reason

Early proximal convoluted tubule (PCT)—contains brush border. Reabsorbs all of the glucose and amino acids and most of the HCO3–, Na+, Cl–, PO43–, K+, and H2O. Isotonic absorption. Generates and secretes NH3, which acts as a buffer for secreted H+.

877
Q

Among the following, the most Radiosensitive tumor
astrocytoma

glioma

schanoma

meningeoma

medulloblastoma

A

E) Reason

meduloblastoma are most radiosensitive tumors and glioblastoma are least radiosensitive.

878
Q

74 year old man with symptoms of urinary obstruction and frequency, has firm mass on DRE, PSA is 19, Dx..?
BPH

CA colon

CA rectum

Adenocarcinoma prostate

hemoroids

A

D) Reason

Prostatic adenocarcinoma:Common in men > 50 years old. Arises most often from the posterior lobe (peripheral zone) of the prostate gland and is most frequently diagnosed by inc. PSA and subsequent needle core biopsies. Prostatic acid phosphatase (PAP) and PSA are useful tumor markers (inc. total PSA, with inc. fraction of free PSA).

879
Q

laterally bladder has a posterior relation with
ureter

seminiferous tubules

seminal vesicle

prostate

Pudendal nerve

A

C) Reason

SEMINAL VESICLESThe seminal vesicles are paired organs that lie on the posterior surface of the bladder and lateral to the terminal part of the vas deferens.

880
Q

A 48 year old male has dec. range of motion of upper limb, has normal CBC and other baseline with Ca 7.8 and PO4 6.4, albumin 6.2, Creatinine 7.8, Dx..?
parathyroid adenoma

CA parathyroid

excess vit. D

renal failure

parathyroid hyperplasia

A

E) Reason

Secondary Hyperparathyroidism: 2° hyperplasia due to decreased gut Ca2+ absorption and increased PO43-, most often in chronic renal disease (causes hypovitaminosis D decreased Ca2+ absorption).

881
Q

cyclosporine MOA in transplant patients
inhibit B cells

inhibit T cell differentiation

inhibit NK cells

inhibit lymphocytes

inhibit monocytes

A

B) Reason

Cyclosporine: MoA, Calcineurin inhibitor; binds cyclophilin. Blocks T cell activation by preventing IL-2 transcription.Uses: Transplant rejection prophylaxis, psoriasis, rheumatoid arthritis.

882
Q

A 28 year old male patient comes to you, his laparotomy scar is raised and hard, Dx..?
keloids

hernia

excessive elastin

Hypertonic scar

burst abdomen

A

D) Reason

Scar formation 70–80% of tensile strength returns at 3 months following wound; little additional tensile strength will be regained. Abnormally thick and tough struture formed by excessive scarring is the Hypertrophic scar.

883
Q

While performing cubital venesectiom which nerve is likely to be damaged?
radial

ulner nerve

axillary artery

radial artery

median

A

E) Reason

From medial to lateral, the cubital fossa contains the median nerve, the bifurcation of the brachial artery into the ulnar and the radial arteries, the tendon of the biceps muscle,and the radial nerve and its deep branch. Median nerve is the most commonly damaged during venopunture in cubital fossa.

884
Q

Regarding neoplasia, radiation cause protein damage that cause carcinogenesis by
bcr

abl

Rb

TET

BCl2

A

E) Reason

Bcl-2 prevents cytochrome c release by binding to and inhibiting Apaf-1. Apaf-1 normally induces the activation of caspases. If Bcl-2 is overexpressed (e.g., follicular lymphoma), then Apaf-1 is overly inhibited, leading to inc. caspase activation and tumorigenesis.

885
Q

which renal function is determined with specific gravity of urine?
secretion

filtration

absorption

concentration

excretion

A

D) Reason

Specific gravity of urine is the measurement of the concentration of solutes in urine. As the concentrating ability of the kidney is impaired, the specific gravity changes.

886
Q

lab. measurement of vit.D involves which metabolite
1 hydroxy vit.D

vit. D1
vit. D2

25 dihydroxy vit D

1,25 dihydroxy vit. D

A

D) Reason

Vitamin D D2 = ergocalciferol—ingested from plants.D3 = cholecalciferol—consumed in milk, formed in sun-exposed skin (stratum basale).25-OH D3 = storage form. 1,25-(OH)2 D3 (calcitriol) = active form

887
Q

Pudendal nerve roots
S1

S2-3

S1-2

S2-4

S3-4

A

D) Reason

The pudendal nerve (S2, 3, and 4) leaves the pelvisthrough the greater sciatic foramen and enters the perineum through the lesser sciatic foramen.• Nerves to the piriformis muscle.

888
Q

most common risk of giving systemic steroids to diabetic
delayed healing

poor sugrar control

obesity

edema

wheezing

A

B) Reason

TOXICITY Iatrogenic Cushing syndrome—buffalo hump, moon facies, truncal obesity, muscle wasting, thin skin, easy bruisability, osteoporosis (treat with bisphosphonates), poor sugrar control in diabetics, adrenocortical atrophy, peptic ulcers, diabetes (if chronic).Adrenal insufficiency when drug stopped abruptly after chronic use.

889
Q

Rheumatic fever is the example of which type of HSR
delayed response

type 1 HSR

type 3 HSR

type 4 HSR

type 2

A

E) Reason

Type II: Autoimmune hemolytic anemiaPernicious anemiaIdiopathic thrombocytopenic purpuraErythroblastosis fetalisAcute hemolytic transfusion reactionsRheumatic feverGoodpasture syndromeBullous pemphigoidPemphigus vulgaris

890
Q

vulva has nerve supply of?
femoral

Obturator

sacral plexus

lumber plexus

pudendal

A

E) Reason

Muscles of the perineum, including the external analsphincter, mucous membrane of the lower half of theanal canal, perianal skin, skin of the penis, scrotum,clitoris, and labia majora and minora (vulva).

891
Q

inhaled anesthetic is given with Ca channel bloker, it will cause
headache

bleeding

hypertension

hypotension

rapid onset

A

D) Reason

Ca channel blockers are the Vasodilators that are used along with the volatile anesthetics. Their Vasodilation property is the cause of hypotension.

892
Q

A pt”s labs show normal PT & deranged APTT. what is most likely cause?
aspirin

factor 8 inhibition

Clopidogrel

heparin

warfarin

A

B) Reason

It is clear that wheneither Factor VIII or platelets are in short supply,this step is deficient. Factor VIII is the factor thatis missing in a person who has classic hemophilia (darranged PTT and normal PT),for which reason it is called antihemophilic factor.Platelets are the clotting factor that is lacking inthe bleeding disease called thrombocytopenia

893
Q

correct regarding xenograft
twins

same specie

from siblings

other species

same person

A

D) Reason

Autograft From self.Syngeneic graft From identical twin or clone.Allograft From nonidentical individual of same species.Xenograft From different species.

894
Q

In a 29 year old women with sudden onset of bleeding disorder, here suspected DIC, diagnostic test
inc. PT

inc. PTT

D dimers

inc. fibrin
inc. thrombin

A

C) Reason

Labs: schistocytes, increased fibrin split products (d-dimers), decreased fibrinogen, decreased factors V and VIII

895
Q

round ligament of liver is remnant of
umbilical artery

yolk sac

umbilical vein

foramin ovale

ductus venosus

A

C) Reason

Falciform Ligament Connects Liver to anterior abdominal wallIt contains Ligamentum teres hepatis/ Round ligament(derivative of fetal umbilical vein)

896
Q

1 yr old boy has reccurent URTI and cough and chronic dirrohea, test for diagnosis
stool culture

suputum culture

jejunal biopsy

colonoscopy

chest HRCT

A

C) Reason

Celiac Disease: Autoimmune-mediated intolerance of gliadin (wheat) leading to malabsorption and steatorrhea. Associated with HLA-DQ2, HLA-DQ8, and northern European descent. Findings include anti-endomysial, anti-tissue transglutaminase, and anti-gliadin antibodies; blunting of villi; and lymphocytes in the lamina propria. decreased mucosal absorption that primarily affects distal duodenum and/or proximal jejunum. Jejunal biopsy is the diagnostic evaluation.

897
Q

which structure forms roof of ischiorectal fossa?
perineal membrane

levator ani

perineal muscles

bulbourethral gland

bulbospongeosus

A

B) Reason

Ischiorectal FossaThe ischiorectal fossa is a wedge-shaped space on each sideof the anal canal . The base of the wedge is superficial and is formed by the skin. The edge of the wedge isformed by the junction of the medial and the lateral walls.The medial wall and roof is formed by the sloping levator ani muscleand the anal canal; the lateral wall is formed by the lower part of Obturator internus muscle.

898
Q

blood is stored mostly with citrate. why?
bind to heparin

binds to Ca

bind to platelet

bind to fibrinogen

bind to fibrin

A

B) Reason

The citrate ion combines withcalcium in the blood to cause an un-ionized calciumcompound, and the lack of ionic calcium preventscoagulation. Citrate anticoagulants have an importantadvantage over the oxalate anticoagulants becauseoxalate is toxic to the body

899
Q

cervix lymphatic drains into
internal iliac

external iliac

epigastric

deep inguinal

pelvic

A

A) Reason

Lymph DrainageFrom the fundus, the lymph vessels follow the ovarian arteryto the para-aortic nodes at the level of the first lumbar vertebra. From the body and the cervix, the lymph vessels draininto the internal and the external iliac nodes.

900
Q

level of left kidney”s hilum
L3

L2

L4

L5

L2-4

A

B) Reason

The right kidney (L3) is slightly lower than the left kidney (L2) because of the large sizeof the right lobe of the liver.

901
Q

bulbourethral gland is located at
pelvic pouch

pouch duglis

true pelvis

deep perineal pouch

false pelvis

A

D) Reason

CONTENTS OF THE DEEP PERINEAL POUCHIn males, the deep perineal pouch contains the following:• The membranous part of the urethra.• The sphincter urethrae.• The bulbourethral glands.• The deep transverse perineal muscles.• The internal pudendal vessels.• The dorsal nerves of the penis

902
Q

eversion of foot is action of
sartorius

adductor longus

peroneus longus

soleus

gastrocnemius

A

C) Reason

Inversion (movement of the foot so that the sole facesmedially): Tibialis anterior, extensor hallucis longus, medial tendons of extensor digitorum longus, and tibialis posterior muscles.• Eversion (opposite movement of the foot so that the solefaces laterally): Peroneus longus, peroneus brevis, peroneus tertius, and lateral tendons of extensor digitorumlongus muscles.

903
Q

Cardiolipin is present in
RER

SER

nucleus

cell membrane

mitochondria

A

E) Reason

The inner membrane is impermeable to most small ions (Na+, K+, H+) and small molecules (ATP, adenosine diphosphate, pyruvate). The impermeability is likely related to the high content of the lipid cardiolipin

904
Q

Pt. with dirrohea, pH 7.3 , HCO3 22, CO2 35, normal anion gap, Diagnosis?
respiratory acidosis

renal tubular acidosis

metabolic acidosis

respiratory alkalosis

Barter syndrome

A

B) Reason

Normal anion gap (8−12 mEq/L):HARD-ASS:HyperalimentationAddison diseaseRenal tubular acidosisDiarrheaAcetazolamideSpironolactoneSaline infusion

905
Q

Marked leucocytosis is indicative of?
plymghocytosis

inc. monocytes
inc. mast cells

neutrophilia

inc. Basophils

A

D) Reason

Acute appendicitis is an example of the acute inflammatory process, and cells that mediate an acute inflammation are neutrophils, while lymphocytes mediate the chronic inflamation.

906
Q

near gall bladder, anterior relation of portal vein
epiploic foramin

callot”s triangle

lesser sac

greater sac

lesser omentum

A

A) Reason

BOUNDARIES OF THE EPIPLOIC FORAMEN• Anteriorly: Free border of the lesser omentum, the bileduct, the hepatic artery, and the portal vein.• Posteriorly: Inferior vena cava.• Superiorly: Caudate process of the caudate lobe of theliver.• Inferiorly: First part of the duodenum.

907
Q

Pain in chest which agravates with respiration ,Reason?
myocardium

endocardium

pericardium

infarct

Pneumonia

A

C) Reason

Acute pericarditis Commonly presents with sharp pain, aggravated by inspiration, and relieved by sitting up and leaning forward. Presents with friction rub. ECG changes include widespread ST-segment elevation and/or PR depression.

908
Q

diabetic pt. on diuretics presents with hyperkalemia, cause?
thiazide

spironolactone

furosemide

manitol

Acetazolamide

A

B) Reason

TOXICITY Hyperkalemia (can lead to arrhythmias), endocrine effects with spironolactone (e.g., gynecomastia, antiandrogen effects.

909
Q

sesamoid bone is found in
femor

rectus femoris

Sartorius

psiform

thenar eminence

A

D) Reason

They are related to an articular or nonarticular bony surface, and the surfaces of contact are covered with hyaline cartilage and lubricated by a bursa or synovial membrane. Examples: patella, pisiform, fabella.

910
Q

Supply of flexor carpi ulnaris is
median

radial

axillary

posterior interossius

ulnar

A

E) Reason

It is supplied by muscular branches of ulnar nerve.

911
Q

Regarding immunology, ASO titer is used for the diagnosis of
Endocarditis

rheumatic fever

Myasthenia gravis

tyohioid fever

HIV

A

B) Reason

Associated with Aschoff bodies (granuloma with giant cells [blue arrows in A]), Anitschkow cells (enlarged macrophages with ovoid, wavy, rod-like nucleus, inc. ASO titers.Immune mediated (type II hypersensitivity); not a direct effect of bacteria. Antibodies to M protein cross-react with self antigens.

912
Q

which of following is result of parasympthatic activity?
dec. motility

dec. gut secretion
dec. sphincter
inc. glandular secretion

closed sphincter

A

D) Reason

Parasympathetic system is characterized by the rest and digest, at GIT it causes opening of sphincters, increase gut motility and increases gut secretions

913
Q

which anticoagulant is natural product?
warfarin

cumaidin

heparin

Clopidogrel

anti thrombin

A

C) Reason

Heparin: Heparin is a natural powerful anticoagulant,but its concentration in the blood is normally low, sothat only under special physiologic conditions does it have significant anticoagulant effects

914
Q

People with fish and meat diet with no fruit, will have deficiency of
vit. D

vit. C

B1

B6

B12

A

B) Reason

Vitamin C is found in fruits and vegetables. DEFICIENCY Scurvy—swollen gums, bruising, hemarthrosis, anemia, poor wound healing, perifollicular and subperiosteal hemorrhages, “corkscrew” hair. Weakened immune response.

915
Q

foramen for passage of accessory nerve
rotundum

jugular

spinosum

magnum

ophthalmic

A

B) Reason

The jugular foramen lies between the petrous part ofthe temporal bone and the condylar part of the occipitalbone. It transmits the glossopharyngeal, the vagus, and theaccessory nerves

916
Q

lesser omentum is attached to
pancreas

spleen

stomach

mesocolon

caudate lobe

A

E) Reason

The lesser omentum suspends the lesser curvature ofthe stomach to the fissure for the ligamentum venosum andthe porta hepatis and caudate lobe of the liver. The gastrosplenic omentum (ligament) connects the stomach tothe hilus of the spleen.

917
Q

Thrombus in which artery will result in infarct in visual centre?
anterior cerebral

posterior cerebral

vertebral

anterior communicating

middle meningeal

A

B) Reason

Visual cortex is located in the occipital lobe that is located in posterior cranial fossa supplied by the posterior cerebral artery.

918
Q

How is GFR measured in CKD pt.?
inulin

urea

PAH

PABA

Creatinine clearance

A

E) Reason

Estimates of GFR with blood urea nitrogen (BUN) and serum [creatinine]:■ Both BUN and serum [creatinine] increase when GFR decreases.Serum Creatinine clearance is used to measure GFR in chronic renal disease.■ In prerenal azotemia (hypovolemia), BUN increases more than serum creatinine and there is an increased BUN/creatinine ratio (>20:1).■ GFR decreases with age, although serum [creatinine] remains constant because of decreased muscle mass.

919
Q

Levator scapulae is innervated by?
C2

C4-5

C5-6

C1-3

C3-4

A

E) Reason

Levator scapulae is supplied by the spinal roots of the 3rd and 4th cervical nerves, Its function is to Raises the medial borderof the scapula.

920
Q

primary heart tube is developed at
4th week

5th week

7th week

8th week

10th week

A

A) Reason

Cardiac looping Primary heart tube loops to establish left-right polarity; begins in week 4 of gestation

921
Q

end product of purine metabolism
uric acid

urea

Creatinine

ammonia

CO2 and H2O

A

A) Reason

Lesch-Nyhan syndromeDefective purine salvage due to absent HGPRT, which converts hypoxanthine to IMP and guanine to GMP. Results in excess uric acid (end product of purine metabolism)production and de novo purine synthesis. X-linked recessive

922
Q

Small cell CA lung is associated with which of following
no basement membrane

no basal lamina

keratin nests

more elastin

more fibrin

A

C) Reason

Undifferentiated and very aggressive.Has keratin pearls on histology.May produce ACTH, ADH, or Antibodies against presynaptic Ca2+ channels (Lambert-Eaton myasthenic syndrome). Amplification of myconcogenes common. Inoperable; treat with chemotherap

923
Q

derivative of 2nd pharyngeal arch
hyoid bone

parathyroid

thyroid

thymus

stylohoid ligament

A

E) Reason

Derivatives of 2nd pharyngeal Arch: Muscles of facial expression: Stapes Plus: Styloid process Digastric (posterior belly) Lesser horn and UJ:Stylohyoid body of hyoid bone Stapedius.

924
Q

Which nerve damage will result in loss of lateral eye movement?
CN V

CN VIII

CN VI

CN VII

CN IV

A

C) Reason

It supplies the lateralrectus muscle. The abducent nerve is therefore responsiblefor turning the eye laterally.

925
Q

failure to rotate the anterior bud to the duodenum will result into
horseshoe kidney

pancreatic atresia

duodenal atresia

jejunal atresia

annular pancreas

A

E) Reason

A defect in the rotation and fusion of the ventral and dorsal buds results in an annular pancreas which can constrict or obstruct the duodenum and result in polyhydramnios.

926
Q

abnormal histological findings on 3.5cm above gastro-esophageal junction is
adeno CA

lymphoma

dysplasia

achlasia

Barret esophagus

A

E) Reason

Barrett esophagus Glandular metaplasia—replacement of nonkeratinized (stratified) squamous epithelium with intestinal epithelium (nonciliated columnar with goblets cells) in the distal esophagus. Due to chronic acid reflux (GERD).

927
Q

What should be V/Q for physiological shunt?
infinite

1

zero

2

maximum

A

C) Reason

V/Q =0 = airway obstruction (shunt). In shunt, 100% O2 does not improve Po2.V/Q =∞ = blood flow obstruction (physiologic dead space). Assuming < 100% dead space, 100% O2 improves Po2

928
Q

vulva is lined by which epithelium
simple cuboidal

stratified squamous

simple columner

simple squamous

keratinized

A

B) Reason

Stratified squamous epithelium is found in the oral cavity, Vulva pharynx, and esophagus (non-keratinized) and in the skin (keratinizing).

929
Q

Pt. sustains injury to nerve in axilla resulting in slight supination & flexion. Nerve damaged is?
radial

ulnar

axillary

muskulocutaneus

median

A

D) Reason

Musculocutaneous Nerve: Coracobrachialis, biceps brachii, and brachialis muscles; supplies the skin along the lateral border of forearm when it becomes the lateral cutaneous nerve of the forearm

930
Q

A 4 month ill looking and Lethargic child in ER with Na 145 and K 2.5, Dx..?
alkalosis

respiratory acidosis

ATN

renal tubular necrosis

ARF

A

D) Reason

Key finding: granular (“muddy brown”) casts A.3 stages:1. Inciting event2. Maintenance phase—oliguric; lasts 1–3 weeks; risk of hyperkalemia, metabolic acidosis3. Recovery phase—polyuric; BUN and serum creatinine fall; risk of hypokalemiaCan be caused by ischemic or nephrotoxic injury.

931
Q

Which nerve will be damaged in order to affect mass movement of intestine?
sympathetic

myenteric

parasympathetic

splanchnic nerves

aurbach plexus

A

E) Reason

Submucosa—includes Submucosal nerve plexus (Meissner)- secretionsMuscularis externa—includes Myenteric nerve plexus (Auerbach)- gut motility.

932
Q

anterior superior iliac spine is fractured. which muscle is avulsed?
adductor longus

soleus

gastrocnemius

sartorius

rectus femoris

A

D) Reason

Sartoriua is long and strong muscle and it takes origin from the anterior superior Iliac spine (ASIS). Its avulsion can cause fracture of ASIS.

933
Q

Due to trauma to temple region pt. suffered headache & loss of vision. Lesion is at?
anterior meningeal artery

middle miningeal artry

vertebral artery

posterior cerebral

anterior communicating

A

B) Reason

The middle meningeal artery ascends between theroots of the auriculotemporal nerve to enter the skull throughthe foramen spinosum. It runs laterally within the skull anddivides into anterior and posterior branches. The anteriorbranch is particularly important, because it lies close to themotor area of the cerebral cortex. It injury can cause loss of vision.

934
Q

boy fell outstretched hands, can”t abduct arm upto 40 degree, but assisted to 40 then he can abduct overhead, muscle involved is
bicep

deltioid

brachialis

supraspinatus

tricep

A

D) Reason

Abduction: Middle fibers of the deltoid muscle, assistedby the supraspinatus muscle (overhead abduction).

935
Q

Anterior relation of CBD to duodenum
2nd

3rd

4th

1st

head of pancreas

A

D) Reason

It descends in front of theopening into the lesser sac and passes behind the first partof the duodenum and then the head of the pancreas. Thebile duct ends below by piercing the medial wall of the duodenum approximately halfway down its length. Itis usually joined by the main pancreatic duct, and, together,they open into a small ampulla in the duodenal wall calledthe ampulla of Vater

936
Q

dilated vessel around umbilicus, vessel compromised?
portal vein

spleenic vein

pancreatic vein

epigastric vein

umbilical vein

A

D) Reason

Clinically, theenlarged subcutaneous anastomosis between the lateral thoracic vein, a tributary of the axillary vein, andthe superficial epigastric vein, a tributary of thefemoral vein, may be seen on the thoracoabdominalwall.

937
Q

Absorbance of csf is carried out at?
Arachnoid plexus

subdural place

Arachnoid villi

epidural space

sub Arachnoid space

A

C) Reason

Choroid plexus secretes CSF into all ventricles. Arachnoid granulations (Arachnoid villi) are sites of CSF resorption

938
Q

median nerve damaged before flexor retinaculum, there is sensory loss at thenar, what other loss will be found?
opposition of thumb

flexion of thumb

extension of thumb

addiction of thumb

abduction of thumb

A

A) Reason

MEDIAN NERVE AND RESTRICTED SPACEIn the carpal tunnel the median nerve lies in a restricted space between the flexor digitorum superficialis and the flexor carpi radialis muscles. It may become compressed at this site. Manifests as loss of sensation of thenar eminence and loss of opposition of thumb.

939
Q

Which nerve damage results in adduction & lateral rotation of thumb?
radial

ulnar

median

musculocutaneus

median

A

E) Reason

Anterior interosseousbranch of themedian nerve supplies the flexor pollisus longus, its damage can cause addiction and lateral rotation of thumb.

940
Q

which nerve emerges from pons
CN V

CN V-VII

CN V-VI

CN 6-8

CN 5-8

A

E) Reason

Nerves originating from Pons are: V. The trigeminal (mixed).VI. The abducent (motor).VII. The facial (mixed).VIII. The vestibulocochlear (sensory).

941
Q

Smudge cells are characteristics of
ALL

CML

ALL

CLL

multiple myeloma

A

D) Reason

CLL: Age: > 60 years. CD20+, CD5+ B-cell neoplasm. Often asymptomatic, progresses slowly; smudge cells in peripheral blood smear; autoimmune hemolytic anemia.

942
Q

What must be depleted for thirst derive?
ICF fluid

ECF fluid

dec. ECF Osmolarity
inc. ICF Osmolarity

hypertension

A

B) Reason

When there is decreased Blood volume or ECF and in case of increased ECF Osmolarity, thirst center is actived in order to take ECF Osmolarity and ECF volume towards normal.

943
Q

if anterior cruciate ligament is damaged lateral condyle of femur & tibia will move to?
anteroposterior

posterior

lateral

anterolateral

anterior

A

E) Reason

The anterior cruciate ligament is attached below tothe anterior intercondylar area of the tibia, and itpasses upward, backward, and laterally to be attached to thelateral femoral condyle.

944
Q

supply of cremasteric reflex is
ilioinguinal

genitofemoral

pudendal

cremasteric

pelvic

A

B) Reason

Genitofemoral: Cremaster muscle in scrotum in male;nerve (L1, 2) skin over anterior surface of thigh;nervous pathway forcremasteric reflex

945
Q

Supply of root of tongue is
tractus solitarius

vagus

CN IX

CN VII

CN VIII

A

A) Reason

SENSORY INNERVATION• Anterior two thirds: The lingual nerve (general sensation) and chorda tympani (taste).• Posterior third: The glossopharyngeal nerve, tractus solitarius nucleus(general sensation and taste).

946
Q

which nerve is bound to submandibular gland?
mandibular nerve

maxillary nerve

ophthalmic nerve

lingual nerve

facial nerve

A

D) Reason

The submandibular ganglion is a parasympathetic ganglionthat lies deep to the submandibular salivary gland and is attached to the lingual nerve by small nerves.

947
Q

A 29 year old male is treated for fever, now presented with jaundice and painful legs, Diagnosis?
Hemolytic crisis

Aplastic crisis

CML

ALL

CLL

A

A) Reason

Most common enzymatic disorder of RBCs.X-linked recessive.Defect in G6PD: dec. glutathione inc. RBC susceptibility to oxidant stress. Hemolytic anemia following oxidant stress (classic causes: sulfa drugs, antimalarials, infections, fava beans)

948
Q

histology of tissue with less smooth muscles
cervix

uterus

Fallopian tube

ovary

vulva

A

A) Reason

Ectocervix Stratified squamous epithelium, nonkeratinizedEndocervix Simple columnar epithelium-less smooth muscles.Transformation zone Squamocolumnar junction (most common area for cervical cancer)

949
Q

which is important during 2nd stage of labor
dilatation

effacement

descent

contraction

involution

A

D) Reason

During 1st stage, cervical dilatation and effacement is important. During 2nd stage, utrine contraction is important. While during 3rd stage utrine contraction and involution is important.

950
Q

is developed in which week
3rd

4th

2nd

5th

6th

A

A) Reason

Third week: Gastrulation and early development of nervous and cardiovascular systems; corresponds to first missed period.

951
Q

respiration is directly proportional to
alveoli

alveolar ducts

alveoli sac

surfactant

alveoli surface area

A

E) Reason

Diffusion: Vgas = A/T × Dk(P1 – P2) where A = area, T = thickness, and Dk(P1 – P2) ≈difference in partial pressures: A dec. in emphysema. T inc. in pulmonary fibrosis.

952
Q

If adductor canal is compressed which structure will be damaged?
femoral

fibular

peroneal

deep femoral artery

superficial femoral artry

A

E) Reason

Theartery descends through the femoral triangle andthe adductor canal (can be damaged by compression over the canal), and it leaves the front of the thigh by passing through the opening in the adductor magnus and then entering the popliteal space as the popliteal artery.

953
Q

urine conc. is 100, urine flow rate 2 and plasma conc. is 10, find renal clearance?
2

10

50

2000

20

A

E) Reason

Renal clearance Cx = UxV/Px = volume of plasma from which the substance is completely cleared per unit time. = 100 x 2 / 10= 20.

954
Q

which is site of secretary proteins synthesis
SER

mitochondria

golgi complex

nucleus

RER

A

E) Reason

Rough endoplasmic reticulum:Site of synthesis of secretory (exported) proteins and of N-linked oligosaccharide addition to many proteins.

955
Q

inc. conductance in ventricles have increased conductance because of?
less muscles

more muscle

purkinji fibers

less fibrin

more fibrin

A

C) Reason

Speed of conduction—Purkinje > atria > ventricles > AV node.Pacemakers—SA > AV > bundle of His/Purkinje/ventricles

956
Q

IL1 cause fever. How?
TNF

proatacyclin

fibrin

prostagladins

elastin

A

D) Reason

Fever1. Pyrogens increase the production of interleukin-1 (IL-1) in phagocytic cells. ■ IL-1 acts on the anterior hypothalamus to increase the production of prostaglandins. Prostaglandins increase the set-point temperature, setting in motion the heat-generating mechanisms that increase body temperature and produce fever.

957
Q

which muscle does no part in wrist?
brachialis

bracheoradialis

supinator

bicep

flexer carpi radialis

A

B) Reason

Brachioradialis is supplied by Radial nerve. It Flexes the forearm at theelbow joint, rotates theforearm to the midproneposition

958
Q

active thyroid hormone is
T3

T4

free form

protein bound

fat bound

A

C) Reason

T4 is major thyroid product; converted to T3 in peripheral tissue by 5”-deiodinase.Free form of T3 (active form) binds receptors with greater affinity than T4.Peroxidase is enzyme responsible for oxidation and organification of iodide as well as coupling of monoiodotyrosine (MIT) and diiodotyrosine (DIT).

959
Q

Thrombus in which vessel will result in infarct of lower segment of heart?
RCA

LXA

posterior branch

LCA

marginal branch

A

D) Reason

Left anterior descending artery (LAD) Branch of LCA—supplies anterior 2/3 of interventricular septum, anterior papillarymuscle, and anterior surface of left ventricle

960
Q

blood supply of lungs
pulmonary artery

bronchial arteries

alveolar artry

splanchnic artery

pleural artery

A

B) Reason

BLOOD SUPPLY OF THE LUNGSThe bronchi, connective tissue, and visceral pleura are supplied by the bronchial arteries

961
Q

Prostagladins D2, E2 and F2 features are
Vasodilation

vasoconstriction

mediate inflammation

clear debris

for pain

A

A) Reason

LTC4, D4, and E4 function in bronchoconstriction, vasoconstriction, contraction of smooth muscle, and inc. vascular permeability.PGI2 inhibits platelet aggregation and promotes vasodilation

962
Q

syndemosis is present in
at sutures

teeth

vertebrae

knee joint

b/w radius & ulna

A

E) Reason

The bones are connected by the interossesous ligament . Example includes radioulnar joint and inferior tibiofibular joint.

963
Q

Among the following, which of following has no graft rejection after transplant
HIV

CML

Aplastic crisis

Aplastic anemia

SCID

A

E) Reason

Severe combined immunodeficiency (SCID): Failure to thrive, chronic diarrhea, thrush. Recurrent viral, bacterial, fungal, and protozoal infections.Treatment: bone marrow transplant (no concern for rejection).

964
Q

internal jugular vein on emerging from jugular foramen is related anteriorly to?
hypoglossal

axillary nerve

acessary nerve

axillary artery

Subclavian vein

A

C) Reason

The internal jugular vein is a large vein that drains bloodfrom the brain, face, scalp, and neck. It starts as acontinuation of the sigmoid venous sinus and leaves theskull through the jugular foramen. It then descends throughthe neck in the carotid sheath lateral to the vagus nerve andanterior to the acessary nerve and the internal and the common carotid arteries.

965
Q

acid production activity is done by?
H2 receptor

H1 receptor

gastrin

parietal mucosa

sympathetic nerve

A

A) Reason

■ stimulates H+ secretion by activating H2 receptors on the parietal cell membrane.■ The H2 receptor is coupled to adenylyl cyclase via a Gs protein.■ The second messenger for histamine is cAMP.■ H2 receptor–blocking drugs, such as cimetidine, inhibit H+ secretion by blocking the stimulatory effect of histamine

966
Q

Patient in ICU has to shift on vent. what is necessary to tell to patient/attendant
detail of procedure

consent

keep them in ICU

take to home

do what is necessary without communicating

A

A) Reason

In order to ensure the best communication between the Dr. and Patients, it is necessary to explain the procedure.

967
Q

After surgery of a 37 year old male HIV patient, how to clean the theater
alcohol

formaldehyde

UV light

gamma rays

2 % glutraldehyde

A

E) Reason

Glutaraldehyde, which has two reactive aldehyde groups, is 10times more effective than formaldehyde and is less toxic. In hospitals, it is used tosterilize respiratory therapy equipment, HIV and HBV used instruments, endoscopes, and hemodialysis equipment.

968
Q

spinal nerves are
sensory

motor

double supply

no ganglion

mixed

A

E) Reason

Spinal nerves: There are mixed nerves, 31 nerves in total: 8 cervical, 12 thoracic, 5 lumbar, 5 sacral, 1 coccygeal.Nerves C1–C7 exit above the corresponding vertebra. All other nerves exit below (e.g., C3 exits above the 3rd cervical vertebra; L2 exits below the 2nd lumbar vertebra).

969
Q

external carotid artery terminates at
parotid gland

Submandibular gland

Sublingual gland

mandibular angle

mandibular body

A

A) Reason

External Carotid ArteryThe external carotid artery begins at the level of the upperborder of the thyroid cartilage as one of the terminalbranches of the common carotid artery. It ascendsto terminate in the parotid salivary gland behind the neck ofthe mandible, where it divides into the superficial temporaland the maxillary arteries.

970
Q

carcinoma compressing optic tract causes
unilateral blindness

ipsilatral blindness

homonymous hemianopia

bitemporal hemianopia

contralatral loss

A

C) Reason

Visual field defects: 1. Ipsilatral anopia (optic nerve).2. Bitemporal hemianopia (pituitary lesion, chiasm)3. contralatral omonymous hemianopia ( optic tract lesiong

971
Q

level of body of hyoid bone
C1

C3

C2

C4

C3-5

A

B) Reason

The hyoid bone may be felt a few centimetres below and behind the chin, especially if the neck isextended. It may be palpated between finger and thumb and moved from side to side. The hyoid bone liesapproximately at the level of the third cervical vertebra.

972
Q

most abundant cells in respiratory tree
type 2 cells

clara cells

Type 1 cells

squamous cells

cuboidal cells

A

C) Reason

Type I cells 97% of alveolar surfaces. Line the alveoli. Squamous; thin for optimal gas diffusion

973
Q

which Hb is absent in fetal blood
alpha chain

gamma chain

delta chain

beta chain

theta chain

A

D) Reason

Hemoglobin development:Fetal hemoglobin (HbF) = α2γ2.Adult hemoglobin (HbA) = α2β2.HbF has higher affinity for oxygen due to less avid binding of 2,3-BPG. This allows HbF to extract oxygen from (HbA) maternal hemoglobin across the placenta.

974
Q

No. of lactiferous ducts are found in mammary gland?
13-15

15-20

18-20

20-25

25-30

A

B) Reason

Each gland consists of 15 to 20 lectiferous ducts that radiate outward from the nipple. There is no capsule. Each lobe is separated from its neighbor by connective tissue septa that extend from the skin to the deep fascia and that serve assuspensory ligaments. Th

975
Q

visible intercostal pulsation with notching on X-ray is found. Diagnosis
pneumonia

pericarditis

aortic arch aneurysm

mitral stenosis

aortic stenosis

A

C) Reason

Associated with cystic medial degeneration due to hypertension (older patients) or Marfan syndrome (younger patients). Presents with intercostal pulsation with notching on X-ray chest. Also historically associated with 3° syphilis (obliterative endarteritis of the vasa vasorum).

976
Q

pulmonary pressure and volume curve shows
ventilation

compliance

perfusion

circulation

pressure

A

B) Reason

Compliance—change in lung volume for a given change in pressure; dec. in pulmonary fibrosis, pneumonia, and pulmonary edema; inc. in emphysema and normal aging

977
Q

cremesteric fascia is derived from
transversus abdominis

external oblique

transversalis facia

internal oblique

conjoint tendon

A

D) Reason

Covering layers of the spermatic cord are from anterior abdominal wall:• External spermatic fascia derived from the externaloblique muscle and attached to the margins of the superficial inguinal ring.• Cremasteric fascia derived from the internal obliquemuscle.• Internal spermatic fascia derived from the fasciatransversalis and attached to the margins of the deep inguinal ring.

978
Q

Relation of uterine artery, after arising from internal iliac
ureter

Fallopian tube

ovary

femoral artery

Pudendal nerve

A

A) Reason

Uterine artery: This artery runs medially on the floor ofthe pelvis and crosses the ureter superiorly. Itpasses above the lateral fornix of the vagina to reach theuterus, where it ascends between the layers of the broadligament along the lateral margin of the uterus.

979
Q

cervical smear is showing stratified squamous epithelium, cause
anaplasia

Metaplasia

dysplasia

hyperplasia

hypoplasia

A

B) Reason

Disordered epithelial growth; begins at basal layer of squamocolumnar junction (transition zone) and extends outward. Classified as CIN 1, CIN 2, or CIN 3 (severe dysplasia or carcinoma in situ), depending on extent of dysplasia. Associated with HPV 16 and HPV 18, which produce both the E6 gene product (inhibits p53 suppressor gene) and E7 gene product (inhibits RB suppressor gene)

980
Q

Tyoe of joint between intervertebral discs
amphiarthrosis

syndemosis

gomphosis

symarthrosis

chondrosis

A

A) Reason

Amphiarthroses are joints at which slight movement is possible. A pad of cartilage lies between the bone surfaces, and there are fibrous ligaments to hold the bones and cartilage in place. The cartilages of such joints also act as shock absorbers, e.g. the intervertebral discs

981
Q

In patient with chronic recurrent typhoid relapses, Typhoid carrier stage is
obstructive

jaundice

gall stones

cholangitis

asymptomatic

A

E) Reason

Carrier state of S. typhi resides in gall bladder, and is asymptomatic state of the typhoid fever.

982
Q

dermatome of thumb
C1

C2

C6

C7

C8

A

C) Reason

C2—posterior half of a skull “cap.”C3—high turtleneck shirt.C4—low-collar shirt. C5–deltoid, C6–lateral one and half digits.

983
Q

adduction of wrist is combined effort of flexor carpi ulnaris and
flexor carpi radialis

extensor pollicis

extensor indicis

brachialis

extensor carpi ulnaris

A

E) Reason

Abduction: Flexor carpi radialis, extensor carpi radialislongus and brevis, abductor pollicis longus, and extensorpollicis longus and brevis muscles.• Adduction: Flexor and extensor carpi ulnaris muscles.

984
Q

In Pathogenesis of Atherosclerosis, 1st gross sign of atheroma
platelet adhesion

platelet plug

embolism

endothelial injury

fatty streaks

A

E) Reason

The atheromas can develop from the fatty streak, a lesion that is characterized by focal accumulations in the intima of lipid-laden foam cells that may appear as early as the first year of life and that is present in the aorta of most older children.

985
Q

In a 27 year old male patient, which is first step related to thrombin generation
platelet plug

fibrin clots

Elastin fibers

platelet adhesion

macrophages deposits

A

D) Reason

Adhesion (a) Vessel injury exposes subendothelial collagen, leading to platelet adhesion (adherence to the subendothelial surface) . (b) Interaction of specific platelet-surface glycoprotein receptors and subendothelial collagen is mediated by von Willebrand factor.

986
Q

2 posterior cerebral artery are derived from
middle meningeal

anterior cerebral

occipital

basilar artry

vertebral artery

A

D) Reason

The basilar artery ascends in a groove on theanterior surface of the pons. It gives off branches to thepons, the cerebellum, and the internal ear. It finally dividesinto the two posterior cerebral arteries.

987
Q

muscle which remains intact on little finger injury
lumbricals

extensor indicis

dorsal interossous

flexor carpi ulnaris

extensor carpi radialis

A

C) Reason

Anatomically, there are only four dorsal interossi from 1st digit to the 4th one. Hence dorsal interossous will not be affected in little finger (5th digit) injury.

988
Q

deep cervical artey is branch of
internal thoracic

costocervical

vertebral

internal memory

Subclavian

A

B) Reason

The costocervical trunk runs backward over the dome ofthe pleura and divides into the superior intercostalartery, which supplies the first and the second intercostalspaces, and the deep cervical artery, which supplies thedeep muscles of the neck.

989
Q

Among the following, foam cells are seen in
fatty streaks

atheroma

embolism

thrombi

thrombosis

A

B) Reason

The atheromas can develop from the fatty streak, a lesion that is characterized by focal accumulations in the intima of lipid-laden foam cells that may appear as early as the first year of life and that is present in the aorta of most older children

990
Q

fracture of sub occipital bone, artery involved
basilar

occipital

vertebral

posterior cerebral

post. communicating

A

C) Reason

The vertebral artery ascends in the neck through theforamina in the transverse processes of the upper six cervical vertebrae . It supplies the occipital lobe.

991
Q

strongest ligament of body
genitofemoral

quadrate ligament

iliofemoral

broad ligament

Ligamentum teres

A

C) Reason

Iliofemoral LigamentThe iliofemoral ligament is the strongest and most importantligament of the hip joint . It is shaped like an inverted Y. Its base is attached to the anteroinferior iliac spineabove, and the two limbs of the Y are attached to the upperand the lower parts of the intertrochanteric line of the femur

992
Q

A 15 yr old boy died of Acute rheumatic fever, cause of death is due to
pericarditis

Endocarditis

valvular damage

autoimmune disorder

myocarditis

A

E) Reason

Rheumatic fever:A consequence of pharyngeal infection with group A β-hemolytic streptococci. Early deaths due to myocarditis. Late sequelae include rheumatic heart disease, which affects heart valves—mitral > aortic&raquo_space; tricuspid (high-pressure valves affected most).

993
Q

A 48 year old male is suspected to have GB syndrome, type of hypersensitivity in GB syndrome
type 3

type 4

delayed response

type 1

type 2

A

B) Reason

Type IV: Multiple sclerosisGuillain-Barré syndromeGraft-versus-host diseasePPD (test for M. tuberculosis)Contact dermatitis (e.g., poison ivy, nickel allergy)

994
Q

pain fibers to uterus travel via
Cruciate ligament

Ligamentum teres

uterine ligament

broad ligament

quadrate ligament

A

D) Reason

Each ovary is attached to the back of the broad ligament bythe mesovarium . The ovarian vessels and nerves(through the broad ligament)enter the ovary at the hilum

995
Q

right sided hemiplegia with left sided mouth deviation, lesion
medulla

Midbrain

basal ganglia

uncus

pons

A

E) Reason

Facial nerve nucleus arises from the Pons, in lesions of the Pons the will be hemiplegia and the facial nerve palsy of the other side.

996
Q

In a 39 year old lady with bile obstruction, after cholecystectomy, which is raised
AST

ALT

ALP

bilirubin

amylase

A

C) Reason

In an any Inflammatory or the obstruction of the billiary tract, there is raised level of the ALP, like in choledocholithiasis, cholangitis and obstruction.

997
Q

unable to extend MCP joint and abduct thumb but sensation intact, nerve damaged
ulnar

median

musculocutanous

deep radial

Posterior interossious branch of radial nerve

A

E) Reason

Extensor pollicis longus Extends the distal phalanxof the thumbExtensor indicis Extends the metacarpophalangeal joint of theindex finger, Both are supplied by deep branch of radial nerve.

998
Q

If left lower lobe of thyroid is excised, nerve damage
external laryngeal

recurrent laryngeal

superior laryngeal

inferior laryngeal

vagus

A

A) Reason

The external laryngeal nerve is motor; it is located closeto the superior thyroid artery and supplies the cricothyroidmuscle. It is mostly injured in lower lobe thyroid surgery.

999
Q

foramen ovale & otic ganglion are related as
superior

lateral

anterior

below

posterior

A

D) Reason

Otic GanglionThe otic ganglion is a parasympathetic ganglion that is located medial to the mandibular nerve just below the skull,and it is inferior to the foramen ovale and adherent to the medial pterygoid muscle.

1000
Q

A 35 years old male history of with fever and lymphadenopathy, H/O sleeping outside and bitten by flies, Dx..?
malaria

giardiasis

onchocercosis

yellow fever

kalazar

A

E) Reason

Leishmania donovani: Transmitted by Sandfly, causes Visceral leishmaniasis (kala-azar): spiking fevers, lymphadenopathy,hepatosplenomegaly, pancytopenia.

1001
Q

student observed a cell with its organelle in dividing phase, this is
RER

SER

mitochondria

ribosome

peroxisome

A

C) Reason

Mitochondria are the cellular organelle that have their own DNA as their genetic material and hence can replicate their own.

1002
Q

left renal vein is related to
posterior to aorta

anterior to aorta

anterior to IVC

posterior to IVC

lateral to aorta

A

B) Reason

Vein:The renal vein, right one is slightly higher and just anterior to aorta, drains into the inferior vena cava.

1003
Q

Pt. with CA rectum, pain radiates to posterior of thigh, nerve involved
lumbar plexus

Pudendal nerve

sacral plexus

pelvic nerve

Obturator nerve

A

C) Reason

Branches of Sacral plexus: The sciatic nerve (L4 and 5; S1, 2, and 3) is the largestnerve in the body and the largest branch of the sacralplexus.• The superior gluteal nerve supplies the gluteus medius,the gluteus minimus, and the tensor fasciae latae muscles.• The inferior gluteal nerve supplies the gluteus maximusmuscle.• The nerve to the quadratus femoris muscle also supplies the inferior gemellus muscle.• The nerve to the obturator internus muscle also supplies the superior gemellus muscle.• The posterior cutaneous nerve of the thigh suppliesthe skin of the buttock and the back of the thigh.

1004
Q

which is COX-2 inhibitor
aspirin

Ketorolac

phenylbutone

endomethicin

celecoxib

A

E) Reason

COX-2 inhibitors (celecoxib)MECHANISM: Reversibly inhibit specifically the cyclooxygenase (COX) isoform 2, which is found in inflammatory cells and vascular endothelium and mediates inflammation and pain; spares COX-1, which helps maintain the gastric mucosa.

1005
Q

loop diuretics act on
PCT

DCT

loop of Henle

descending limb

ascending limb

A

E) Reason

MECHANISM: Sulfonamide loop diuretic. Inhibits cotransport system (Na+/K+/2 Cl-) of thick ascending limb of loop of Henle. Abolishes hypertonicity of medulla, preventing concentration of urine. Stimulates PGE release (vasodilatory effect on afferent arteriole); inhibited by NSAIDs.

1006
Q

what to check to assess nutritional status
protein content

body fats

carbohydrates

liver size

adipose tissue

A

A) Reason

Proteins are nuitrional constituents that are present in different parts of the body and are the essential part of the balanced diet. Nuitrional assessment is checked by body proteins contents (kwashirkow etc)

1007
Q

true about nucleolus
no RNA

least RNA

more proteins

high conc. of RNA

no DNA

A

D) Reason

Nucleolus: The nucleolus is responsible for ribosomal RNA (rRNA) synthesis and ribosome assembly. It contains high concentration of the RNA.

1008
Q

Among the following, the most reliable sign of irreversible cell injury
hyperemia

Apoptosis

nuclear atypia

Hypoxia

autophagy

A

E) Reason

Autophagy of cellular components involves generation of autophagic vacuoles. These vacuoles fuse with lysosomes whose hydrolytic enzymes breakdown cellular components. It is an irreversible cell injury.

1009
Q

which anterior pituitary hormone is inhibited by somatostatin?
LH

calcitonin

GH

TSH

MSH

A

C) Reason

REGULATION Released in pulses in response to growth hormone–releasing hormone (GHRH). Secretion inc. during exercise and sleep. Secretion inhibited by glucose and somatostatin.

1010
Q

renal column consists of
lobular artery

glomerulus

interlobar artery

peritubular capillaries

perilobar capillaries

A

C) Reason

Blood Circulation The renal artery enters the kidney at the hilum, near the ureter. The artery branches into interlobar arteries, which travel to the medulla-cortex border remaining outside the medullary pyramids.

1011
Q

head of pancreas is related posteriorly to?
aorta

hilum

colon

IVC

dudenum

A

D) Reason

The head is disc shaped and lies withinthe concavity of the C-shaped duodenum just posterior to the IVC.

1012
Q

Regarding neoplasms, which form of carcinoma uses metalloproteinases
early

advance

metastasis

invasive

pleomorphism

A

D) Reason

Invasive carcinoma• Cells have invaded basement membrane using collagenases and hydrolases (metalloproteinases)• Can metastasize if they reach a blood or lymphatic vessel

1013
Q

at Mcburny incision which nerve is likely to be compromised?
genitofemoral

iliohypogastric

ilioinguinal

Pudendal

sacral plexus

A

B) Reason

Iliohypogastric Nerve (L1) External oblique, internal oblique,transversus abdominis musclesof anterior abdominal wall;skin over lower anteriorabdominal wall over the ileac fossa and buttock.

1014
Q

epiploic foramen is formed inferiorly by?
jejunum

ilium

colon

duodenum

pancreas

A

D) Reason

BOUNDARIES OF THE EPIPLOIC FORAMEN• Anteriorly: Free border of the lesser omentum, the bileduct, the hepatic artery, and the portal vein.• Posteriorly: Inferior vena cava.• Superiorly: Caudate process of the caudate lobe of theliver.• Inferiorly: First part of the duodenum.

1015
Q

In a 59 year old female, which LNs involved in cervical cancer stage IV
internal iliac

external illeic

superficial inguinal

deep inguinal

hypogastric

A

D) Reason

Lymph DrainageFrom the fundus, the lymph vessels follow the ovarian arteryto the para-aortic nodes at the level of the first lumbar vertebra. From the body and the cervix, the lymph vessels draininto the internal and the external iliac nodes. In addition, afew lymph vessels pass through the inguinal canal to the superficial and deep (especially in late stage of malignancy) inguinal nodes.

1016
Q

Pt has seizures and excitatory muscle contraction with Ca 6, glucose 120 and PTH 13, Diagnosis.?
hyperparathyriod

Hyperkalemia

tetany

hypoparathyroid

hypophosphtemia

A

C) Reason

hypocalcemia–Tetany:Chvostek sign—tapping of facial nerve (tap the Cheek) inc. contraction of facial muscles.Trousseau sign—occlusion of brachial artery with BP cuff (cuff the Triceps) lead yo carpal spasm

1017
Q

45% Hematocrit mean, 45 percent of blood volume is formed by
RBC

wbc

platelet

All blood cells

cell & proteins

A

D) Reason

Hematocrit Mean: is the percentage of blood volume that is consisted of the blood cellular corpuscles, largely contributed by RBCs.

1018
Q

extensor carpi ulnaris is innervated by
ulnar

radial

axillary

median

posterior interossous

A

E) Reason

interosseous nerve which is continuation of deep branch of radial nerve is the correct answer.

1019
Q

Which segment of nephron sense fluid load?
PCT

macula densa

DCT

loop of Henle

thick limb

A

B) Reason

Tubuloglomerular feedback, in which increased renal arterial pressure leads to increased delivery of fluid to the macula densa. The macula densa senses the increased load and causes constriction of the nearby afferent arteriole, increasing resistance to maintain constant blood flow

1020
Q

what phenomenon is carried out at dead space?
conduction

convection

radiation

ventilation

perfusion

A

A) Reason

physiologic dead space = anatomic dead space of conducting airways plus functional dead space in alveoli; apex of healthy lung is largest contributor of functional dead space. Volume of inspired air that does not take part in gas exchange.

1021
Q

poorly differentiated cells are associated with
pleomorphism

anaplasia

monomorphism

heteromorphism

dimorphism

A

A) Reason

Morphologic manifestations include disorderly maturation and spatial arrangement of cells; marked variability in nuclear size and shape (pleomorphism) ; and increased, often abnormal, mitosis.

1022
Q

platelet aggregation is inhibbited aggressively by?
PGE2

PGD2

PGA2

PGC4

PGI2

A

E) Reason

Anti-aggregation factors:PGI2 and NO (released by endothelial cells) ↑ blood flow ↓ platelet aggregation

1023
Q

about kidney fascia which is right?
covers its gland

deficient anteriorly

deficient lateraly

covers hilum

covers

A

A) Reason

Renal fascia: This is a condensation of areolar tissue outside the perirenal fat. It encloses the kidneys and thesuprarenal glands

1024
Q

portal hypertension involves which structure?
IVC

spleenic vein

hepatic vein

portal vein

mesenteric vein

A

D) Reason

Portal hypertension is characterized by the development of venous collaterals with varices in the submucosal veins of the esophagus, the hemorrhoidal plexus, and other sites. This condition is often classified by the site of portal venous obstruction

1025
Q

inferior wall MI is result of thrombus in which artery?
left marginal

right marginal

LAD

LCA

LXA

A

B) Reason

Acute marginal artery—is on the right side and is the branch of the RCA, itsupplies right ventricleand the inferior surface of the heart.

1026
Q

Only one parent is sufferer in which genetic trait
X-linked recessive

autosomal recessive

autosomal dominant

X-linked dominant

multifactorial

A

C) Reason

Autosomal dominant: Often due to defects in structural genes. One of the parents is affected to transfer the trait. Many generations, both male and female, affected.

1027
Q

In a 53 year old female with CA breast, grade 4 tumor mean which percent of undifferentiated cells
Metaplasia

dysplasia

atrophy

anaplasia

hypertrophy

A

D) Reason

Grade: Determined by degree of cellular differentiation and mitotic activity on histology. Usually graded 1–4; 1 = low grade, well differentiated; 4 = high grade, poorly differentiated, anaplastic.

1028
Q

In hot weather atmospheric temp exceeds body temp. heat loss by?
sweating

conduction

convection

radiation

exhausion

A

A) Reason

When the atmosphere temperature is greater than the body temperature, sympathetic system activation causes periphral Vasodilation and sweating to loose extra heat.

1029
Q

which vitamin Excess of which vitamin will result in renal calcium calculi
vit. E

vit. B2
vit. B3
vit. C
vit. D

A

E) Reason

Vitamin D. As 70%Stones are calcium stones so factor that will inc calcium will be responsible for it so vit D is correct answer.

1030
Q

which part of nephron is responsible for absorption of glucose & aminoacids?
DCT

loop of Henle

thick limb

collecting duct

PCT

A

E) Reason

Early proximal convoluted tubule (PCT)—contains brush border. Reabsorbs all of the glucose and amino acids and most of the HCO3–, Na+, Cl–, PO43–, K+, and H2O. Isotonic absorption.

1031
Q

Pt. is unable to rotate eye. Reason
sinus thrombus

optic chiasm fracture

pituitary tumor

sella Turcica fracture

cavernous sinus infection

A

E) Reason

It passes forward through the cavernoussinus (mostly affected by sinus infection) with the internal carotid artery, enters the orbitthrough the superior orbital fissure, and supplies the lateralrectus muscle. The abducent nerve is therefore responsiblefor turning the eye laterally.

1032
Q

polyarteritis nodosa is an example of
delayed response

type 1 HSR

type 2 HSR

type 4 HSR

type 3 HSR

A

E) Reason

Type III SLEPolyarteritis nodosaPoststreptococcal glomerulonephritisSerum sicknessArthus reaction (e.g., swelling and inflammation following tetanus vaccine).

1033
Q

features of SCC
Elastin

keratin

fibrin

Ca

Excess atypia

A

B) Reason

SCC: Hilar mass arising from bronchus; Cavitation; Cigarettes; hyperCalcemia (produces PTHrP).Keratin pearls and intercellular bridges

1034
Q

insulin is antagonized by
GH

LH

FSH

glucagon

Somatostatin

A

E) Reason

REGULATION Glucose is major regulator of insulin release. GH (causes insulin resistance inc. insulin release) and β2-agonists inc. insulin. One potent inhibitor of insulin is Somatostatin.

1035
Q

visceral layer of serous pericardium forms?
Endocarditis

pericardium

epicardium

pleura

tracheal

A

C) Reason

Layers of the Heart Wall:The heart wall is composed of 3 distinct layers: an outer epicardium, a middle myocardium and an inner endocardium. The epicardium, or visceral layer of serous pericardium, consists of a simple squamous epithelium (mesothelium) and its underlying connective tissue

1036
Q

inferior cervical ganglion combined with 1st thoracic ganglion to form?
otic

ciliary

optic

stellate

salivary

A

D) Reason

In most people, the inferior cervical ganglion is fused to thefirst thoracic ganglion to form the stellate ganglion. It is located between the transverse process of the seventh cervical vertebra and the neck of the first rib.

1037
Q

hippocampus injury causes what
memory loss

aphasia

agraphesthesia

asteriognosis

aphonia

A

A) Reason

Hippocampus (bilateral):Anterograde amnesia—inability to make new and short termmemories

1038
Q

Regarding genetics, most beneficial role of p53
prevent inflamation

anticoagulant

prevent overcrowding

anti TNF

anti inflammatory

A

C) Reason

p53 and hypophosphorylated Rb normally inhibit G1-to-S progression; mutations in these genes result in unrestrained cell division (e.g., Li-Fraumeni syndrome). p53 also prevents the overcrowding if the cells.

1039
Q

Of the following, which Non lymphoid lesion that can transform to AML
CML

AML

myelodysplastic

ALL

CLL

A

C) Reason

Myelodysplastic syndromesStem cell disorders involving ineffective hematopoiesis defects in cell maturation of all non-lymphoid lineages. Caused by de novo mutations or environmental exposure (e.g., radiation, benzene, chemotherapy). Risk of transformation to AML.

1040
Q

No. of parts of CBD is
1

3

5

4

2

A

D) Reason

CBD has four parts

1041
Q

no. of frontonasal prominances for face development
4

3

2

5

1

A

E) Reason

The face develops from 5 primordia of mesoderm (neural crest) of the first pharyngeal arch: a single frontonasal prominence, the pair of maxillary prominences, and the pair of mandibular prominences.

1042
Q

fixed splitting is characteristic of
VSD

ASD

PDA

TOF

TGA

A

B) Reason

Fixed splitting: Seen in ASD. ASD leads toleft-to-right shunt inc. RA and RV volumes inc. flow through pulmonic valve such that, regardless of breath, pulmonic closure is greatly delayed.

1043
Q

Injury in axilla resulting in loss of supination & flexion of arm. Nerve involved?
Musculocutaneous

radial

ulnar

axillary

median

A

A) Reason

Musculocutaneous Nerve Supplies: Coracobrachialis, biceps brachii, and brachialis muscles; supplies the skin along the lateral border of forearm when it becomes the lateral cutaneous nerve of the forearm.

1044
Q

baroreceptors act & decrease compliance of what?
arteries

venules

arterioles

muscle

veins

A

E) Reason

baroreceptors respond to the decrease in arterial pressure by decreasing the firing rate of the carotid sinus nerves. A coordinated response from the vasomotor center then increases sympathetic outflow to the heart and blood vessels and decreases parasympathetic outflow to the heart. As a result, heart rate, contractility, decreased venous complience, TPR, and venous return increase, and blood pressure increases toward normal.

1045
Q

tibialis anterior and posterior are damaged, which movement is compromised
rotation

dorsiflexion

Planterflexion

inversion

eversion

A

D) Reason

Inversion (movement of the foot so that the sole facesmedially): Tibialis anterior, extensor hallucis longus, medial tendons of extensor digitorum longus, and tibialis posterior muscles.

1046
Q

which clotting factors are detected wit PT?
1,2,5,7

1,2,5,7,10

2,7,9

2,7,9,10

7,9,10

A

B) Reason

PT—tests function of common and extrinsic pathway (factors I, II, V, VII, and X). Defect increases PT. PTT—tests function of common and intrinsic pathway (all factors except VII and XIII). Defect increases PTT.

1047
Q

pterigomandibular ligament is attached to
mandible angle

body of mandible

pterygoid

ala of nose

buccinator

A

E) Reason

Pterygomandibular Ligament: Connects the hamularprocess of the medial pterygoid plate to the posterior end ofthe mylohyoid line of the mandible. It also gives attachmentto the superior constrictor and the buccinator muscles.

1048
Q

Pt has inability to evert foot, muscle damaged
peroneus longus

adductor longus

tibialis

soleus

gastrocanemius

A

A) Reason

Eversion (opposite movement of the foot so that the solefaces laterally): Peroneus longus, peroneus brevis, peroneus tertius, and lateral tendons of extensor digitorumlongus muscles.

1049
Q

lateral thoracic artery supplies which structure?
breast

pec. major
pec. minor

pleura

thymus

A

A) Reason

ARTERIESLateral thoracic and thoracoacromial arteries, branches ofthe axillary artery, and perforating branches of the internalthoracic and intercostal arteries supply the breast.

1050
Q

adrenal medulla gets pre-ganglionic fibers from
sacral plexus

lumber plexus

renal plexus

sacral splanchnic

lumber splanchnic

A

C) Reason

Chromaffin cells are neural crest cells, which migrate into the adrenal medulla to form postganglionic sympathetic neurons. That reaches the gland through renal plexus.

1051
Q

while observing surgery for 1st time a med student falls on floor, BP 90/60, reason?
dec. tone

inc. tone
dec. TPR
inc. TPR

vasoconstriction

A

C) Reason

Sympathetic system is characterized by fight and flight, here the student”s sympathetic system was activated due to stress laeding to sec. TPR and cuase hypotension and syncope.

1052
Q

For physiological shunt to occur V/Q must be?
infinite

zero

1

2

10

A

B) Reason

V/Q is 0 = airway obstruction (shunt). In shunt, 100% O2 does not improve Po2.V/Q is ∞ = blood flow obstruction (physiologic dead space). Assuming < 100% dead space, 100% O2 improves Po2.

1053
Q

acessary meningeal artery passes through which foramen?
rotundum

spinosum

jugular

magnum

ovale

A

E) Reason

The foramen ovale lies posterolateral to the foramen rotundum. It perforates the greater wing of the sphenoid, and it transmits the acessary meningeal artery, the sensory and the motor roots of themandibular division of the trigeminal nerve. It also transmitsthe lesser petrosal nerve (p. 232).

1054
Q

hyaline cartilage is present in between which joint
syndemosis

gomphosis

secondary cartilaginous

tertiary Cartilagenous

primary Cartilagenous

A

E) Reason

Primary cartilaginous joints (synchondrosis, or hyaline cartilage joints): The bones are united by a plate of hyaline cartilage so that the joint is immovable and strong.

1055
Q

Pattern of Duchene muscular dystrophy
X-linked dominant

autosomal dominant

X-linked recessive

autosomal recessive

Y linked

A

C) Reason

Muscular dystrophy (most commonly Duchenne): X-linked recessive deletion of dystrophin gene.

1056
Q

Which facial bone fracture is most prevalent?
nasal

ethmoid

zygomatic

frontal

parietal

A

A) Reason

FRACTURES OF THE FACIAL BONESNasal fractures, because of the prominence of thenose are the most common facial fractures. Fracturesof the maxilla are common and usually result from adirect anteroposterior blow to the face

1057
Q

ketones production is found elevated. which hormone is responsible?
glucagon

insulin

Somatostatin

GH

calcitonin

A

A) Reason

FUNCTION Catabolic effects of glucagon:Glycogenolysis, gluconeogenesisLipolysis and ketone production

1058
Q

A 56 year old male with central chest pain, the most initial raise in MI
trop I

trop T

Myoglobin

CKMB

CPK

A

D) Reason

CK-MB is predominantly found in myocardium but can also be released from skeletal muscle. Useful in diagnosing reinfarction following acute MI because levels return to normal after 48 hours.

1059
Q

what will happen if sympathetic nervous system activates?
accomodation

pupil constriction

edema

hypremia

pupil dilation

A

E) Reason

Sympathetic system is characterized by fight and flight, It causes puplillary dilatation while parasympathetic stimulation causes pupillary construction.

1060
Q

what causes increased gastrin secretion?
HCL

secretin

amylase

phenylalanine

lipase

A

D) Reason

Stimuli for secretion of gastrin■ Gastrin is secreted from the G cells of the gastric antrum in response to a meal.■ Gastrin is secreted in response to the following:(1) Small peptides and amino acids in the lumen of the stomach ■ The most potent stimuli for gastrin secretion are phenylalanine and tryptophan.

1061
Q

anterior and posterior longitudinal ligament stabilize which structure
spinous process

spines

lateral wings

canal

vertebral bodies

A

E) Reason

The anterior and posterior longitudinal ligaments runas continuous bands down the anterior and the posteriorsurfaces of the vertebral column from the skull to thesacrum and provide stability to vertebral bodies.

1062
Q

tumor right temporal lobe compresses optic tract, will cause
right Homonimous hemianopia

Left homonimous hemianopia

right total loss

left total loss

quadrinopia

A

B) Reason

Optic tract lesion causes contralatral Homonemous hemianopia, hence in rigjt optic tract lesion, left homonymous hemianopia will be the manifestation.

1063
Q

Broca area damage will cause which aphasia
motor

sensory

mixed

global

frontal

A

A) Reason

Broca: Nonfluent motor aphasia with intact comprehension.Broca area—inferior frontal gyrus of frontal lobe

1064
Q

during menstruation and parturation which layer sheds off?
basalis

parietalis

endometrial

myometrial

functionalis

A

E) Reason

The functionalis is the part that is sloughed off at menstruation and parturation then replaced during each menstrual cycle, and the basalis is the portion retained after menstruation that subsequently proliferates and provides a new epithelium and lamina propria.

1065
Q

pain from visceral pleura is carried via
parasympathetic fibers

bronchial unit

vagus

sympathetic nerve

pulmonary plexus

A

E) Reason

Parietal PleuraThe parietal pleura is sensitive to pain, temperature, touch,and pressure. The costal pleura is supplied by the intercostal nerves, the mediastinal pleura by the phrenic nerve,and the diaphragmatic pleura over the domes by thephrenic nerve and around the periphery by the lower intercostal nerves.Visceral PleuraThe visceral pleura receives an autonomic nerve (sympathetic ) supplyfrom the pulmonary plexus. It is sensitive only to stretching.

1066
Q

On ECG ST segment in v4 is elevated. which artery is responsible?
LXA

RCA

posterior Interventricular

anterior Interventricular

LAD

A

E) Reason

V4 is one of the anteriorolateral chest lead, Left anterior descending artery (LAD)—supplies anterior 2/3 of interventricular septum, anterior papillarymuscle, and anterior surface of left ventricle.

1067
Q

A 26 year old lady with 2 weeks of H/O fever, chest pain not affected by respiration, cause
endocardium

epicardium

pleura

myocardium

pericardium

A

D) Reason

pain in myocarditis is not affected by respiration.

1068
Q

Posterior 1/3 of tongue is innervated by?
acessary nerve

tractus solitarius

nucleus ambiguous

amygdala

glossopharyngeal

A

B) Reason

SENSORY INNERVATION• Anterior two thirds: The lingual nerve (general sensation) and chorda tympani (taste).• Posterior third: The glossopharyngeal nerve (nucleus of tractus solitarius, general sensation and taste).

1069
Q

A 54 year old male Patient with cough and bronchial spasm, cause
IgE

IgD

IgA

IgM

IgG

A

A) Reason

IgE: Binds mast cells and basophils; cross-links when exposed to allergen, mediating immediate (type I) hypersensitivity through release of inflammatory mediators such as histamine. Mediates immunity to worms by activating eosinophils. Lowest concentration in serum

1070
Q

A baby died of severe Hemolytic anemia after few days of birth, most affected post mortem part of baby died of Rh incompatiblility
uncus

Pons

medulla

midbrain

basal ganglia

A

E) Reason

In severe form of the Rh incompatiblility as in subsequent pregnancies, the excessive level of bilurubin also deposits in brain (basal ganglia) causing CNS symptoms known as Kernecterus.

1071
Q

relaxing protein in muscle
troponin

elastin

myosin

tropomyosin

actin

A

D) Reason

Relaxation occurs when Ca2+ is reaccumulated by the SR Ca2+-ATPase (SERCA). Intracellular Ca2+ concentration decreases, Ca2+ is released from troponin C, and tropomyosin again blocks the myosin-binding site on actin. As long as intracellular Ca2+ concentration is low, cross-bridge cycling cannot occur

1072
Q

which muscle has no role at wrist
flexor carpi ulnaris

flexor carpi radialis

palmaris

Brachioradialis

crocobrachialis

A

D) Reason

Brachioradialis is Supplied by Radial nerve, it Flexes the forearm at theelbow joint, rotates theforearm to the midproneposition

1073
Q

In a 58 year old male patient of CKD, how to figure out either to do dialysis or transplant
urea

uric acid

Creatinine clearance

GFR

renal blood flow

A

C) Reason

Creatinine is raised in patients with chronic renal failure, its level in serum indicate the effectiveness of the therapy. So less effective control indicate the renal transplantation option.

1074
Q

which one is parasympathetic component
thoracic splanchnic

lumber splanchnic

sacral splanchnic

lumber plexus

sciatic

A

C) Reason

Parasympathetic system is the CranioSacral outflow while Sympathetic System is the ThoracoLumber outflow.

1075
Q

TPR dec. in exercising muscles. why?
sympathetic

local metabolites

parasympathetic

O2

somatic

A

B) Reason

■ During exercise, when demand is high, these local metabolic mechanisms are dominant.■ The local vasodilator substances are lactate, adenosine, and K+ that dec. TPR and increase blood flow.

1076
Q

adduction and lateral rotation of thumb, nerve damaged?
ulnar

radial

axillary

median

Musculocutaneous

A

D) Reason

Branches of the Median Nerve in the Forearm• Muscular branches: Pronator teres, flexor carpi radialis,palmaris longus, and flexor digitorum superficialis muscles.• Articular branches: Elbow joint

1077
Q

nerve to Obturator internus and gemellus is branch of…division of sacral plexus???
anterior

superior

lateral

posterior

inferior

A

D) Reason

Posterior division of Sacral plexus: The nerve to the quadratus femoris muscle also supplies the inferior gemellus muscle.• The nerve to the obturator internus muscle also supplies the superior gemellus muscle.

1078
Q

which diuretic acts on smooth muscles
thiazide

furosemide

spironolactone

Acetazolamide

manitol

A

B) Reason

FurosemideMECHANISM Sulfonamide loop diuretic. Inhibits cotransport system (Na+/K+/2 Cl-) of thick ascending limb of loop of Henle. Abolishes hypertonicity of medulla, preventing concentration of urine. Stimulates PGE release (vasodilatory effect on smooth muscles on afferent arteriole); inhibited by NSAIDs.

1079
Q

external carotid artery begins at
cricothyroid muscle

cricoid cartilage

thyroid cartilage

jugular foramen

isthmus

A

C) Reason

External Carotid ArteryThe external carotid artery begins at the level of the upperborder of the thyroid cartilage as one of the terminalbranches of the common carotid artery.

1080
Q

coagulation pathway is on going. what is most valuable step
thrombin

platelets adhesion

platelet aggregation

fibrin formation

clot formation

A

D) Reason

Most important step in coagulation pathway is the formation of the fibrin from Fibrinogen. then Fibrin mesh acts tostabilize platelet plug

1081
Q

To increase pulse pressure what must change
inc. rate

inc. output
inc. stroke volume
inc. diastolic pressure
dec. diastolic pressure

A

C) Reason

■ is the difference between the systolic and diastolic pressures.■ The most important determinant of pulse pressure is stroke volume. As blood is ejected from the left ventricle into the arterial system, arterial pressure increases because of the relatively low capacitance of the arteries. Because diastolic pressure remains unchanged during ventricular systole, the pulse pressure increases to the same extent as the systolic pressure

1082
Q

what is the reason heart muscle act as synctium.
gap junction

desmosome

less myofibrils

more neurofibers

no gap junctions

A

A) Reason

Gap junctions■ are present at the intercalated disks.■ are low-resistance paths between cells that allow for rapid electrical spread of action potentials.■ account for the observation that the heart behaves as an electrical syncytium.

1083
Q

slow pain fibers responsible for slow pain detection are
C fibers

A fibers

D fibers

delta fiber

gamma fibers

A

A) Reason

■ Fast pain is carried by group III fibers. It has a rapid onset and offset, and is localized.■ Slow pain is carried by C fibers. It is characterized as aching, burning, or throbbing that is poorly localized

1084
Q

who carries more total body water
female

adult male

new born

old age

obese people

A

C) Reason

■ Total body water (TBW) is approximately 60% of body weight.■ The percentage of TBW is highest in newborns and adult males and lowest in adult femalesand in adults with a large amount of adipose tissue.

1085
Q

A 35 years old woman with warty lesion in vulva, poikilocytosis on smear, Dx..?
SCC

BCC

condylomata acumulata

lymphoma

granuloma

A

C) Reason

Condylomata acuminata: Caused by HPV 6 and 11.Genital warts, koilocytes.

1086
Q

with moderate excercise what organ has unaffected blood flow
coronary

cerebral

pleural

pulmonary

ophthalmic

A

B) Reason

Constriction of arterioles causes intense peripheral vasoconstriction and increased TPR. Blood flow to other organs (e.g., kidneys) is significantly reduced in an attempt to preserve blood flow to the brain.

1087
Q

A 54 year old diabetic lady with blurr vision and clouding, fundoscopy and ICP normal, Cause
retinal detachment

glaucoma

retinopathy

occulopathy

cataract

A

E) Reason

Osmotic damage (sorbitol accumulation in organs with aldose reductase and decrease or absent sorbitol dehydrogenase):Neuropathy (motor, sensory, and autonomic degeneration)Cataracts by accumulation of sorbitol.

1088
Q

A 28 year old female with pruritis, ALP 1600, ALT 75, AST 250, direct bilurubin 15, cause
cirrhosis

extra hepatic cholestasis

fibrosis

cholelithiasis

hepatitis

A

B) Reason

ALP is raised in any obstructive cases of the billiary tract, moreover the direct bilurubin raised indicate the extra hepatic stasis of the billiary tract.

1089
Q

A boy with blue sclera and H/O multiple fractures by minor trauma, pattern of inheritance
X-linked recessive

X-linked dominant

autosomal dominant

autosomal recessive

multifactorial

A

C) Reason

Osteogenesis imperfecta:Genetic bone disorder (brittle bone disease ) caused by a variety of gene defects.Most common form is autosomal dominant with dec. production of otherwise normal type I collagen. OI manifestations can include:Multiple fractures with minimal trauma; may occur during the birth processBlue sclerae due to the translucency of the connective tissue over the choroidal veins.

1090
Q

Type of cartilage present in between discs of joints
hyaline

articular cartilage

chondroitin

mixed

fibrocartilage

A

E) Reason

Intra-articular fibrocartilages, articular discs or menisci, complete or incomplete, help in spreading the synovial fluid throughout the joint cavity, but particularly between the articular surfaces, e.g. temporomandibular joint.

1091
Q

which anti-epileptic cause Steven johson syndrome.
valproate

ethosuxamide

dormicum

phenytion

benzodiazapine

A

D) Reason

Nystagmus, diplopia, ataxia, sedation, gingival hyperplasia, hirsutism, peripheral neuropathy, megaloblastic anemia, teratogenesis (fetal hydantoin syndrome) SLE-like syndrome, induction of cytochrome P-450, lymphadenopathy, Stevens Johnson syndrome, osteopenia.

1092
Q

Stagnant hypoxia is result of
anemia

polycythemia

Thrombocytopenia

CML

ALL

A

B) Reason

Polycythemia often presents as intense itching after hot shower. Rare but classic symptom is erythromelalgia (severe, burning pain and reddish or bluish coloration) due to episodic blood clots due to hyperviscosity in vessels of the extremities. Hyperviscosity and clots in vessels is the cause of stagnant Hypoxia.

1093
Q

Which ligament injury will lead to eversion loss?
Cruciate

anterior ligament

deltoid

suspensory

short

A

C) Reason

Medial (Deltoid) LigamentThe medial ligament is very strong and is attached by itsapex to the tip of the medial malleolus. Its injury results in loss of eversion.

1094
Q

A 46 year old female with pruritis and hepatospleenomegaly, investigation to confirm
USG

ERCP

CT abdomen

CBC

ESR

A

C) Reason

Primary billiary cirrhosis, PRESENTATION: Pruritus, jaundice, dark urine, light stools, hepatosplenomegaly. Confirmed by CT and MRCP.

1095
Q

in adults level of tracheostomy?
C1

C2

C3

C2-3

C3-4

A

D) Reason

The isthmuscrosses the midline in front of the second, the third, andthe fourth rings of the trachea. Tracheostomy is done at C2-3 and save the isthmus injury.

1096
Q

supply of head of femor
Obturator artery

femoral artry

Pudendal artery

uterine artery

genital artery

A

A) Reason

In the young, the epiphysis of the head is supplied bya small branch of the obturator artery, which passes tothe head along the ligament to the femoral head

1097
Q

supply of head of femor
Obturator artery

femoral artry

Pudendal artery

uterine artery

genital artery

A

A) Reason

In the young, the epiphysis of the head is supplied bya small branch of the obturator artery, which passes tothe head along the ligament to the femoral head

1098
Q

transpyloric plain is at level of lower border of which of these?
L2

L3

L1

L4

L5

A

C) Reason

The pylorus lies on the transpyloric plane at thelevel of the first lumbar vertebra

1099
Q

Protamin sulphate anatagonize heparin is an example of which interaction
drug-receptor

drug-drug

cell-drug

drug-antidote

drug-serum

A

A) Reason

Protamine can lessen the risk of serious bleeding that canresult from excessive unfractionated heparin by occupying its receptor site. Protamine only partially reverses the effects of LMWheparins

1100
Q

natural defense against tumor
necrosis

Apoptosis

Hypoxia

hypotrophy

anaplasia

A

B) Reason

Natural killer cells Use perforin and granzymes to induce apoptosis of virally infected cells and tumor cells.Only lymphocyte member of innate immune system

1101
Q

atlanto-axial joint movement is known as
Flexion

extension

adduction

rotation

abduction

A

D) Reason

MOVEMENTSRotation of the atlas with the head on the axis is possible.

1102
Q

which nerve damage will result in claw hand?
lower trunk

radial

median

axillary

Musculocutaneous

A

A) Reason

LOWER TRUNK LESIONS OF THE BRACHIALPLEXUS (KLUMPKE PALSY)Lower trunk lesions of the brachial plexus result fromtraction injury, as in excessive abduction of the arm.The first thoracic nerve is usually torn, all the smallmuscles of the hand are paralyzed, and the patient develops a claw hand

1103
Q

thirst is relieved by
gut reflux

least secretions

dec. Osmolarity

gut distention

more ECF volume

A

D) Reason

Autonomic center of Hypothalamus■ Temperature regulation centerthrough central and periphral mechanisms.■ Thirst and food intake regulatory centers as gut distention is associated with increased serum Osmolarity and causes thirst stimulation.

1104
Q

RCA blocked by thrombus before origin of marginal artery, site affectd is?
SA node

AV node

Purkinje fibers

bundle of His

atrial fibers

A

A) Reason

SA and AV nodes are usually supplied by RCA before the origin of the marginal branch. Infarct may cause nodal dysfunction (bradycardia or heart block).

1105
Q

vol. expansion alongwith hypokalemic metabolic alkalosis is due to
Cushing syndrome

klinefilter syndrome

WPS syndrome

primary hypertension

Conn syndrome

A

E) Reason

aldosterone-secreting adrenal adenoma (Conn syndrome), resulting in hypertension, hypokalemia, metabolic alkalosis, and lowplasma renin. Normal Na+ due to aldosterone escape = no edema due to aldosterone escape mechanism. May be bilateral or unilateral.

1106
Q

external oblique at its origin interdigitate with which muscle?
trapezius

serratus posterior

latismus dorsi

quadratus lumborum

serratus anterior

A

E) Reason

External oblique Originate from Lower right ribs where it interdigitate with serratus anterior. It is inserted into the Xiphoid process, linea alba, pubic crest, pubic tubercle, iliac crest.

1107
Q

most suitable pain killar in pancreatitis
morphine

Ketorolac

endomethacin

pethidine

NSAID

A

D) Reason

Pathedine is a synthetic analgesic with anti-spamodic action as well. That is the reason, it can be used in severe pain of the pancreatitis.

1108
Q

time taken for spread of potential from sub-endocardium to epicardium
T wave

p wave

QRS complex

ST segment

PR interval

A

C) Reason

QRS complex—ventricular depolarization (normally < 120 msec). It the time taken by conductive system to conduct electric potential from sub-endocardium (the conductive system) to the ventricle (including its pericardium).

1109
Q

in mediolateral episiotomy which of these is likely to be injured?
perineal muscles

perineal pouch

bulbospongiosus

perineal membrane

bulbourethral

A

C) Reason

Bulb of the vagina is attached to the undersurface of the urogenital diaphragm and is covered by thebulbospongiosus muscles. It is the most commonly damaged muscle during mediolateral episiotomy.

1110
Q

receptor act as transducer because of?
modulater

necessary

physical stimuli

augment stimuli

chemical stimuli

A

D) Reason

Receptors are the specific type of molecules usually proteins that are present at cell surface (usually) inside cell. Receptors mediate the function of a ligand molecule or stimulus.

1111
Q

Which of these replaces transplanted bone?
cartilage

endostium

periosteum

osteoclasts

tendon

A

C) Reason

Mature long bones are covered on all surfaces by dense cortical bone with an outer periosteum, with the exception of articular surfaces, which are covered by hyaline articular cartilage. Transplanted bones are replaced by this periostium.

1112
Q

Dynamic chemotactic agent
LTB1

LTB4

LTC4

LTE4

PGI2

A

B) Reason

Chemotactic products released in response to bacteria: C5a, IL-8, LTB4, kallikrein, platelet-activating factor

1113
Q

measurment of renal plasma flow is by?
Urea

Inulin

PABA

Creatinine

PAH

A

E) Reason

Measurement of renal plasma flow (RPF)—clearance of para-aminohippuric acid (PAH)■ PAH is filtered and secreted by the renal tubules.■ Clearance of PAH is used to measure RPF.

1114
Q

A 23 year old male with Hb 8, WBC 3k and platelet 30k, and skin pigmentation, cause
folate

B12 deficiency

CML

ALL

CLL

A

B) Reason

Hypersegmented neutrophils, pancytopenia, glossitis, dec. B12, inc. homocysteine, inc. methylmalonic acid.Neurologic symptoms: subacute combined degeneration (due to involvement of B12 in fatty acid pathways and myelin synthesis):

1115
Q

normal saline is
9g in 1000ml

9g in 1000ml

0.9g in 1000ml

1g in 91ml

9g in 100ml

A

A) Reason

Normal saline is with NaCl concentration of 0.9 percent, That means 0.9g in 100ml and 9g in 1000ml.

1116
Q

following is true about mean > median > mode.
sensitivity

specificity

negative skew

positive skew

PPV

A

D) Reason

Positive skew Typically, mean > median > mode. Asymmetry with longer tail on right. Negative skew: Typically, mean < median < mode.

1117
Q

“what is actually happening” is studied in which type
cohort

observational

rertrospective

cross control

experimental

A

D) Reason

Cross-sectional study:Collects data from a group of people to assess frequency of disease (and related risk factors) at a particular point in time.Asks, “What is happening?”

1118
Q

superficial plexus is located below?
beneath chest wall

parietal pleura

lung surface

endothelium

visceral pleura

A

E) Reason

superficial plexus lies beneath the visceral pleura anddrains over the surface of the lung toward the hilum, wherethe lymph vessels enter the bronchopulmonary nodes.

1119
Q

longest erythrocytic shizgony phase is of
P. falciparum

P. vivax

P. malarie

P. ovale

toxaoplama

A

C) Reason

P. malarie is a specie of the Plasmodium that is known as quatran malaria (fever spikes on fourth day) due to its long duration of about 48-72 hours of erythrocytic shizgony phase. While other species cuase tertiary malaria (fever spikes on 3rd day) due to relatively short erythrocytic shizgony phase.

1120
Q

lung cancer spread through
Hematogenous

direct invasion

nerve plexus

chemokines

lymphatics

A

E) Reason

In the lung, metastases (usually multiple lesions) are more common than 1° neoplasms. Most often from breast, colon, prostate, and bladder cancer. Lung cancer metastases through lymphatics to—adrenals, brain, bone (pathologic fracture), liver (jaundice, hepatomegaly).

1121
Q

RBCs are most suitable to do
viscosity

Hb

immunity

clotting

gases exchange

A

E) Reason

Erythrocytes: Carries O2 to tissues and CO2 to lungs. Anucleate and biconcave, with large surface area-to-volume ratio for rapid gas exchange. Life span of 120 days.

1122
Q

In a 74 year old man with CA prostate, which nodes will be affected in prostate ca.
Pudendal

deep inguinal

superficial inguinal

external iliac

internal iliac

A

E) Reason

Lymph DrainageThe lymph drains into the internal iliac nodes.Nerve SupplyInferior hypogastric plexuses supply the prostate.

1123
Q

sensory nerve supply of larynx above vocal chords
internal laryngeal

reccurent laryngeal

external laryngeal

vagus

auricular

A

A) Reason

NERVE SUPPLYSensory Nerves• Above the vocal folds: The internal laryngeal nerve.• Below the vocal folds: The recurrent laryngeal nerve.

1124
Q

Resection of vagal trunk results in
no acid reflux

delayed gastric emptying

bile gastritis

pancreatic cancer

abscess

A

B) Reason

Truncal vagotomy also causes disturbed motility of the antrum and pylorus of stomach besides the sec. acid secretion. That is why, truncal vagotomy causes delayed gastric emptying, and now selective vagotomy is in practice to avoid this side effect.

1125
Q

which cell has different origin
osteoblasts

osteocytes

Chondrocytes

osteoclasts

fibroelasts

A

D) Reason

Macrophage rank binds to osteoblast RAN KL and macrophage becomes an osteoclast. While osteoblasts and osteocytes are the bone forming cells.

1126
Q

classical example of multifactorial involvement
Down syndrome

DM

osteoarthritis

infective Endocarditis

rheumatic fever

A

B) Reason

DM is a multifactorial trait that presents with Polydipsia, polyuria, polyphagia, weight loss, DKA (type 1), hyperosmolar coma (type 2).Rarely, can be caused by unopposed secretion of GH and epinephrine.

1127
Q

In portal hypertension, veins are engorged. which are?
lower esophagus

abdomen

upper esophagus

gastric

spleenic

A

A) Reason

Esophageal varices Painless bleeding of dilated submucosal veins in lower 1 ⁄3 of esophagus 2° to portal hypertension

1128
Q

cervical branch of facial nerve supplies?
trapezius

cricothyroid

platysma

omohyoid

latismus dorsi

A

C) Reason

Platysma Muscle is supplied by the spinal part of the facial nerve, It Depresses themandible and angleof the mouth

1129
Q

treatment of feverish(febrile) feeling after blood transfusion
adrenaline

antihistamine

IV fluids

anti serum

antipyretics

A

E) Reason

Blood reaction that doesn”t involve the severe manifestations of the anaphylactic shock, is a simple febrile reaction and is easily managed by antioyretics and close monitoring.

1130
Q

long term prophylaxis for Rheumatic fever with Endocarditis
benzathene penicillin

gentamicin

penicillin

benzyl penicillin

metronidazole

A

A) Reason

benzathine penicillin once each month for several years) in persons who have hadrheumatic fever is important to prevent recurrence of the disease. There is noevidence that patients who have had AGN require similar penicillin prophylaxis

1131
Q

zona glomerulosa is mainly controlled by
Angiotensin-II

LH

GH

FSH

cortisol

A

A) Reason

Zona glomerulosa is the part of the adrenal cortex that secrets Aldosterone in an influence of renin-Angiotensin-aldosterone system of blood pressure control.

1132
Q

which is delayed allergy mediator
chemokine

prostagladins

ILs

TNF

lymphokines

A

E) Reason

Type IV Delayed (T-cell-mediated) type—sensitized T lymphocytes encounter antigen and then release lymphokines (leads to macrophage activation; no antibody involved).

1133
Q

what is absent at L4 spinal cord
anterior horn

posterior horn

lateral horn

spine

column

A

C) Reason

Preganglionic neurons of the sympathetic nervous system originate in spinal cord from lateral horns of the segments T1–L3 or the thoracolumbar region. So, there is no lateral horn bellow L3.

1134
Q

ECF vol. is raised by which fluid
Hypertonic saline

hypotonic saline

5 percent dextrose

more water

less Osmolarity

A

A) Reason

Hypertonic solution causes the shrinkage of the cells and outflow of fluids outside the cell leading to increased ECF volume.

1135
Q

which causes hyperchloremic metabolic acidosis
furosemide

Acetazolamide

spironolactone

mannitol

thiazide

A

B) Reason

TOXICITY: Hyperchloremic metabolic acidosis, paresthesias, NH3 toxicity, sulfa allergy

1136
Q

while performing surgery ovary is adherent to fossa, which artery will be injured?
ovarian

Fallopian

pelvic

uterine

internal iliac

A

E) Reason

Internal iliac artery is damaged.

1137
Q

which fascia is of thyroid gland?
superficial

pre tracheal

para tracheal

membranous

deep cervical

A

B) Reason

Pretracheal LayerThe pretracheal layer is a thin layer that is attached above tothe laryngeal cartilages. It surrounds the thyroid gland.

1138
Q

thymus is supplied by?
superior thyroid

thymic artery

pleural artery

internal carotid

inferior thyroid artery

A

E) Reason

The inferior thyroid artery ascends to reach the posterior surface of the thyroid gland, where it is closely relatedto the recurrent laryngeal nerve. The inferior thyroidartery supplies the thyroid, thymus and the inferior parathyroidglands.

1139
Q

toxoplamosis is caused by
fungi

bacteria

virus

parasite

antigen

A

D) Reason

Toxoplasma gondii, a protozoa:Brain abscess in HIV (seen as ring-enhancing brain lesions on CT/MRI); congenital toxoplasmosis = “classic triad” of chorioretinitis, hydrocephalus, and intracranial calcifications

1140
Q

area at node of ranvier has
more myelination

least myelination

fast fibers

less myelination

no mylen

A

E) Reason

In myelinated neurons, the area in between the myelinated parts is called the node of ranvier where there is no myelin. It is responsible for rapid conduction, the saltatory conduction.

1141
Q

insulin dependent uptake in
RBC

brain

cornea

exercising muscle

kidney

A

D) Reason

Insulin-dependent glucose transporters:GLUT-4: adipose tissue, skeletal muscleInsulin-independent transporters:GLUT-1: RBCs, brain, corneaGLUT-5 (fructose): spermatocytes, GI tractGLUT-2 (bidirectional): β islet cells, liver, kidney, small intestine

1142
Q

common cause of inc. bleeding time
hemophilia

rifampin toxicity

pyrizinamide side effects

aspirin toxicity

Clopidogrel

A

D) Reason

Aspirin is commonly used drug for pain killers and other purposes. TOXICITY of Aspirin: Gastric ulceration, tinnitus (CN VIII). Chronic use can lead to acute renal failure, interstitial nephritis, increased bleeding time, and upper GI bleeding.

1143
Q

Drainage of medial quadrant of
axillary nodes

apical nodes

internal memory

external memory

internal thoracic LNs

A

E) Reason

Lymphatic DrainageThe lateral quadrants of the breast drain into the anterior axillary or the pectoral nodes. The medial quadrantsdrain into the internal thoracic nodes. A few lymph vesselsalso drain posteriorly into the posterior intercostal nodes,and some communicate with the lymphatic vessels of theopposite breast and with those of the anterior abdominalwall.

1144
Q

if radial groove is injured what will be most likely damaged?
radialis

flexor carpi ulnaris

extensor hallucis

extensor of wrist

Brachioradialis

A

D) Reason

NERVE SUPPLYAnterior interosseous nerve (flexor and adductor of wrist) from the median and the deepbranches of the radial (extensor of wrist) and the ulnar nerves supply the wristjoint

1145
Q

Pain of parotid swelling is due to compression of which nerve?
auriculotemporal

fascial

mandibular

maxillary

nasal

A

A) Reason

The postganglionic secretomotorfibers reach the parotid salivary gland via the auriculotemporal nerve. Its compression is the main cause of parotid gland pain.

1146
Q

A 7 year old boy have soft tissue tumor with vimentin positive, Dx..?
carcinoma

leukemoid reaction

lymphoma

rhabdomyosarcoma

leukemia

A

D) Reason

Staining for different tissue tumors, Vimentin: Connective tissueDesMin: MuscleCytokeratin: Epithelial cellsGFAP: NeuroGliaNeurofilaments: Neurons

1147
Q

Diplopia on going downstairs, which nerve is damaged?
CN6

CN 5

CN 7

CN 3

CN 4

A

E) Reason

4th cranial nerve supplies the superior oblique muscle which is involved in looking downwards and medially.

1148
Q

femoral hernia descends behind
inguinal ligament

inguinal canal

inguinal ring

femoral artry

femoral nerve

A

A) Reason

The neck of the femoral hernial sac lies at the femoral ringand is related anteriorly to the inguinal ligament,posteriorly to the pectineal ligament, laterally to thefemoral vein, and medially to the sharp, free edge ofthe lacunar ligamen

1149
Q

Which test is most relevant for pulmonar function to check obstructve vs restrictive diseases
ventilation

RV

FEV1/ FVC

FVC

RC

A

C) Reason

Obstructive lung diseases:Obstruction of air flow resulting in air trapping in the lungs. Airways close prematurely at high lung volumes inc. RV and dec. FVC. PFTs: decreased FEV1/FVC ratio (hallmark),

1150
Q

pulmonary embolism of pulmonary artery will result in
zero V/Q

V/Q is 1

no ventilation

Hypoxia

inc. V/Q

A

E) Reason

Pulmonary emboli:V/Q mismatch leads to hypoxemia and respiratory alkalosis. Sudden-onset dyspnea, chest pain, tachypnea. May present as sudden death.Types: Fat, Air, Thrombus, Bacteria, Amniotic.

1151
Q

anterior dislocated shoulder will result in which artery damage?
acromial

axillary

Anterior circumflex

posterior circumflex

clavicular

A

C) Reason

Collateral circulation around the shoulder joint wouldinvolve the following:A. The subscapular arteryB. The superficial cervical arteryC. The suprascapular arteryD. The anterior circumflex humeral artery, most commonly injured in anterior dislocation.

1152
Q

A 15 years old boy with reddish nodule for many years and change in size, Dx..?
mole

nevus

birth mark

myoma

hemangioma

A

E) Reason

Strawberry hemangioma:Benign capillary hemangioma of infancy. Appears in first few weeks of life (1/200 births); grows rapidly and regresses spontaneously at 5–8 years old.Cherry hemangioma: Benign capillary hemangioma of the elderly. Does not regress. Frequency inc. with age.

1153
Q

left gastric vein drains into
portal vein

spleenic vein

IVC

mesenteric vein

hepatic vein

A

A) Reason

Veins:The veins drain into the portal circulation. The right and leftgastric veins drain into the portal vein. Left gastric vein drain directly into the portal vein. The short gastric and the left gastroepiploic veins drain into the splenic vein,and the right gastroepiploic vein drains into the superiormesenteric vein.

1154
Q

Cushing reflex is character of
brain Ischemia

more ECF Osmolarity

more ECF volume

vol. contraction

red reflex

A

A) Reason

The Cushing reaction is an example of the response to cerebral ischemia. Increases in intracranial pressure cause compression of the cerebral blood vessels, leading to cerebral ischemia and increased cerebral Pco2. The vasomotor center directs an increase in sympathetic outflow to the heart and blood vessels, which causes a profound increase in arterial pressure.

1155
Q

A 12 year old girl with tumor of vermis with rosettes present, Dx..?
SCC

myxoma

astrocytoma

ependymoma

medulloblastoma

A

D) Reason

Characteristic perivascular rosettes seen in Ependymoma. Rodshaped blepharoplasts (basal ciliary bodies) found near nucleus.

1156
Q

Which of these are false ribs?
8th-10th

1st 3

last 3

7th-9th

9th-11th

A

A) Reason

False ribs: The eighth, ninth, and tenth pairs are attachedanteriorly to each other and to the seventh rib by theircostal cartilages and small synovial joints.

1157
Q

superficial external Pudendal artery emerges through what?
femoral ring

inguinal ring

inguinal canal

saphenus opening

adductor canal

A

D) Reason

The saphenous opening is a gap in the deep fascia in thefront of the thigh and just below the inguinal ligament. It allows passage of the great saphenous vein, some smallbranches of the femoral artery, and lymph vessels.

1158
Q

superficial epigastric artery passes through which opening?
saphenus opening

inguinal canal

inguinal ring

femoral canal

femoral ring

A

A) Reason

The saphenous opening is a gap in the deep fascia in thefront of the thigh and just below the inguinal ligament. It allows passage of the great saphenous vein, some smallbranches of the femoral artery, and lymph vessels.

1159
Q

Effect of alpha adrenergic receptor activation
hypotension

pupil construction

pupil dilation

accomodation

inc. ICP

A

C) Reason

Adrenergic stimulation of α1: Hypotension (vasoconstrictor), ocular procedures (mydriatic), rhinitis (decongestant)

1160
Q

90 percent of anthrax pt has which finding
lung involved

saddle nose

from mammals

brain lesions

skin lesion

A

E) Reason

Cutaneous anthrax: most common presentation, Boil-like lesion and ulcer with black eschar (painless, necrotic) causes uncommonly progresses to bacteremia and death.Pulmonary anthrax: Inhalation of spores causes flu-like symptoms that rapidly progress to fever, pulmonary hemorrhage, mediastinitis, and shock.

1161
Q

1st sign of deficiency of vitamin A
chelosis

corneal degeneration

corneal vascolurization

pancytopenia

night blindness

A

E) Reason

DEFICIENCY Night blindness (nyctalopia); dry, scaly skin (xerosis cutis); alopecia; corneal degeneration (keratomalacia); immune suppression.

1162
Q

A 46 year old female has ovarian tumor with inhibin positive, Dx..?
adeno ca

germ cell tumors

yolk sac tumor

granulosa cell tumor

theca cell tumor

A

D) Reason

Granulosa cell tumor Most common sex cord stromal tumor. Predominantly women in their 50s. These are the inhibin positive tumors. Often produce estrogen and/or progesterone and present with abnormal uterine bleeding, sexual precocity (in preadolescents), breast tenderness

1163
Q

Chief investigation for hemophilia
platelet count

PT

Electrophoresis

periphral smear

APTT

A

E) Reason

Hemophilia A: deficiency of factor VIII.Hemophilia B: deficiency of factor IX. These are the factors of intrinsic pathway and can be assessed by PTT.

1164
Q

best advice for increasing vitamin intake
more liver foods

more meats

more pulses

inc. vegetables
inc. egg

A

D) Reason

Vitamins are abundant in vegetables and fruits. Moreover it ia an easy and cheap source of vitamins.

1165
Q

which organ uses glucose solely as energy source
liver

kidney

heart

skeletal muscle

brain

A

E) Reason

Carbohydrates are the main sources of energyin the body. Brain cells and RBCs are almostwholly dependent on carbohydrates as theenergy source.

1166
Q

level of lumbar puncture is between
L1

L2-3

L3-4

L4-5

L5

A

D) Reason

A lumbar puncture is used to inject anesthetic material in the epidural space or to withdraw CSF from the subarachnoid space. • A spinal tap is typically performed at the L4 and L5 interspace.

1167
Q

in muscles, ATPase activity is shown by
myosin

actin

troponin

sarcoplasm

tropomyosin

A

A) Reason

Myosin has 3 different biological activities:a. Myosin molecules assemble into filaments.b. Myosin acts as the enzyme ATPase.c. Myosin binds to actin polymer which is the majorcomponent of the thin filaments

1168
Q

gastric emptying is inhibited by?
gastrin

secretin

CCK

enterokinase

bile

A

C) Reason

Increases pancreatic secretionIncreases gallbladder contractionDecreases gastric emptyingIncreases phincter of Oddi relaxation

1169
Q

Young girl with ambiguous genitalia and 46 XY, Diagnosis?
true hermaphrodite

false hermaphrodite

klinefilter syndrome

Patau syndrome

CAH

A

E) Reason

Due to excessive and inappropriate exposure to androgenic steroids during early gestation (e.g., congenital adrenal hyperplasia or exogenous administration of androgens during pregnancy.

1170
Q

accessory muscle of expiration is?
trapezius

platysma

rectus abdominis

external oblique

transversus abdominus

A

C) Reason

Forced expiration is an active process accomplished bycontraction of the muscles of the anterior abdominal wall, rectus abdominus(forcing the relaxed diaphragm upward by raising intraabdominal pressure)

1171
Q

damage of tibial nerve spares which muscle so that flexion exists ??
peroneus longus

peroneus brevis

poplitius

Sartorius

short head of bicep femoris

A

E) Reason

Flexion: Biceps femoris, semitendinosus, and semimembranosus muscles. Bicep femoris is supplied by sciatic nerve that is why in tibial nerve lesion, bicep femoris cause some Flexion of knee.

1172
Q

A 43 year old male patient died of MI, on autopsy, coagulative necrosis with neutrophils found, age of lesion
1-2 hrs

4 hrs

6 hrs

6-12 hrs

24 hrs

A

E) Reason

12- 24 hr:Occluded artery InfarctDark mottling; pale with tetrazolium stainEarly coagulative necrosis, neutrophils and release of necrotic cell contents into blood; edema, hemorrhage, wavy fibers.

1173
Q

GnRH is inhibited during lactation by
Prolactin

LH

FSH

GH

TSH

A

A) Reason

FUNCTION Stimulates milk production in breast; inhibits ovulation in females and spermatogenesis in males by inhibiting GnRH synthesis and release.

1174
Q

metabolic acidosis with normal anion gap is in
coma

diarrohea

hyperventilation

vomiting

fever

A

B) Reason

Metabolic acidosis with Normal anion gap (8−12 mEq/L):HyperalimentationAddison diseaseRenal tubular acidosisDiarrheaAcetazolamideSpironolactoneSaline infusion

1175
Q

a lady is taking only ferrous sulphate, she will have deficiency of
Ca

B12

K

Na

folate

A

E) Reason

Folate deficiency Causes: malnutrition (e.g., alcoholics), malabsorption, antifolates (e.g., methotrexate, trimethoprim, phenytoin), increased requirement (e.g., hemolytic anemia, only iron intake, pregnancy).

1176
Q

heat loss of an intubated individual is by
conduction

evaporation

convection

radiation

hyperventilation

A

B) Reason

Tracheostomy is an artificial mean of opening the airway through the trachea. As it is an artificial opening and it is always open, and causes heat and vapors loss continually.

1177
Q

Which of these is membrano-cartilagenous bone
femor

tibia

clavicle

fibula

humerus

A

C) Reason

Membrano-cartilaginous bones ossify partly in membrane and partly in cartilage. Examples: clavicle, mandible, occipital, temporal, sphenoid.

1178
Q

HLA matching is best detected by
RBC

platelet

WBCs

eosinophils

liver cells

A

C) Reason

HLA (Human Leukocytic Antigen) are the specific type of antigens present on Leukocytes that have specific antigenicity and that is why used for tissue matching.

1179
Q

osmosis is result of the movement of following molecules across cell membrane
CO2

liquids

solvents

water

solutes

A

D) Reason

■ Osmosis is the flow of water across a semipermeable membrane from a solution with low solute concentration to a solution with high solute concentration.

1180
Q

CN 5,6 & 7 damaged, which area involved
pons

Midbrain

medulla

cortex

spinal cord

A

A) Reason

Midbrain—nuclei of CN III, IVPons—nuclei of CN V, VI, VII, VIIIMedulla—nuclei of CN IX, X, XIISpinal cord—nucleus of CN XI

1181
Q

drugs are inactive in plasma, reason
free

protein bound

in liver

in adipose tissue

in gut

A

B) Reason

In plasma, protein bound drugs are inactive form of the drugs while free form of the drugs that are not bound to plasma proteins are the active form of the drugs.

1182
Q

which descending tract has function related to lateral spinothalamic tract
tactospinal

thalamospinal

limbospinal

rubrospinal

reticulospinal

A

D) Reason

Lateral corticospinal tract(voluntary motor). Rubrospinal tract■ originates in the red nucleus and projects to interneurons in the lateral spinal cord.■ Stimulation of the red nucleus produces stimulation of flexors and inhibition of extensors

1183
Q

Reason of hyperventilation if breathing in paper bag
inc. pH

inc. HCO3
inc. K
dec. H
inc. CO2

A

E) Reason

If a person is breathing in a paper bag, there will be increased level of CO2 accumulation that will be washed out by hyperventilation.

1184
Q

collagen type related to wound strength
type 4

type 3

type 1

type 2

type 5

A

C) Reason

Type I Collagen: Most common (90%)—Bone (made by osteoblasts), Skin, Tendon, dentin, fascia, cornea, late wound repair (deficiency causes delayed and less strong wound healing).

1185
Q

Neurotransmitter of Parkinson disease
GABA

gluatamate

Aspartate

Dopamine

glycine

A

A) Reason

Parkinson disease Degenerative disorder of CNS associated with Lewy bodies (composed of α-synuclein—intracellular eosinophilic inclusion) and loss of dopaminergic neurons (i.e., depigmentation) of the substantia nigra pars compacta. While GABA related CNS disorder is the Huntington disease.

1186
Q

chelosis, sebhoric dermatitis and corneal vascularizatio, vitamin deficiency..?
B1

B2

B3

B6

B12

A

B) Reason

DEFICIENCY Cheilosis (inflammation of lips, scaling and fissures at the corners of the mouth), Corneal vascularization.

1187
Q

all organ systems of fetus are formed by following week
8

3

4

6

10

A

A) Reason

All major organ systems begin to develop during the embryonic period (weeks 3-8). By the end of this period, the embryo begins to look human.

1188
Q

nuclei vanish during which phase of erythrocyte development
early normoblast

late normoblast

erythroid cells

progenitor cells

in marrow

A

B) Reason

In the process of erythropoisis, red blood cells have their nuclei but as they mature their nuclei disappear in late stages of development and ultimately mature rbcs have no nuclei.

1189
Q

superior mesenteric artery infarction will not affect?
stomach

duodenum

descending colon

pancreas

spleen

A

C) Reason

All the organs except descecding colon given in scenario”s option are derivatives of foregut supplied by Superior mesenteric artery, and descending colon is supplied by inferior mesenteric artery.

1190
Q

uterine prolapse is due to chronic cervicitis with
squamous metaplasia

columner Metaplasia

damaged ligaments

damaged pelvis

nulliparus

A

A) Reason

Damage supporting structures during childbirthor chronic Metaplasiaor as a result of general poor body muscular tone mayresult in downward displacement of the uterus, calleduterine prolapse

1191
Q

protein that binds to hemoglobin
haptoglobin

Myoglobin

ferritin

thyroglobin

albumin

A

A) Reason

HaptoglobinIt is synthesized in the liver. It transports free hemoglobin inthe plasma to reticulo endothelial system.

1192
Q

which nerve is related to Superior thyroid artery?
reccurent laryngeal

external laryngeal

vagus

Superior laryngeal

hypoglossal

A

B) Reason

The superior thyroid artery curves downward to the upperpole of the thyroid gland. It is accompanied by the external laryngeal nerve, which supplies thecricothyroid muscle.

1193
Q

bilateral damage of which nerve causes death
Trigeminal

vagus

hypoglossal

acessary

facial

A

B) Reason

Bilateral damage of the vagus nerve will lead to totally lost inervation of the heart leading to cardiac arrest and death.

1194
Q

which hormone always acts through adenylate cyclate
GH

LH

TSH

insulin

glucagon

A

C) Reason

TSH increases both the synthesis and the secretion of thyroid hormones by the follicular cells via an adenylate cyclase–cAMP mechanism.■ Chronic elevation of TSH causes hypertrophy of the thyroid gland

1195
Q

A 9 year old boy with nasal lesion with lymphadenopathy and AFB positive, Dx..?
TB

leprosy

skin TB

anthrax

Sarcoidosis

A

B) Reason

Leprosy (Hansen disease):Caused by Mycobacterium leprae, an acid-fast bacillus that likes cool temperatures (infects skin and superficial nerves—“glove and stocking” loss of sensation) and cannot be grown in vitro. Causes lymphadenopathy and typical lepromatous nose lesion.

1196
Q

A person lives in deserted land n drinks, from where does maximum absorption takes place
DCT

loop of Henle

PCT

thick limb

collecting ducts

A

C) Reason

Early proximal convoluted tubule (PCT)—contains brush border. Reabsorbs all of the glucose and amino acids and most of the HCO3–, Na+, Cl–, PO43–, K+, and H2O. About 65-80 percent of these substances are reabsorbed from PCT.